[go: up one dir, main page]

0% found this document useful (0 votes)
70 views89 pages

Medicine Preprof and Prof 39 To 48

Uploaded by

Ajay
Copyright
© © All Rights Reserved
We take content rights seriously. If you suspect this is your content, claim it here.
Available Formats
Download as PDF, TXT or read online on Scribd
0% found this document useful (0 votes)
70 views89 pages

Medicine Preprof and Prof 39 To 48

Uploaded by

Ajay
Copyright
© © All Rights Reserved
We take content rights seriously. If you suspect this is your content, claim it here.
Available Formats
Download as PDF, TXT or read online on Scribd
You are on page 1/ 89

48 Preprof (MEDICINE 1) (Red Ones are Dept: key) Q6.

Clinical feature hyperkalemia include


A. Tall peaked t waves and ST depression on ECG
Q1. The volume of air remains in the lungs after forceful B. Asystole and ventricular fibrillation
maximal expiration is? C. Peripheral paresthesia
A. Forced expiratory volume D. Widening of the QRS and conduction defects on
B. Total lung Capacity ECG
C. Residual volume E. Flat T wave
D. Vital capacity (Both A and D)
E. Inspiratory Capacity
Q7. 25 years male patient sanitary worker by
Q2. 25-year-old African-American man is given occupation, smoker (10 pack-year) came to emergency
anti-malarial prophylaxis for a trip to West Africa. Over with complain of difficulty in passing urine and severe
the next week he develops Increasing fatigue. On colicky pain in left lumber region for two days U/s KUB
physical examination there are no abnormal findings. was advised with shows multiple calculi > 5mm in left
Laboratory studies show a hematocrit of 30%. ureter; What is most common type of Renal calculi
Examination of his peripheral blood smear shows red found in this age.
blood cells with numerous Heinz bodies. There is a A. Calcium phosphate
family history of this disorder, with males, but not B. Calcium oxalate
females, affected. Which of the following is the most C. Uric acid stone
likely diagnosis? D. Magnesium Aluminum phosphate
A. Beta-thalassemia E. Cysteine stone
B. Sickle cell anemia
C. Alpha-thalassemia Q8. Which of the following is a cause of extravascular
D. Hereditary spherocytosis haemolysis?
E. G6PD deficiency A. Hereditary spherocytosis
B. Paroxysmal nocturnal haemoglobinuria
Q3. 40 years old male patient presented with high grade C. Disseminated intravascular coagulation
fever 103F for 10 days and burning micturition for 8 D. Mismatched blood transfusion
days. On examination patient is tachypanic with RR of E. Haemolytic uraemic syndrome
36b/min and BP 60/40mmhg. Which of the following is
not included in the criteria for SIRS: Q9. Which of the following is the most common cause of
A. Temperature- primary glomerulonephritis?
B. Blood pressure A. IgA nephropathy
C. Pulse rate B. Anti-GBM disease
D. Leucocyte count C. AL port syndrome
E. Respiratory rate D. Post streptococcal glomerulonephritis
E. Membranous nephropathy
Q4. Most common cause of anemia in CKD is
A. Erythropoietin deficiency Q10. A 35 years old lady having H/O HTN plus allergy
B. Proteinuria on beta blockers now presented in ER with c/O:
C. Hypertension shortness of breath since 2 days that was associated
D. Diabetes with non-productive cough. His vitals were pulse:
E. Electrolyte imbalance 98b/min, temp-99F,R/R:306/min, BP was 120/80.On
chest auscultation of chest Bilateral wheezes were
Q5. The recurring distressing thoughts and repetitive audible. What can be the condition she is suffering
action such as washing hands many times is feature of from?
A. Depression A. COPD
B. Panic attack B. TB
C. Obsessive compulsive disorder C. CA Lung
D. Schizophrenia D. Bronchiectasis
E. Somatic symptom disorder E. Asthma
Q11. 50 years male patient known case of diabetes Q15. 50 years old female pt admitted via Emergency
mellitus -II came to emergency with complain of with Pallor and lethargy for the past 2 months, her blood
earache for 5 days nasal laceration and hemoptysis for tests suggestive of macrocytic anemia with one of
3 days and Hematuria for 2 days, you advised him following is most appropriate management.
steroids as treatment, What is probable diagnosis? A. Blood Transfusion + folic acid
A. Takayasu Arteritis B. Oral Prednisolone
B. Good pasture syndrome C. Vitamin B12 1/M + start folic acid oral when B/12
C. Giant cell Arteritis normal
D. Wegner granulomatous D. Vitamin B12 IV + ferrous sulphate
E. IgA nephropathy E. Folic acid oral only

Q12. 26 years old young male patient has been recently Q16. 20 year old male patient came to ED with
diagnosed with HIV. He is worried about his illness and complain of Abdominal pain and Bloody diarrhea for 2
want to get immunization. His CD4 count is 650 days and Joint pain in hip joint and ankle joint Bilaterally
cells/mm3. Which one of the following vaccination is and palpable purpuric rash on Buttocks for 6 days What
contraindicated in this patient? is probable diagnosis?
A. Yellow fever A. Porphyria cutanea tarda
B. MMR B. Good pasture syndrome
C. IV poliomyelitis C. Henoch-Schonlien purpura
D. Pneumococcus D. IgA nephropathy
E. Oral poliomyelitis E. Juvenile onset Rheumatoid Arteritis

Q13 A 34-year-old man who is HIV positive is starting Q17. Persistent issues with communication and
treatment for Burkitt's lymphoma. His chemotherapy Interaction, repetitive behavior and activities, language
regime includes cyclophosphamide, vincristine, or intellectual impairment comes under?
methotrexate and prednisolone. Around 24 hours after A. Attention deficit disorder
starting chemotherapy he becomes confused and B. Mood disorder
complains of muscle cramps in his legs. Which one of C. Autism spectrum disorder
the following is most likely to have occurred? D. Schizophrenia spectrum disorder
A. Prednisolone-induced psychosis E. Obsessive compulsive disorder
B. Hypercalcaemia
C. Methotrexate pneumonitis leading to Q18. The most common common membranous
D. Haemorrhagic cystitis leading failure to acute nephropathy in children is
E. Tumour lysis syndrome A. Asymptomatic microscopic Hematuria
B. Acute nephritic syndrome.
Q14. 60 years old male patient diagnosed case C. Nephrotic syndrome
pulmonary TB, Patient has developed ocular toxicity D. Acute renal failure
symptoms which one of the following drug is E. Nephritic syndrome
responsible for such symptoms in this
patient? Q19. A 75 years old female patient came in fir with C/O:
A. Isoniazid Shortness of breath with cough for 5 days. Her vitals
B. Rifampicin pulse:78b/min,Temp: 101F, BP:90/50mmhg, R/
C. Ethambutol R:32b/min .On examination: right sided chest
D. Moxifloxacin crepitations with dull percussion note and Increased
E. Pyrazinamide vocal resonance while left sided normal vesicular
breathing were audible. What curb score she can fit in?
A.0
B.1
C.2
D.3
E.5
Q20. Patient with chronic kidney disease has a Q25. A middle age male presented in ER with central
creatinine of 350 umol/l and has persistent proteinuria, chest pain, ECG shows STEMI raised preferred
which of the following drug is most likely the benefit to treatment should be
her proteinuria? A. Primary percutaneous coronary Intervention,
A. ACEI B: Thrombolysis
B. Aspirin C: Nitrates + anticoagulant
C. Clopidogrel D: Aspirin alone
D. Dexazosin E. E: glycoprotein lib/llia inhibitor
E. Methotrexate
Q26. 25 year old male pt known case of
Q21. Which of the following parasite is responsible for hemochromatosis his blood tests are done to assess
the disease malaria and is a leading cause of deaths when he needs venesection which one of the following
occurring in tropical and subtropical regions? blood test should be used to assess adequacy of
A. Hepatitis B Virus venesection?
B. Zika Virus A. Ferritin
C. Plasmodium falciparum B. Serum Iron
D. Pneumocystis Carinii C. Hematocrit
E. Enteric Fever D. Hemoglobin
E. TIBC
Q22. 36 years old female patient with sickle cell anemia
presented in medical ward for abdominal pain on Q27. Which of the following CANNOT be transmitted via
examination she in noted to have splenomegaly and infectious droplets?
clinically anemic, which one of following is most likely A. Rubella
diagnosis? B. Common cold
A. Salmonella infection C. Influence
B. Thrombotic crises D. Measles
C. Parvovirus infection E. None of above
D. Liver cirrhosis
E. Sequestration crisis Q28. A young female presented with tachycardia
shortness of breath, palpitations, tremors , sweating,
Q23. Crushing chest pain 15 minutes which of following she is diagnosed as generalized anxiety disorder, all of
enzyme level raises first in Myocardial infarction. the following car features of GAD except?
A. Myoglobin A. Increased blood pressure
B. CK MB B. Epigastric pain
C. Trop T C. Urinary incontinence
D. LDH D. Headache
E. AST E. Nausea

Q24. Which of the following is not a diagnostic criteria Q29. 60 year old female presented in ER unconscious
for ARDS? state having B.P 50/60mmhg her ECG shows irregular
A. Acute onset R-R interval with no p wave best treatment should be
B. B/L infiltrates on chest X-ray A. B blocker
C. Hypoxemia B. Ca channel
D. Respiratory rate>30b/min C. Digoxin
E. Pulmonary edema D. Cardioversion
E. None
Q30. A 20 year old girl presented in ER with C.O cough Q35. Must common cause of death in acute rheumatic
with sputum that is brown in color sometimes contains fever
blood. Her vitals are pulse 92b/min temp 100F R/R A. Pericarditis
32b/min B.P 100/70mmhg On examination Wheezes B. Endocarditis
are audible throughout chest On workup CBC shows C. Myocarditis
eosinophilia IgE level are raised Sputum was taken for D. Mitral stenosis
microscopy that shows growth of "Fungal Hyphae” What E. Mitral regurgitation
can be the of diagnosis?
A. TB Q36. 64 years old male patient is admitted at pulmonary
B. Lung CA ward for hemoptysis and weight loss, which one of the
C. Bronchiectasis following is significant risk factor for developing lung
D. ABPA cancer?
E. Pneumonia A. Asbestosis
B. 30 pack years of smoking
Q31. Which of the following site is involved for iron C. 20 pack years hx of smoking
reabsorption in Fe+ form? D. Fibrosing alveolitis
A. Peyer's patches of the stomach E. All of above
B. Duodenum
C. Jejunum Q 37. Which statement about schizophrenia is not true?
D. Ileum A. Benzodiazepine are treatment of choice
E. Ileocecal junction B. There is no laboratorial method for confirming
the diagnosis
Q32. 40 years old male patient smoker presented with C . The cause is multifactorial including genetics
high grade fever 102F for 3 days. Fever is associated environmental
with cough, cough is productive in nature. Chest x-ray D. There may or may not history of failure, loss or
show consolidation in right lower zone. What is most physical illness
likely causative organism: E. Has positive symptoms and negative symptoms
A. Staphylococcus aureus
B. Streptococcus pneumoniae Q38. Most common initial presentation of acute
C. Haemophilus influenza rheumatic fever is.
D. E.Coli A. Migratory polyarthritis
E. Klebsiella B. Cordite's
C. Erythema marginatum
Q33. 25 years old female patient presented with fever D. Subcutaneous nodules
101F for 8 days associated with headache and pain E. Sydenham chorea
around eyeballs and muscle pain. She is living in a
unhygienic environment. On examination there is Q39. Which cardiomyopathy is most common
Maculopapular rash all over body. CBC report shows A. Dilated cardiomyopathy
thrombocytopenia which of the following is most likely B. Restrictive cardiomyopathy
diagnosis C. Hypertrophic cardiomyopathy
A. Malaria D. Tamponade
B. Typhoid E. Peripartum cardiomyopathy
C. Dengue (Verify)
D. Infectious mononucleosis
E. Measles

Q34. Indication for thrombolysis


A. CNS neoplasm
B. Ischemic stroke
C. Bleeding diathesis
D. Aortic dissection
E. STEMI
Q40. 32 year old male patient presented with high Q44. 40 years old male patient known case of diabetes
grade fever for 20 days associated with rigor and chills. and hypertension presented with chest pain that is
On examination there is blackish discoloration of skin, crushing in character and lasts more than 30 minutes.
hepatomegaly and massive splenomegaly. CBC shows Chest pain is associated with increased sweating and
pancytopenia. The most likely diagnosis is: vomiting. ECG shows ST segment elevation in leads 2,3
A. Kala azar and AVF. ECG findings are suggestive of:
B. Infectious mononucleosis A. Anterior wall MI
C. HIV B. Inferior wall MI
D. Measles C. Anteroseptal wall MI
E. CML D. Posterior wall MI
E. Anterolateral wall MI
Q41. 38 Years old male patient who is known case of
bronchiectasis, which one of the following organism is Q45. 60 years old male patient known case of diabetes
most likely to be seen from his sputum? mellitus presented with shortness of breath and anuria,
A. Klebsiella Laboratory investigations shows Cr 6.5 Urea 130 and
B. Haemophilus influenza potassium 6.7 meq/l. The most specific ECG finding in
C. Pseudomonas aeruginosa this patient will be:-
D. Strep pneumonia A. U waves
E. E-coli B. Inverted T waves
C. Tall T waves
Q42. 20 years old female patient presented with joint D. St segment elevation
pain for 1 week that is migratory in character and E. Peaked P wave
relieved by aspirin. She has had sore throat 2 weeks
back. On cardiac auscultation there is mid diastolic Q46. Which one of the following is common
murmur at apex. She has been suspected for dermatologic feature of Sarcoidosis in females?
Rheumatic fever. Which of the following is not included A. Erythema Marginatum
in major criteria for rheumatic fever: B. Erythema nodosum
A. Migratory polyarthritis C. Club pernio
B. Carditis D. Necrobiosis lipoidica
C. Sydenham chorea E. Rheumatoid nodules
D. Erythema marginatum
E. Fever Q47. 30 years old female patient presented with high
grade fever for last 2 weeks. On examination there is
Q43. 38 years old male pt nonsmoker is diagnosed as anemia, clubbing. splenomegaly. On auscultation there
emphysema, on further investigation alpha 1 antitrypsin is audible murmur. Which of the following will be the
deficiency is found, which one of following is main role most specific diagnostic test in this patient:
of alpha 1 antitrypsin deficiency? A. ECG
A. Phosphodiesterase inhibitor B. Echocardiography
B. Protease inhibitor C. Blood culture
C. Trypsin activator D. Cardiac catheterization
D. Nicotinic receptor activator E. Chest x-ray
Q48. 40 years old female patient presented with Q53. Which of following can cause pure motor neuron
shortness of breath. On examination there is bibasilar neuropathy.
inspiratory crackles. Chest x ray shows left atrium and A. HIV neuropathy
left ventricular enlargement. On the basis of above B. Paraneoplastic neuropathy
findings mitral regurgitation is suspected. Which of the C. Diabetic neuropathy
following murmur will be audible in this patient D. Lead poisoning
A. Pansystolic murmur E. Alcohol induce nephropathy
B. Ejection systolic murmur
C. Early diastolic murmur Q54. In Wilson disease following finding is not present?
E. Flow murmur A. Autosomal recessive disease
D. Late diastolic murmur B. Excessive deposition of copper in brain and liver
C. Urinary copper excretion is high
Q49. In idiopathic thrombocytopenic purpura what are D. Serum ceruloplasmin is low
the autoantibodies most commonly directed at? E. Hepatic copper concentration is low
A. Platelet activating factor
B. Glycoprotein IIb/Illa complex Q55. Most unwanted side effect of sulfonylureas is?
C. ATP receptor A. Weight loss
D. Anti-thrombin III receptor B. less energy
E. ADP receptor C. it has long duration of action
D. Hypoglycemia
Q50. A patient presented with tachycardia, dyspnea, E. it increase micro vascular risk associated with
palpitations diagnosed as panic disorder, the first line of diabetes
treatment.?
A. Antidepressants Q56. What is most dominant mechanism of
B. Benzodiazepine gastroesophageal reflux disease?
C. Barbiturates A. Increased transient lower esophageal relaxations
D. Beta blocker B. Swallow induced lower esophageal relaxations
E. Antihistamine C. Absent basal lower esophageal sphincter pressure
D. Straining
(MEDICINE 2) E. Acid hypersecretion

Q51. Which of the following features are not typically Q57. 16 years old girl presented with complain of loose
seen in a patient with adult onset Still's disease? motion for 2 years, bloating, abdominal pain and iron
A. Maculopapular rash deficiency anemia what is your diagnosis?
B. RA factor A. Hyperthyroidism
C. Pyrexia B. Chronic pancreatitis
D. High ferritin level C. Celiac disease
E. Lymphadenopathy D. Tropical sprue
E. Lymphoma
Q52. 45-year-old female with a history of rheumatoid
arthritis presents to the Emergency Department with a Q58. 20 Years old female patient presented with
two day history of a hot, painful, swollen right elbow complain of abdominal pain, fever and bleeding per
Joint. What is the most appropriate management? rectum and arthralgia. What is your diagnosis?
A. Start infliximab A. Crohn's disease
B. Oral high-dose prednisolone B. Amoebic colitis
C. Joint Aspiration C. Ischemic colitis
D. Short course of methotrexate D. Ulcerative colitis
E. Methylprednisolone injection E. Hemorrhoids
(Verify)
Q59. Which of the following is the most common cause Q64. 25 years old young male patient presented with
of colonic obstruction? abdominal pain and vomiting. He is drowsy and
A. Crohn's stricture confused. His blood glucose level is 500 mg/dl and
B. Diverticulitis ketones are +2. What change occur in anion gap in this
C. Intussusception patient:
D. Ventral hernia A. Normal anion gap
E. Adenocarcinoma B. Increased anion gap
C. Decreased anion gap
Q60 50 years old patient came with complain of D. Initially decrease then increases
epigastric pain that is aggravated by food intake, having E. Initially increase then suddenly decreases
history arthralgia taking NSAIDS. What is diagnosis?
A. Functional dyspepsia Q65. A 40 Y/O female K/C of RA since 15 years not
B. Gastro esophageal reflux disease controlled since last 6 years presented with difficulty in
C. Peptic ulcer disease climbing stairs and combing her hair. She has
D. Carcinoma of stomach cushingoid features. Serum CPK normal. What is most
E. Gastrinoma probably cause of her complains?
A. Cushing Disease
Q61. 40 years old male patient presented with B. Addison disease
palpitation and episodic hypertension. On CT abdomen C. Polymyositis
there is a mass in right adrenal gland which of the D. SLE
following finding is consistent with patient's symptoms: E. Steroid induced myopathy
A. Increased methylmalonic acid
B. Increased serum vanillylmandelic acid Q66. Medical Treatment for Pheochromocytoma
C. Increased urinary vanillylmandelic acid include?
D. Increased homocysteine level A. Dopamine agonist
E. Decreased serum vanilmandelic acid B. Dopamine Antagonist
C. Alpha Blocker followed by Beta blocker
Q62. 30 years old female patient presented with fever D. Beta blocker followed by Alpha Blocker
and swelling in neck that moves with deglutition. The E Beta blocker only
swelling is painful. What will be the initial step in the
management of this patient? Q67. The factor which indicates the worst prognosis in
A. Thyroxine cirrhosis includes?
B. NSAIDS A. Presence of HB e antigen
C. Propylthiouracil B. Prolonged prothrombin time
D. Carbimazole C. Raised urea
E. Steroids D. Presence of HBs antigen
E. Decreased platelet count
Q63. 20 years old female patient presented with weight
loss despite increased appetite, there is history of Q68. 50 years old female patient presented with
oligomenorrhea since last 8 months. On examination complain of chest pain, retrosternal burning and
there is large palpable thyroid gland without any numbness, tingling of feet and bluish discoloration of
nodules. Radioactive iodine uptake in this patient will fingers of hand and feet what is diagnosis?
be: A. Diffuse esophageal spasm
A. Patchy increased uptake B. GERD
B. Decreased uptake C. Achalasia
C. Generalized increased uptake D. Scleroderma
D. No any change in uptake E. NUT Cracker esophagus
E. Generalized decreased uptake
Q69. 40 years old female patient presented with Q75. In a patient with fever,. Right upper quadrant pain,
amenorrhea for last 8 months and galactorrhea. Her tenderness, hepatomegaly, weight loss, CBC Shows
serum prolactin level is 500mu/l. What is the most neutrophilic leukocytosis. The most probably diagnosis.
common pathological cause of hyperprolactinoma? A. Acute hepatitis
A. Drug induced B. Amoebic liver abscess
B. CRF C. Chronic liver disease
C. Pituitary adenoma D. Malaria
D. Hypothyroidism E. Alcoholic fatty liver disease
E. Stress
Q76. 40 years old male patient present to the
Q70. The serological pattern in acute hepatitis will not Emergency Department with one month history of
be? swollen painful left knee aspirated joint fluid shows
A. Hbs antigen +ve calcium pyrophosphate crystals. His RBS is raised.
B. Anti Hbc +ve Which of following blood tests is most revealing an
C. Hbcag +ve underlying cause?
D. Anti HBe +ve A. ACTH
E. anti Hbs negative B. Transferrin Saturation
C. ANA
Q71. All are the causes of acute liver failure except? D. Serum ferritin
A. Decrease level of albumin E. LDH
B. Shock
C. Heat stroke Q77. A 30 years old presented with.complain-of loose
D. acetaminophen poisoning motions for 2 years, bloating, abdominal pain and iron
E. Anti tuberculous drugs deficiency anemia, what is your diagnosis
A. Hyperthyroidism
Q72. In a patient with fever upper right quadrant pain, B. Chronic pancreatitis
tender hepatomegaly and jaundice, the lab findings C. Celiac disease
which favor the alcoholic liver disease is? D. Tropical sprue
A. Deranged creatinine E. Lymphoma
B. Elevated bilirubin
C. AST>ALT by a factor of 2 or more Q78. Which one of the following is most recognized as a
D. ALT is >AST by a factor of two or more potential complication in a patient with ankylosing
E. Deranged PT, INR spondylitis?
A. Heart block
Q73. A patient known case CLD present with abdominal B. Aortic stenosis
distension. On Investigation the serum ascitic albumin C. Achalasia
gradient is >1.1 Indicates? D. Diabetes Mellitus
A. Spontaneous bacterial peritonitis E. Bronchiectasis
B. Exudative fluid
C. Hypoalbuminemia Q79. A 30 years old male patient presented with altered
D. Transudative fluid level of sensorium since morning. He has had previous
E. Non portal cause of ascites 4 episodes of ALOC in last 3 months. His RBS is 60
mg/dl, BP 70/40 mmhg RR 18/min, Temp 97F. His
Q74. In a patient with decrease glucuronyl transferase, electrolytes are Na 129, K 5.2, CI 98. What will be the
decrease bilirubin uptake having mild jaundice, most specific diagnostic test in this patient?
probable diagnosis is? A. 24 hour urinary cortisol
A. Gilbert syndrome B. Short synacthin test
B. Criglernajjar type I C. Low dose dexamethasone test
C. Criglernajjar type II D. High dose dexamethasone test
D. Dubinjohnson syndrome E. Salivary cortisol
E. Rotors syndrome
Q 80. Reactive arthritis is associated with which one of Q86. A previously healthy primigravida patient Suffered
the following HLA antigens? from gestational diabetes.what is the main mechanism
A. HLA-B27 of gestational diabetes?
B. HLA-A3 A. Increase craving of sweets causing hyperglycemia
C. HLA-DR4 B. Increase cortisol level causing hypernatremia
D. HLA-B5 C. Inability to increase insulin secretion to compensate
E. HLA-DR pregnancy induced insulin resistant
D. Increase gluconeogenesis
Q81. The most important serological marker in E. Decreased activity level of pregnant female
rheumatoid arthritis?
A. RA factor Q87. Following are the side effects of insulin therapy
B. ANA except?
C. Anti-Ro A. Hypoglycemia.
D. Anti-ccp B. Weight gain
E. IL-7 C. Peripheral edema
D. Lipodystrophy
Q82. Tophi is feature of which disease? E. Renal failure
A. SLE
B. RA Q88. A 66 years old healthy male presented with
C. Gout headache and confusion over the course of 2 days he
D. Systemic Sclerosis had history of fever 3 days on examination sign of
E. MCTD meningeal irritation positive. CSF DR revealed WBC
2000, 90% neutrophils and 60 RBS. Which of following
Q83. Which of the following will most likely distinguish is the most common cause of bacterial meningitis.
pseudo gout from gout? A. Mycobacterium
A. Acute onset B. Haemophilus influenza
B. Positive birefringent crystals C. Streptococcus pneumoniae
C. Involvement of single joint D. Herpes simplex virus
D. Involvement of large joint E. Toxoplasmosis gonads
E. Involvement of multiple joints
Q89. 30 years old female is diagnosed as case of
Q84. 25 years old medical student presented in restless leg syndrome. Which of following is metal is
emergency department with altered sensorium on commonly deficient in these patients.
presentation he was drowsy with blood pressure 90/60, A. Mercury
pulse 100/min, R/R 28, ABGs show PH of 7.1, HCO3-10 B. Iron
serum potassium 3.0 serum sodium of 134 his RBS was C. Copper
288mg/dl, most likely diagnosis is ? D. Lead
A. Metabolic acidosis E. Magnesium
B. Hyperglycemic hyperosmolar state
C. Diabetic ketoacidosis Q90. 30 years old male with no prior history presented
D. Aspirin poisoning to er with history of sudden severe band like headache
E. heat exhaustion associated with nausea. Patient had history of disturbed
sleep cycle due to bury schedule. This type headache is
Q85. Most common cause of death in diabetic patient called?
is? A. Tension headache
A. Cancer B. Migraine headache
B. Renal failure C. Cluster headache
C. Infections D. Pain due in arterite
D. Cardiovascular disease E. Healtsche due subarachnoid hemorrhage
E. Diabetic ketoacidosis
Q91. The broca's area located in which lobe Q95. 65 years old male has history of right sided severe
A Temporal headache which is not relieved after taking pain killers
B. Parietal on examination he is vitally stable and has tenderness
C. Frontal over right temporoparietal region is positive right
D. Occipital temporal artery is palpable. Investigations reveals
E. None of these raised ESR CT scan brain normal. Most appropriate
step is.
Q92. Young boy with no prior co morbids presented with A. MRI brain plain
sudden onset multiple episodes of fits in the form of B. MRI brain contrast
tonic clonic movements of his extremities with urinary C. CSF analysis
incontinence along with tongue bite. Which of following D. CT scan brain e with contract
term describe this type of seizure. E. Temporal artery biopsy with IV Steroids
A. Simple partial seizure
B. Generalized seizure Q96. 40 years old male presented with history a fever
C. complex partial seizure for last 3 weeks followed by altered sensorium for 3
D. Absence seizure days. CT scan showed cerebral edema so CSF DR was
E. Secondary epilepsy done that showed glucose 30 mg/dl protein 70mg cells
70% with lymphocytes predominant the patient should
Q 93. 65 years old female presented with tremor of be managed on line
hands which feature suggest a diagnosis of essential A. Bacterial meningitis
tremor rather than parkinson's disease.? B. Viral meningitis
A. Difficulty in initiating movement C. Tuberculosis meningitis
B. Tremor worse following alcohol D. Cryptococcus meningitis
C. Postural instability E. Aseptic meningitis
D. Unilateral symptoms
E. Tremor is worse when arm is out stretched Q97. Late complication of Organophosphate poisoning
which developed 2 to 3 weeks after ingestion of
Q94. 25 years old male presented to emergency Organophosphate
department for weakness of lower limbs for last 24 A. Coma
hours. He had history of gastroenteritis one week back. B. Seizures
On examination he was vitally stable with GCS of 15/15 C. Intermediate Syndrome
power in lower limb was 3/5 with impaired reflexes and D. Respiratory Depression
impaired sensation. Investigations reveals hb 12.5g/di E. Organo-Phosphate induced Polyneuropathy
tic8900 plt145000. His random blood sugar was
90mg/dl serum creatinine was 0.8mg/dl serum sodium Q98. A 57-year-old woman was admitted with acute
134mmol/L serums Potassium 3.5mmol/L. What is most abdominal pain and a history of alcohol excess. She
likely diagnosis. was sweaty, her heart rate was 115 beats/minute,
A. Hypokalemic periodic paralysis serum amylase was 1250 U/L (normal range 60-180
B. motor neuron disease U/L). She was diagnosed with acute severe pancreatitis.
C. Transverse myelitis Her Apache Il score was 9. Which of the following would
D. GuillianBarre syndrome be the most appropriate for nutritional support?
E. Potts disease A. Elemental diet
B. Nasogastric feeding
C. Nasojejunal feeding
D. Oral nutrition with nutritional supplements
E. Parenteral nutrition
(Verify)
Q99. Anti Dote for Warfarin Over Dose ? Prof….
A. Fresh Frozen Plasma
B. Protamine sulphate
C. Vitamin K
D. Factor 8
E. Factor 10

Q100. Flumazenil (Anti Dote) of Benzodiazepines Is


contraindicated in All of the following Except.?
A. Benzodiazepines ingestion with TCAS
B. Benzodiazepines ingestion with Cocaine
C. Benzodiazepines ingestion Alone
D. Benzodiazepines ingestion with Alcohol
E. Epilepsy
47 Preprof (MEDICINE 1) 6. In presence of Anaemia, the ability to produce
adequate circulating red blood cell is best measured
1. Investigation of the choice in patient with typhoid by?
fever in 1 week is? A. WBC count
A. Widal test B. Serum iron binding capacity
B. Blood Culture C. Reticulocyte count
C. Typhoid test D. Total eosinophil count
D. Urine Culture E. Total Lymphocyte count
E. Stool Culture
7. 18 years male with sickle cell disease present with
2. A 40-year-old male presented with a 24-hour history osteomyelitis which organism is responsible?
of palpitations. An ECG revealed atrial fibrillation with a A. Mycobacterium tuberculosis
ventricular rate of 130 beats per minute, drug is most B. Salmonella
likely to restore sinus rhythm? C. Pseudomonas
A. Adenosine D. Group A streptococci
B. Bisoprolol E. E.coli
C. Digoxin
D. Flecainide 8. Auer rods an eosinophilic like inclusion bodies are
E. Verapamil pathogenomnic of?
A. CML
3. With regard to cardiac troponins, which ONE of the B. AML
following statements is correct? C. ALL
A Elevated plasma troponin concentrations are specific D. Myelofibrosis
markers of ischaemic heart disease E. CLL
B. Plasma troponin concentrations are typically elevated
three weeks after an acute myocardial Infarction 9. Immediate treatment of Tension Pneumothorax is?
C. Plasma troponin concentrations are typically reduced A. Chest intubation
in subjects with atrial fibrillation B. Needle decompression
D. The specificity of troponins for myocardial injury is C. Bronchodilators
similar to that of creatine kinase- MB D. Steroids
E. The clinical role of troponins is to rule out acute E. Antibiotics
myocardial infarction in patients presenting with chest
pain 10. Which among following bronchogenic carcinoma is
common in non-smoker patient
4. A 25-year-old male intravenous drug user presented A. Adenocarcinoma
with an injection site abscess. On examination, his B. Large cell carcinoma
temperature was 38.5°C and there is a pansystolic C. Broncho alveolar carcinoma
murmur at apex radiate to axilla. Which organism is D. Squamous cell carcinoma
most likely to be cultured from the blood? E. Small cell carcinoma
A. Streptococcus bovis
B. Klebsiella pneumoniae 11. Presence of pus in pleural space is termed as?
C. Staphylococcus aureus A. Para pneumonic effusion
D. Staphylococcus epidermidis B. Chylothorax
E. Streptococcus viridans C. Emphyema
D. Exudative effusion
5. Cause of death in established tetanus is ? E. Hemorrhagic effusion
A. Spinal cord compression
B. Septic shock
C. Renal failure
D. Respiratory failure
E. Pulmonary embolism
12. In community the most common disorder over age 18. Immediate treatment of hyperkalemia is?
of 65 years is? A. Intravenous 10% calcium gluconate 10ml over 10min
A. Alzheimer's disease B. Intravenous 5% dextrose 250 ml/hour
B. Late onset Schizophrenia C. Intravenous 0.9% saline at 500 ml/hour
C. Mood disorders D. Oral calcium resonium
D. Delirium E. Loop diuretics
E. Vascular dementia
(Verify) 19. Which one of the following is most important
complication during the over correction of sodium?
13. Normal anion gap metabolic acidosis occurs A. Central pontine myelinolysis
in? B. Dehydration
A. Lactic acidosis C. Hypokalemia
B. Alcoholic ketoacidosis D. Hyperkalemia
C. Diabetic ketoacidosis E. Renal failure
D. Renal tubular acidosis
E. Methanol overdose 20. Which of the following is the characteristic feature of
interstitial lung disease?
14. Acute renal failure may be distinguished from A. Increased vital capacity
chronic renal failure by which of the following? B. Rapid response to bronchodilators
A. An increased urinary Na excretion C. Restrictive pattern on pulmonary function test
B. Left ventricular hypertrophy on the ECG D. High prevalence in smokers
C. Hypophosphatemia E. Decreased FEV1
D. Renal size on ultrasound scan
E. Hyperkalemia 21. With regard to cardiac troponins, which ONE of the
following statements is correct?
15. Pulmonary hypertension is diagnosed when mean A. Elevated plasma troponin concentrations are specific
pulmonary artery pressure at rest is? markers of ischaemic heart disease
A. 15 mmHg B. Plasma troponin concentrations are typically elevated
B. 30 mmHg three weeks after an acute myocardial infarction
C. 25 mmHg C. Plasma troponin concentrations are typically reduced
D. 30 mmhg in subjects with atrial fibrillation
E. 35 mmHg D. The specificity of troponins for myocardial injury is
similar to that of creatine kinase- MB
16. Which of the following is common symptom of E. The clinical role of troponins is to rule out acute
schizophrenia? myocardial infarction in patients presenting with chest
A. Hypertension pain
B. Hallucination
C. Headache 22. Left axis deviation is seen on the ECG in which of
D. Fear the following condition?
E. Decreased mood A. Ventricular septal defects
B. Ebstein's anomaly
17. A 25-year-old boy presents with frank hematuria. C. Cor Pulmonale
His symptoms had started 2 weeks after developing a D. Patent ductus arteriosus
sore throat. His blood pressure was 138/88 mmHg. E. Tetral Tetralogy of Fallot (Verify)
Urinalysis was positive for blood (4+) and protein (2+).
What is the most likely diagnosis? 23. In the diagnosis of rheumatic fever, which of
A. Poststreptococcal Glomerulonephritis the following will be helpful?
B. IgA nephropathy A. Generalized macular-papular rash
C. Lupus nephritis B. ASO titre of less than 1.200.
D. Urinary tract infection C. Migratory Polyarthritis.
E. Goodpasture syndrome D. Splinter hemorrhages.
E. Staphylococcus aureus grown on throat culture
24. During auscultation of the heart you discover & wide
fixed splitting of the second heart sound. In which of the 30. A 60 year old male is Investigated for recurrent
following condition does this occur? episodes of gout. On examination, she looked plethoric
A. An uncomplicated ASD and has a 6 cm splenomegaly. Labs shows Hb 18.9
B. Aortic stenosis. gm/dl Hct 0.612, platelet count 468 x10^9 ESR 1mm/1st
C. Constrictive pericarditis hour coagulation screen normal What is the diagnosis?
D. Fallot's tetralogy A. B cell lymphoma
E. Right Bundle Branch Block B. Multiple myeloma
C. Polycythemia rubra vera
25. Diagnostic feature of hypertrophic cardiomyopathy D. AML
is: E. Sickle cell disease
A. Collapsing pulse
B. Prominent V-wave in JVP 31. Which of the following is a recognized cause of
C. Triple apical impulse acute renal failure?
D. Ejection click A. Burns
E. Systolic murmur that increase intensity with squatting B. Dermatomyositis
C. Duchenne muscular dystrophy
26. A 15 year old lady has had treatment with D. Penicillin therapy
penicillamine for rheumatoid arthritis. She presents with E. Alport's Syndrome
weakness and pallor claiming that it may be a side
effect of the drug. Her Hb is 5 g/dl, WBC is 2 x 10^9 32. Finding eosinophiluria in patient with acute renal
/L,platelet count is 15 x 10^9, INR is 1.0 and APTT is failure is suggestive of ?
27s .What is the most likely diagnosis? A. Acute allergic interstitial nephritis
A. AML B. Acute glomerulonephritis
B. myelodysplasia C. Acute tubular necrosis
C. Folate deficiency D. Acute uric acid nephropathy
D. Aplastic anemia E. Hypovolemia
E. CML
33. Which neurotransmitter is often associated With
27. Hallmark of COPD is airflow obstruction is depression?
evidenced by? A. Serotonin
A. Increased FEV1/FVC B. Dopamine
B. Increased FEV1 C. Acetylcholine
C. Decreased FEV1/FVC D. GABA
D. Decreased residual volume E. Glutamate
E. Normal ABGS
34. Gold standard investigation in diagnosis of
28. Incubation period of Rabies is ? pulmonary thrombo embolism is
A. 24 hours A. Ventilation perfusion scan
B. 2 to 3 days B. Pulmonary angiography
C. 2 to 3 weeks C. Helical spiral CT scan
D. 3 to 7 weeks D. Trans esophageal echocardiography
E. 6 to 12 months E. MRI chest
(Verify)
35. 40 years old man suddenly develops shortness of
29. Which one among following is transmitted by faeco breath and tachypnea following leg surgery, Chest X-ray
oral route ? is normal. The most common underlying cause is?
A Hepatitis A A. Asthma
B. Hepatitis C B. Pulmonary embolism
C. Tuberculosis C. Fat embolism
D. Scabies D. CCF
E. Brucellosis E. ARDS (Verify)
(MEDICINE 2) 6. Regarding Rheumatoid Arthritis, which of following
suggest an adverse prognosis?
1. A lady presented with non-progressive dysphagia A. Metastatic disease
only for solids Barium study showed proximal B. Polymyalgia rheumatica
esophageal dilatation with distal constriction The most C. Spinal stenosis
likely diagnosis is D. Spondylosis of the lumbar spine
A. Achalasia cardia E. Stress fracture of the pelvis
B. Carcinoma esophagus
C. esophageal spasm 7. 80-year-old man previously fit and healthy presents
D. Lower esophageal img with severe flinging movements of the left upper limb,
E. Peptic Stricture body and lower limbs. Where is the neurological lesion?
A. Caudate nucleus
2. What is the most common cause for Budd Chiari B. Globus pallidus
syndrome C. Ipsilateral thalamus
A. Nephrotic syndrome D. Substantia nigra
B. PNH E. Subthalamic nucleus
C. Polycythemia rubra vera
D. Right ventricular failure 8. A young girl presents with abdominal pain and a
E. Valve in hepatic veins recent change in bowel habit, with passage of mucus in
stool. There is no associated blood in stool and
3. 75 year-old male is diagnosed with Lewy body symptoms are increased with stress. The most likely
dementia. Which one of the following drugs would be diagnosis is:
contraindicated for this patient? A. Irritable bowel syndrome
A. Chlormethiazole B. Ulcerative colitis
B. Donepezil C. Crohn's disease
C. Haloperidol D. Amebiasis
D. L-Dopa E. Celiac disease
E. Selepine
9. Patients with haemochromatosis and end organ
4. A 40-year-old female who has been prescribed damage are at risk of which infection:
thyroid replacement therapy has routine thyroid function A. Staphylococci
tests. On examination she appeared clinically euthyroid B. Streptococci
with no abnormal findings, Her TFTs revealed: C. Yersinia
TSH 3.2 mU/L (0.35-6.0) D. Shigella
Total T4 20 nmol/L (55-144) E. Salmonella
Free T4 2.6 pmol/L (9-24)
Total T3 2.5 nmol/L (0.9-2.8) 10. A 17 year old female who is 16 weeks pregnant
Which one of the following statements is correct? reports that her elder brother has vitamin D resistant
A. Her thyroid hormone replacement is adequate rickets. What is the most likely mode of Inheritance of
B. Investigation of pituitary function is required this condition?
C. She has tertiary hypothyroidism A. Autosomal dominant
D. She has a thyroiditis B. Autosomal dominant with incomplete penetrance
E. She has sick euthyroid syndrome C. Autosomal recessive
D. X-Inked dominant
5. The Hormone secreted by alpha cells of pancreas E. X-Linked recessive
that raises blood sugar when levels is low:
A. Cortisol
B. Epinephrine
C. Glucagon
D. Growth Hormone
E. Insulin
11. When hyperglycemic is seen following an episode of 15. A 40 year old woman with a 12 month history of
hypoglycemic in type 1 DM the condition known as? vague abdominal discomfort. No vomiting, diarrhea or
A. Cushing Syndrome constipation. No weight loss. LFTs normal. There are
B. Down Phenomen 5-6 oval hypoechogenic lesions in the liver, contrast
C. Neuropathy enhancing. Lesions range from 3-7cm in size, are
D. Retinopathy uniform and have no fluid levels, Kidneys are normal.
E. The Somogy Phenomen The most likely diagnosis ist
A. Abscesses
12. A 25 year old woman without any past medical B. Benign cysts
history presents with a 3 month history of arthralgia. C. Hepatoma
She had no past medical history of note. Examination of D. Hydatid disease
swelling of the distal interphalangeal joints of the middle E. Metastatic disease
and beg fingers of the hand and wrist on the right plus a (Verify)
Swollen left ankle Investigations show. ESR 40mm/hr
(0-10). Most likely diagnosis? 16. Which of the following techniques would be most
A. Acute exacerbation of osteoarthritis useful in the differential diagnosis between ectopic
B. Psoriatic arthropathy Cushing's syndrome and pituitary dependent Cushing's
C. Rheumatoid arthritis disease:
D. Reactive arthritis A. Urine free cortisol
E. SLE B. High dose Dexamethasone suppression test
C. ACTH concentrations
13. A 73-year-old man presents with an abrupt onset of D. Inferior petrosal sinus sampling
double vision and left leg weakness. Examination shows E. CRF test
weakness of abduction of the right eye, right-sided facial
weakness affecting upper and lower parts of the face, 17. In a patient with ascending paralysis there is
He also has a left hemiparesis. Where lesion? subsequently respiratory muscle Involvement. CSF
A. Left frontal lobe examination showed albuminocytological dissociation.
B. Left lateral medulla Treatment to be given is:
C. Right corpus striatum A. Oral Prednisolone
D. Right midbrain B. IV methylPrednisolone
E. Right pons C. IV immunoglobulins
D. Cyclosporin
14. An elderly patient presents with a prolonged history E Dexamethasone
of weakness and lethargy. On examination he is found
to be anemic and stool is positive for occult blood. 18. Factors seen to play role in development of type Il
Which of the following is the investigation of choice? Diabetes include
A. Barnum enema A. Autoimmune
B. Barium meal B. Childhood resistance
C. Barium swallow C. Enzyme deficiency
D. Colonoscopy D. Liver disease
E. CT abdomen E. Weight and Heredity

19. Chondrocalcinosis is seen in which of the following


conditions
A. Hypervitaminosis D
B. Hypoparathyroidism
C. Ochronosis
D. Rickels
E. Hyperthyroidism
20. A 33 year old female is admitted with erythema 24. A man presents with sudden left neck pain, left
multiforme and erythematous lesions of the mouth and Horner's and right hemiparesis. The most likely cause
eyes. Which one of the following drugs may account for is:
her presentation? A. left internal carotid artery dissection
A. Diazepam B. left vertebral artery dissection
B. Fluoxetine C. middle cerebral artery CVA
C. Mebeverine D. Posterior inferior cerebellar artery lesion
D. Oral contraceptive E. Demyelination
E. Sulphasalazine
25. A 35 year old woman presented with a five year
21. A patient presented with pain abdomen, ataxia, history of weight gain associated with one year History
constipation Peripheral blood examination showed of amenorrhea Over this she had also noticed hirsutism
basophilic stippling RBC's. He is suffering from and had been trying to conceive. On examination she
poisoning due to had a BMIl of 32 kg/m^2 a pulse was 84 beats per
A. Iron minute, and blood pressure of 154/100 mmHg Features
B. Lead suggestive of Cushing's syndrome were also noted.
C. Cadmium Which of the following would be the most useful
D. Arsenic investigation?
E. Cadmium A. 24 hours urinary free cortisol concentration
B. Combined 9am ACTH concentration and serum
22. 4 year old man with Crohn's disease presents with cortisol concentration
fever, rigors and right lower quadrant pain His C. Midnight cortisol concentration
medications include prednisone 10mg per day and D. Serum sodium and potassium concentrations
sulphasalazine. On examination, he is febrile with temp E. The long overnight dexamethasone suppression test
of 37.6 degrees and has exquisite tenderness in the
right lower quadrant but no generalized peritonitis 26. Most common cause of death in diabetes
Investigations; CT abdomen reveals a right sided mass A. cardiovascular disease
lesion 5-4cm in diameter, containing pus and fluid. What B. Cancer
is the best initial management for this man? C. Diabetes Ketoacidosis
A. IV antibiotics D. Infection
B. IV hydrocortisone E. Renal failure
C. Laparotomy
D. IV hydrocortisone alone 27. 58 year old woman presents with back pain. X Ray
E. Change sulphasalazine to another medication shown Interpreted as collapse of one vertebra &
multiple lytic lesions in other Vertebrae or one dark
23. 20 year old boy, presented with severe vertebrae with other white. Serum electrophoresis
hematemesis. On examination there was no normal, low Immunoglobulins, no paraproteins in urine
hepatomegaly, mild splenomegaly present. On & protein. What is the most likely cause?
endoscopy esophageal varices were seen. What is the A. Osteoporosis
most probable diagnosis: B. Myeloma
A. Cirrhosis liver C. Lymphoma
B. Budd Chiari syndrome D. Breast cancer
C. Non cirrhotic portal fibrosis E. Vitamin D deficiency
D. Veno occlusive disease
E. Thrombocytopenia
28. A 10 year old girl is seen in clinic as she is 32. A young man is found unconscious and pulseless
concerned due to areas of hair loss on the scalp. Past with syringes on the ground around him. He is
medical history includes atopic eczema and she has a resuscitated at the stone with CPR and naloxone and
number of depigmented areas on her hands. What is taken to the hospital. 24 hours later in hospital he is
the most likely diagnosis? found to have abnormal LFTS: AST 10000, GOT 12000,
A. Alopecia areata BMI 16 and INR 1.0 When repeated the next day, the
B. Hypothyroidism blood tests were the same, What is the most likely
C. Seborrheic dermatitis cause for his abnormal LFTs7
D. SLE A. Acute Hepatitis A
E. Trichotillomania B. Hepatitis C
C. Ischemic hepatitis
29. A patient was brought with history of pyrexia D. Drug induced ever dysfunction
contracted pupils Hypotension cyanosis progressing to E. Resuscitation induced liver dysfunction
coma is suspected to be suffering from poisoning due to
A. Cannabis 33. Serum biochemistry of a 69 year old man revealed
B. Dhatura calcium of 1.58 mmol/L and phosphate of 0.55 mmol/l
C. Phenobarbitone with an alkaline phosphatase of 450 IU/Ll Which of
D. Diphenhydramine following most suits with above serum biochemistry?
E. Cocaine A. Osteoporosis
B. Osteomalacia
30. A patient suffers from tonic-clonic and absence C. Paget's Disease
seizures. They are no longer able to tolerate valproate. D. Secondary Hyperparathyroidism
Your next treatment: E. Renal failure
A. Lamotrigine
B. Vigabatrin 34. Young female, 20 weeks pregnant, presents with
C. Carbamazepine chorea. She has a past history of nasal perforation.
D. Gabapentin ANA negative. Next investigation:
E. Ethosuximide A. Cerebral angiograms
B. ANCA
31. A lady with a ten year history of esophagitis treated C. dsDNA
with omeprazole presents complaining of Intermittent D. ASOT
dysphagia for solids. Previous endoscopy has shown E. Antiphospholipid
ulcerative oesophagitis. There has been a history of (In key its B)
weight loss. The next most appropriate test would be?
A. Manometry 35. A 22 year old female, 10 days postpartum, History
B. Endoscopy of headache and seizure, right hemiplegia, confusion.
C. Gastro pH monitoring Best test for diagnosis:
D. Barnum Swallow A. CT with contrast brain
E. Barium meal B. MR venography brain
C. Blood film
D. MRI Brain
E. EEG
47 Prof (MEDICINE 1) 9. 40 years old male patient known case of diabetes
and hypertension presented with chest pain that is
1. Decreased PR interval crushing in character and lasts more than 30 minutes.
Ans WPW Chest pain is associated with increased sweating and
vomiting. ECG shows ST segment elevation in leads 2,3
2. Patient receiving ATT, develops gout. Drug which and AVF. ECG findings are suggestive of:
should be stopped? A. Anterior wall MI
A. Isoniazid B. Inferior wall MI
B. Ethambutol C. Anteroseptal wall MI
C. Pyrazinamide D. Posterior wall MI
D. Streptomycin E. Anterolateral wall MI
E. Eritampucin
10. Silicosis next step in management
Anti TB drug that cause gout Ans Pulmonary TB
Ans Pyrazinamide
11. Non caseating granuloma + bilateral
3. Most common cause of anemia in CKD is lymphadenopathy
A. Erythropoietin deficiency Ans Sarcoidosis
B. Proteinuria
C. Hypertension 12. Not feature of hyperkalemia
D. Diabetes Ans ST depression??
E. Electrolyte imbalance
13. Unequal B.P + radiofemoral delay
Anemia in CKD Ans Coarctation of aorta
Ans Erythropoietin deficiency
14. 6 weeks after cardiac surgery
4. Warfarin and INR 9 Ans Dressler syndrome
Ans FFP
15. Scenario + pericarditis common cause
5. End stage renal disease Ans Viral
Ans GFR less than 15
16. Which one of the following is most important
6.25 years old female patient presented with fever 101F complication during the over correction of sodium?
for 8 days associated with headache and pain around A. Central pontine myelinolysis
eyeballs and muscle pain. She is living in a unhygienic B. Dehydration
environment. On examination there is Maculopapular C. Hypokalemia
rash all over body. CBC report shows thrombocytopenia D. Hyperkalemia
which of the following is most likely diagnosis E. Renal failure
A. Malaria
B. Typhoid 17. 40 years old male patient presented with high grade
C. Dengue fever 103F for 10 days and burning micturition for 8
D. Infectious mononucleosis days. On examination patient is tachypanic with RR of
E. Measles 36b/min and BP 60/40mmhg. Which of the following is
not included in the criteria for SIRS:
7. IV drug user + chronic diarrhea A. Temperature-
Ans Cryptococcus B. Blood pressure
C. Pulse rate
8. Hyponatremia + Pneumonia D. Leucocyte count
Ans Legionella E. Respiratory rate
18. Bed ridden + right bundle branch block + respiratory 28. Renal osteodystrophy low
alkalosis Ans Vit D3 level
Ans Pulmonary angiography
29.A young female presented with tachycardia
19. scenario + vegetarian shortness of breath, palpitations, tremors , sweating,
Ans Megaloblastic anemia she is diagnosed as generalized anxiety disorder, all of
the following car features of GAD except?
20. A 45-year-male presented with fatigue, A. Increased blood pressure
breathlessness and palpitation. On examination: apex B. Epigastric pain
beat is palpable in 5th ICS, taping in character. S1 is C. Urinary incontinence
loud with mid-diastolic murmur at mitral area. What is D. Headache
diagnosis? E. Nausea
A. Aortic stenosis
B. Mitral stenosis Not Feature G.A. Disorder
C. Pericarditis Ans Urine incontinence
D. Atrial myxoma
E. Mitral regurgitation 30. Potent Diuretic for Na & cl
Ans Loop Diuretics
21. Which among following bronchogenic carcinoma is
common in non-smoker patient 31. PEP for HIV
A. Adenocarcinoma Ans Antiretroviral therapy for 4 weeks
B. Large cell carcinoma
C. Broncho alveolar carcinoma 32. Which of the following is common symptom of
D. Squamous cell carcinoma schizophrenia?
E. Small cell carcinoma A Hypertension
B. Hallucination
22. Shipyard worker + white thick pleura C. Headache
Ans Mesothelioma D. Fear
E. Decreased mood
23. Not Feature of Anorexia Nervosa
Ans Obesity 33. Not cause of high output heart failure
Ans Aortic stenosis
24. SLE complication in Respiration
Ans Alveolar Hemorrhage 34. Post streptococcal infection in children
Ans DPGN
25.A 35-year-old presents with fever and
lymphadenopathy. Lymph node biopsy confirms 35. A 15 year old lady has had treatment with
Hodgkin's Lymphoma. Which one amongst the following penicillamine for rheumatoid arthritis. She presents with
types has the best prognosis? weakness and pallor claiming that it may be a side
A. Nodular sclerosis effect of the drug. Her Hb is 5 g/dl, WBC is 2 x 10^9
B. Lymphocyte predominant /L,platelet count is 15 x 10^9, INR is 1.0 and APTT is
C. Lymphocyte depleted 27s .What is the most likely diagnosis?
D. Mixed cellularity A. AML
E. Promyelocytic B. myelodysplasia
C. Folate deficiency
26. CML cytogenetic abnormality D. Aplastic anemia
Ans BCR-ABL E. CML

27. Malaria test


Ans As temperature rises
(MEDICINE 2) 7. 30 Years male presented to emergency department
with history of Predicate and uncontrolled blood
1. Which of the following features are not typically seen pressure for last 2 months he has been put on
in a patient with adult onset Still's disease? Amilodipine thiazide diuretics and ACEI for last one
A. Maculopapular rash month and well compliant clinically with normal
B. RA factor physique, heart rate of 84 beats/minute, BP 180-100
C. Pyrexia mmHg febrile. His CBC is fine while Sodium is normal
D. High ferritin level K 2.5 mg/dl normal blood sugar. ECG is consistent with
E. Lymphadenopathy left ventricular hypertrophy What is the most likely
underlying diagnosis she has
2. Myasthenia gravis contraindication drug A. Conn's Syndrome
Ans Aminoglycosides B. Cushing's Syndrome
C. Pheochromocytoma
3. HBV D. Congenital Adrenal Hyperplasia
Ans Life long immunity E. Diabetes insipidus

4. Refectory HTN HTN + low Potassium


Ans 400 mg - 160 mg Ans Conn disease

5. GBS initial test 8. Hepatorenal syndrome management


Ans LP Ans Albumin + terlipressin

6. A patient with small cell carcinoma of lung develops 9. Xanthochromia


lethargy. Serum electrolytes are drawn that show serum Ans SAH
sodium 118mmo/l. There is no evidence of ed
orthostatic hypotension, or dehydration Urine is 10. Oral pigmentation
concentrated with an osmolal 320 mmol/kg. Serum Ans oral antibiotics??
BUN, Creatinine, glucose are within normal range.
Which the following is the first step management? 11. Celiac disease
A. 5% dextrose infusion Ans Anti ttg + anti endomysial antibodies
B. Diuresis
C. Fluid restriction 12. Multiple sclerosis spasticity
D. Normal saline infusion Ans Baclofen
E. Tetracycline
13. D.M dehydration urine osmolarity >600 mg/dl
SIADH Ans HHS
Ans Fluid restriction
14. Colon cancer
Ans CEA

15. Test done before Hydroxychloroquine


Ans Visual Acuity
16. SLE rash no response to steroids 26. A patient was brought with history of pyrexia
Ans oral hydroxychloroquine contracted pupils Hypotension cyanosis progressing to
17. A lady presented with non-progressive dysphagia coma is suspected to be suffering from poisoning due to
only for solids Barium study showed proximal A. Cannabis
esophageal dilatation with distal constriction The most B. Dratura
likely diagnosis is C. Phenobarbitone
A. Achalasia cardia D. Diphenhydramine
B. Carcinoma esophagus E. Cocaine
C. esophageal spasm
D. Lower esophageal img 27. A farm worker presented to the ER with finding of
E. Peptic Stricture pin point pupils, sweeting, bronchoconstriction, diarrhea
and muscles fasciculation. What is the best treatment?
18. A 30 years old male patient presented with altered A. Administer sodium bicarbonate
level of sensorium since morning. He has had previous B. Naloxone
4 episodes of ALOC in last 3 months. His RBS is 60 C. Pralidoxime
mg/dl, BP 70/40 mmhg RR 18/min, Temp 97F. His D. Lomotil
electrolytes are Na 129, K 5.2, CI 98. What will be the E. All of these
most specific diagnostic test in this patient?
A. 24 hour urinary cortisol 28. Increase lipid
B. Short synacthin test Ans All of these
C. Low dose dexamethasone test
D. High dose dexamethasone test 29. Anti jo 1 antibodies
E. Salivary cortisol Ans dermatomyositis

Hypoaldosteronism 30. Transplantation


Ans Short Synacthen test Ans HIV

19.Cushing's disease is distinguished from Cushing's 31. Osteomalacia


syndrome by the presence of Ans Trendelenburg sign
A. Adrenal cortical hyperplasia
B. A buffalo hump 32. SBP
C. A pituitary neoplasm Ans IV antibiotics
D. Diabetic glucose tolerance curve
E. Hirsutism 33. HBV
Ans polyarteritis nodosa PAN
20. D.M
Ans 130/80 B.P goal 34. Most common cause of death in diabetic patient is?
A. Cancer
21. D.M first management B. renal failure
Ans Lifestyle modification C. Infections
D. cardiovascular disease
22. Child pugh score not included E. Diabetic ketoacidosis
Ans Creatinine
35. Dementia and confusion
23. Bloody diarrhea + low back pain Ans Alzheimer
Ans Ulcerative colitis

24. Diarrhea
Ans Microscopy + culture

25. Thyroid nodule


Ans FNAC
46 Preprof 5. A 31 year old lady has presented with increased
urinary frequency muscle weakness. Her past medical
1. A 22 year old women presented with passage of dark history includes severe depression, for which she takes
color urine for couple of days followed by swelling sertraline. On routine blood calcium is 11.5mg/dl
around eyes associated with malaise, anorexia & (normal 10.2mg/dl). Chest X Ray and urinary normal.
Generalized weakness. She has H/O infection at the What is the most likely cause of hypercalcaemia?
site of skin piercing about 4 weeks back, for which she A. latrogenic
was treated with antibiotics, she took antibiotic only for B. Lung cancer
02 days & then stopped because she felt abdominal C. Primary hyperparathyroidism
pain while taking them Her urine D/R shows protein 3+, D. Secondary hyperparathyroidism
blood 3+ nitrite negative and leukocytes 5-6/HPF. What E. Thyrotoxicosis
is the likely diagnosis.
A. IgA nephropathy 6. A patient with small cell carcinoma of lung develops
B. Alkaptonuria lethargy. Serum electrolytes are drawn that show serum
C. Acute intermittent porphyria sodium 118mmo/l. There is no evidence of ed
D. Membranous GN orthostatic hypotension, or dehydration Urine is
E. Post streptococcal G.N concentrated with an osmolal 320 mmol/kg. Serum
BUN, Creatinine, glucose are within normal range.
2. A 49 year old woman with history of cigarette Which the following is the first step management?
smoking presented with hemoptysis. She has history of A. 5% dextrose infusion
recurrent nose bleed and sinusitis for past 6 months. On B. Diuresis
chest examination she has crepitations with signs of C. Fluid restriction
consolidation on left side of chest. Lab reports CBC D. Normal saline infusion
normal, Na+ 140mmol/L k+5.7 mmol/L, serum E. Tetracycline
creatinine 2mg/dl, c-ANCA positive. What is the likely
diagnosis? 7. 30 Years male presented to emergency department
A Tuberculosis with history of Predicate and uncontrolled blood
B. Churg strauss vasculitis pressure for last 2 months he has been put on
C. Wegener's Granulomatosis Amilodipine thiazide diuretics and ACEI for last one
D. Sarcoidosis month and well compliant clinically with normal
E. Anti-GBM disease physique, heart rate of 84 beats/minute, BP 180-100
mmHg febrile. His CBC is fine while Sodium is normal
3. Regarding good pasture syndrome, which of the K 2.5 mg/dl normal blood sugar. ECG is consistent with
following antibodies help in the diagnosis. left ventricular hypertrophy What is the most likely
A. ANA underlying diagnosis she has
B. ASMA A. Conn's Syndrome
C. P-ANCA B. Cushing's Syndrome
D. Anti-GBM antibodies C. Pheochromocytoma
E. Anti-Ro antibodies D. Congenital Adrenal Hyperplasia
E. Diabetes insipidus
4. All of the following are correct for glomerulonephritis,
which of the following is not associated with low
complement level 8. Which of the following ECG finding is least likely to be
A. Cryoglobulinemia present in a patient with acute anterior ST segment
B. IgA nephropathy elevation MI
C. Post streptococcal GN A. ST elevation in lead V2 to V5
D. Lupus nephritis B. Shortened QT interval
E. Mesangiocapillary GN C. New right bundle branch block
D. ST Depression in leads and AVF
E. Hyperacute T waves in precordial leads
9. Beta blockers are? 15. A 24-year-old man is suspected of having
A. Positive inotropic and negative chronotropic mycoplasma pneumonia based on his symptoms of dry
B. Positive inotropic and positive chronotropic cough and chest X-Ray findings . He is also anemic with
C. Negative inotropic Negative chronotropic hemoglobin of 9.5gm/di, and hemolytic anemia from
D. Negative inotropic and Positive chronotropic cold agglutinins is suspected. Which of the following is
E. None of the above consistent with hemolytic anemia?
A. Increased haptoglobin level
10. Which of the following is not a major criteria for the B. Increased bilirubin in the urine
diagnosis of Rheumatic Fever? C. Increased reticulocyte count
A. Carditis D. Increased myoglobin in the blood
B. Subcutaneous nodules E. Increased hemopexin in the blood
C. Prolong P-R interval
D. Erythema marginatum 16. Following disease is a common cause of
E. Migratory Polyarthritis Eosinophilia:
A. Lymphoma
11. Which one of the following disorders is an acyanotic B. Malana.
heart disease? C. Asthma
A. Tetralogy of Fallot D. Parasitic infection
B. Transposition of great arteries E. Rheumatoid arthritis
C. Truncus Arteriosus
D. Tricuspid Atresia 17. Most definitive diagnostic test for typhoid fever:
E. Coarctation of Aorta A. Widal test
B. Typhidot
12 Which of the following regarding pulse pressure is C. Blood culture
true? D. Bone marrow culture
A. It Is the difference in systolic pressure between left E. CBC
and right arm
B. it is the difference in systolic and diastolic pressure 18. A 19-year-old man presents to his GP with 4 day
C. It is the difference in systolic pressure between upper history of painful facial swelling. fever and lethargy. On
and lower limbs examination, there is bilateral swelling of his parotid
D. Pulse pressure is wide in aortic stenosis glands. What is the most likely diagnosis?
E. Pulse pressure is narrow in regurgitation A. Measles
B. Mumps
13. Regarding primary PCI, the window period of door C. Influenza
to balloon time is? D. Infectious mononucleosis
A. 15 minutes E. Pertussis
B. 30 minutes
C. 60 minutes 19 Most common serious complication of chickenpox is:
D. 90 minutes A. Hepatitis
E. 120 minutes B. Osteomyelitis
C. Meningitis
14. Most common cause of cardiogenic shock is? D. Pneumonia
A. Non-STEMI E. Pericarditis
B. ST segment elevation MI
C. Severe Aortic stenosis 20. Which one among following is transmitted by
D. Cardiac tamponade faecooral route:
E. Hypertrophic cardiomyopathy A. Brucellosis
B. Hepatitis A
C. Hepatitis C
D.Tuberculosis
E. Scabies
21. 20 years old male presented to you in emergency 26. A 40-year-old Indian man presents to accident and
department with generalized spasms for 5 days his emergency with a one-month history of haemoptysis.
abdomen is rigid and hard he had a history of road He is a non smoker. On further questioning. he
traffic accident 20 days back what is the most likely mentions that he has also been having fevers and night
diagnosis.? sweats. Chest x-ray shows nodular shadowing in the
A. Meningitis right upper zone. What is the most likely diagnosis?
B. Tetanus A. Sarcoidosis
C. Malaria B. Small cell carcinoma of the lung
D. HIV C. Primary tuberculosis
E. Peritonitis D. Post-primary tuberculosis
E. Pneumocystis pneumonia
22. Which of following is not a HIV drug.?
A. Ribavirin 27. Hypercapnia is a typical feature of ?
B. Indinavir A. Pulmonary embolism
C. Lopinavir B. Salicylate intoxication.
D. Tenofovir C. Pulmonary fibrosis
E. Lamivudine D. Acute exacerbation of COPD
E. Obstructive sleep apnea
23. Macrocytosis is the feature of:?
A. Iron deficiency 28. Middle aged female having history of childhood TB,
B. Thalassaemia now presented with history of cough with copious
C. Alcoholism amount of foul sputum for six months and she has
D. Sideroblastic Anemia clubbing and on auscultation of chest course crackles
E. Chronic blood loss bilaterally most likely diagnoses. ?
A. Bronchiectasis
24. Haematological causes for stroke are following B. lung abscess
except.? C. Bronchial carcinoma
A. Sickle cell disease D. Sarcoidons
B. Polycythemia E. COPD
C. Hyperviscosity Syndromes
D. APLA Syndrome 29. Clubbing is positive in all except.?
E. Megaloblastic Anemia A. Fibrosing alveolitis
B. Cystic fibrosis
25. A. 37 year old woman presented to you with C. Emphysema
intermittent jaundice and left hypochondriac pain. She D. Bronchiectasis
tells you that this is her firth episode of jaundice and her E. Ca lung
past medical history was very significant for gall stones
but she refused for surgery at that time Her mother had 30. Which is not feature of interstitial disease.?
similar condition On examination she was pale and A. Orthopnea
spleen was palpable. CBC shows Hb-10 mg/dl MCV- B. Anemia
78, MCHC-40. What is your diagnosis? C. Clubbing
A. Hereditary spherocytosis D. Crepitations
B. G6PD deficiency E. Hypoxaemia
C. Sickle cell disease (Verify)
D. Megaloblastic anemia
E. Iron deficiency anemia 31. Which of the antituberculosis drug should be
avoided in pregnancy?
A. INH
B. Rifampin
C. Ethambutol
D. Pyrazinamide
E. Streptomycin
32. 88 Your old male, present with dyspnea, heavy 37. A 20-year-old woman is admitted for hypertensive
smoker, Breath sounds are absent an left side chest, crisis. In the hospital blood pressure is labile and
Percussion dull on left side chest, Trachea appears to responds poorly to anti-hypertensive therapy. The
be deviated towards right which of the following patient complains of palpitations and apprehension. Her
diagnosis most likely. past medical history shows that during her operation for
A. Pneumonia appendicitis, she developed hypotension during surgery
B. bronchial obstruction Hct:49%, TLC: 11×103 mm plasma glucose: 160 mg/dl,
C. Pleural effusion plasma calcium: 11 mg/dl. The most likely diagnosis is
D. Pneumothorax A. Primary hyperparathyroidism
E. Asthma B. Renal artery stenosis
C. Pheochromocytoma
33. A nurse is caring for a client who has been D. Essential hypertension
diagnosed with syndrome of inappropriate anti diurectic E. Insulin-dependent diabetes mellitus with CKD
hormone (SIADH). What type of electrolyte imbalance
would the nurse most likely seen in this situation? 38. Metabolic syndrome is characterized by all
A. Hyponatremia except
B. Hypernatremia A. Hyperinsulinemia
C. Hypocalcemia B. Type II Diabetes Mellitus
D. Hyperkalemia C. Hypertension
E. Hypermagnesemia D. Obesity
E. Pancreatitis
34. Delusions, Hallucinations, disorganized behavior are
characteristic symptoms of and 39. Hypoparathyroids is diagnosed by all of following
A. Depressive disorder except.
B. Obsessive compulsive disorder A. Low calcium
C. Panic disorder B. Low phosphate
D. Phobic anxiety disorder C. Low PTH
E. Schizophrenia D. Low magnesium
E. Low sodium
35. A 22 year old female patient presents with the
complaint of intrusive and repeated thoughts that her 40. A patient was admitted with abdominal pain,
hands are dirty and can be catch some infection in diarrhea, pigmented palmar creases and buccal
response to repeated she washes her hand again and mucosa. What is the most probable dx?
again She reports that she is aware that these thoughts A. Addison's disease
are her own and she wants to get rid of those matching B. Cushing syndrome
thought and repeated actions. The most likely diagnosis C. Pheochromocytoma
in this case D. Hyperthyroidism
A. Conversion disorder E. Hypoparathyroidism
B. Delusional disorder
C. Obsessive compulsive disorder 41. A 42-year-old woman presents with visual
D. Paranoid Schizophrenia disturbances. She reports having double vision which
E. Pyromania was intermittent initially but has now become much
more frequent. In addition, she becomes breathless
36. Cushing's disease is distinguished from Cushing's very easily and experiences palpitations. On
syndrome by the presence of examination, raised, painless lesions are observed on
A. Adrenal cortical hyperplasia the front of her shins and finger clubbing. The most
B. A buffalo hump likely diagnosis
C. A pituitary neoplasm A. De Quervain's thyroiditis
D. Diabetic glucose tolerance curve B. Thyroid storm
E. Hirsutism C. Pheochromocytoma
D. Graves disease
E. Pammer's disease
42. Most common cause of cushing's syndrome is? 47. Which of following is suggestive of Alcoholic
A. Pituitary adenoma (Cushing disease) Hepatitis
B. Adrenal adenoma A. Alcohol Consumption Men 80 gm day for
C. Ectopic ACTH B. Alcohol Consumption 30 gm women day for
D. Exogenic Steroids C. AST>ALT
E. Adrenal carcinoma D. Macrocytic anemia (Folic acid)
E. Megaloblastic anemia (B12)
43. A 50 year old female is 5 feet 6 inches and weight
170 pounds. There is a family history of diabetes 48. What is the most common cause of hepatocellular
mellitus fasting blood glucose is 130 mg/dl on two carcinoma?
occasions and HBA1C 6.5% She is asymptomatic, and A. Hepatitis A
physical shows no abnormalities Which will be first step B. Hepatitis B
of management C. Hepatitis C
A. Observe D. Hepatitis D
B. Life style modification E. Hepatitis E
C. Insulin
D. Sulfonylurea 49. A 26 Years old PG trainee got needle stick injury
E. Metformin from HCV. positive patient in ward, what is the best
advice for her
44. The major action of insulin is. A. She needs HCV Treatment prophylactically
A. Conversion of glucose to glycogen B. HCV immunoglobulin and vaccine
B. Conversion of fatty acids to glucose C. HCV RNA PCR 4 weekly for 12 weeks consequently
C. Gluconeogenesis D No need of treatment
D. Glycogenolysis E None of the above
E. Proteolysis
50. Following is not in the feature of Wilson Disease:
45. A 67-year-old diabetic female is brought into A. Increased Copper
accident and emergency following a collapse at her B. Increased Ceruloplasmin
home. She was found by her daughter who said she C. Caused by mutation in the ATP 7 B gene
saw the patient going to the toilet and then hearing her D. Coombs-ve haemolytic anemia.
collapse. The patient did consciousness and appears E. Autosomal recessive disorder
not lose well. Her supine blood pressure is 100/70 and
standing 115/79. Urine dipstick is positive for glucose, 51. Safest Sedative used in agitation (Psychosis)
nitrates, leukocytes and haematuria. The most likely caused by hepatic encephalopathy which is not
diagnosis is: metabolized in the liver.
A. Diabetic ketoacidosis A. Alprazolam
B. UTI B. Oxazepam
C. Orthostatic hypotension C. Lorazepam
D. Diabetic nephropathy D. Chlordiazepoxide
E. Hypoglycaemia E. Opicids

46. A 16-year-old girl presents to accident and 52. 30 years old male known case of chronic Hep-C
emergency with severe abdominal pain, nausea and treated in 2012 with peg: Interference was not
vomiting. On examination, the patient is tachypneic, responder. Now his Hb 12gm, WBC 5000, Plat: 130000,
capillary refill is 3 seconds and she has a dry tongue. Albumin 3.8gm/dl. MCV 80, HCV RNA PCR positive,
While listening to the patient's lungs, you detect a sweet HCV Genotype 3b. Now your best treatment option will
odour from her breath. The most likely diagnosis is: A. Sofosbuvir + velpatasvir
A. Diabetic ketoacidosis B. Sofosbuvir + Ribavirin
B. Non-ketotic hyperosmolar state C. Sofosbuvir + Simeprevir
C. Gastroenteritis D. Sofosbuvir + lamivudine
D. Pancreatitis E. Repeat pegylated interferon
E. Adrenal crisis
53. Tumour marker for hepatocellular carcinoma is. 57. 45 year old Asian man with chronic and cirrhosis is
A. α fetoprotein referred to your clinic for review. He is not currently on
B. Carcinoembryonic antigen HBV treatment. Abdominal ultrasound shows altered
C. CA 153 texture liver without any liver lesson spleen 13cm and
D. hCG no ascites seen His laboratory values are AST 45 UL.
E. CA 125 ALT 34 UL HBeAg negative, HBeAb positive HBV DNA
3000 IU/mL, Anti HDV negative INR 1.3. Total bilirubin 1
54. Which of the following is not a feature of Crohn's mg/dl. What is the most appropriate step at this time?
disease? A. A liver biopsy to assess for inflammation
A. Cigarette smoking reduces incidence B. An abdominal MRI
B. Fistula Formation C. Start antiviral treatment with tenofovir
C. Abscess formation D. Holding antiviral treatment for now and monitor liver
D. Non caseating granuloma formation E. Function tests over 6 months
E. Associated with transmural inflammation
58. Which of the following agents is recommended for
55. 30yr old female presented with complaint of difficulty primary prophylaxis of large esophageal varices and
in swallowing for 2 years which was occasional initially acts to decrease both portal flow and intrahepatic
but now occurs with every meal associated with chest resistance?
pain. She also complains of on/off vomiting for last few A. Prazosin
days which is non bilious and contains copious amount B. Isosorbide mononitrate
of saliva and food particles of meat she has eaten 2 C. Carvedilol
days before EGD was done which showed dilated D. Sotalol
esophagus with pooling of food particles E. Simvastatin
A. Esophageal Carcinoma
B. Esophagitis 59. A 30 year old female complaining generalized
C. Achalasia itching all over body for one week. She narrates that
D. Gastroparesis itching particularly worsens at night. Examination
E. Mallory Weiss tear reveals multiple erythematous lesions between fingers.
From which of following disease she suffers:
56. A 30 yr old female smoker presented in emergency A. Urticaria
with severe generalized abdominal pain and B. Eczema
constipation for 5 days, she had similar episodes in past C. Impeliga
3 times, at presentation her BP was 160/110 having D. Scabies
pulse of 120/min, denying for hypertensive history in E. Bullous pemphigus
past. Examination was unremarkable, she was admitted
and treated with analgesics. Following day she became 60. A young boy presented in dermatology OPD with
irritable, confused and developed visual hallucinations pruritic skin rash that is intense itchy and feels difficulty
and complaining of dark coloured urine. What is your in sleeping due rash. On examination you find papules
diagnosis? a vesicles on his extensor surface of knee and elbows
A. Acute Pancreatitis and on buttocks. He ha history of passing large bulky
B. Chronic pseudo intestinal Obstruction stools an weight loss for last one year. What underlying
C. Acute intermittent Porphyria skin disease?
D. Colon Carcinoma A. Psoriasis
E. Uremic encephalopathy B. Lichen planus
C. Dermatitis herpetiformis
D. Scabies
E. Pemphigoid
61. A 36 year Woman presents to the rheumatology 66. A 40 Year old make presents with c/o forgetfulness
outpatient clinic with a two month history of stiff hands continuous irregular movement of upper limbs & body
and wrists She mentions that the pain is particularly had He also complain of being depressed AND History of
for the first few hours after waking up and examination. such illness is positive in his family following is true.
her work the metacarpophalangeal interphalangeal A. Huntington Disease
joints wrists, joints and proximal are swollen likely B. Wilson disease
diagnosis?" C. Parkinson Disease
A. Rheumatoid arthritis D. Alzhiemmer disease
B. Osteoarthritis E. Haemochromatosis
C. Sepic arthobs
D. Polymyalgia rheumatica 67. The upper motor neuron impairment produces the
E. Reactive arthritis following change of muscles tone:
A. Flaccidity
62. A 20 year-old woman with a history of vertebral B. Spasticity
crush fractures presents to the osteoporosis outpatient C. Cogwheel rigidity
clinic. Which of the following investigation is most useful D. Myoclonia
to assess the extent of her osteoporosis? E. Tremours
A. Spinal x-rays
B. MRI scan 68. The presence of ptosis suggests damage to cranial
C. Full blood count, bone and liver biochemistry blood nerve:
tests A. IV
D. Vitamin D levels B. V
E. DEXA scan C. III
D. VII
63. Which of the following DMARD is safe in E. X
pregnancy?
A. Methotrexate 69. Antidote for Benzodiazepine toxicity?
B. Sulphasalazine A. Naloxone
C. Cyclophosphamide B. Flumazenil
D. Azathioprine C. Methadone
E. Mycophenolate D. Acetyl Cysteine
E. Dimercaprol
64. A young female known asthmatic presented with
allergic rash on different parts of body rest of the 70. To prevent the barotrauma (lung rupture) during
systemic examination is normal. In urine DR there is ascent phase of deep dive, the diver has to.
proteinuria. What is the most likely diagnosis? A. Ascend rapidly & close breath
A. Good pasture syndrome B. Ascend slowly & breath regularly
B. Churg-strauss syndrome C. Ascend slowly & hold breath
C. Wegener's granulomatosis D. Ascend rapidly & breath regularly
D. Eosinophilic pneumonitis E. Ascent rapidly & breath rapidly
E. SLE

65. A 70 year old male presented with severe lower


back pain. X Ray spine shows lytic lesions on labs
Alkaline phosphatase 305 IU/dl serum Calcium 11
mg/dl. What is the likely diagnosis?
A. Multiple Myeloma
B. Osteopenia
C. Osteomalacia
D. Paget's disease
E. Histiocytosis
46 Prof (MEDICINE 1) 9. Soldier with fever, fatigue, bilateral lymphadenopathy,
hepatosplenomegaly, and bilateral pedal edema
1. Middle aged female having history of childhood TB, Ans kala azar
now presented with history of cough with copious
amount of foul sputum for six months and she has 10. Drug causes gynecomastia
clubbing and on auscultation of chest course crackles Ans Spironolactone
bilaterally most likely diagnoses. ?
A. Bronchiectasis 11. Patient with K level raised, elevated creatinine and
B. lung abscess elevated urea. What initial step of management
C. Bronchial carcinoma Ans IV calcium gluconate
D. Sarcoidons
E. COPD 12. Hyperventilation leads to?
A Metabolic Acidosis
2. Which of the following conditions could result in High B. Metabolic Alkalosis
cardiac output failure? C. Respiratory Acidosis
A. Cor pulmonale D. Respiratory Acidosis with Metabolic Alkalosis
B. Dilated cardiomyopathy E. Respiratory Alkalosis
C. Hypertension
D. Thyrotoxicosis 13. A patient suddenly develops short of breath and
E. Valvular heart disease chest pain on right side. Trachea deviated to left side.
Hyperresonant percussion on right side
3. A 22-Year-old pregnant female has flank pain and Ans Right sided pneumothorax
fever. Her urine contains WBCS cast. What is the likely
diagnosis? 14. Atrial fibrillation occurs in
A. Acute pyelonephritis Ans mitral stenosis
B. Analgesic nephritis
C. Chronic Glomerulonephritis 15. Death due to MI commonly occurs due to
D. Ectopic pregnancy Ans Ventricular fibrillation
E. Pre-eclampsia
16. Acute glomerulonephritis occurs due to
4. Hypercalcemia as paraneoplastic syndrome occurs in Ans Previous streptococcal infection
Ans squamous cell carcinoma
17. Classic hemophilia
5. A patient with nephrotic syndrome and proteinuria. Ans spontaneous bleeding since childhood
Which is initial drug is given
Ans ACEI 18. A patient at anti TB drugs develops pain in right big
toe
6. A patient with hypertension and painless hematuria Ans pyrazinamide
Ans PCKD
19. Haematological causes for stroke are following
7. A child with diarrhea develops acute kidney failure except.?
Ans HUS A. Sickle cell disease
B. Polycythemia
8. Which of the following medications should be avoided C. Hyperviscosity Syndromes
in severe Aortic stenosis? D. APLA Syndrome
A. Antibiotics therapy in bacterial Endocarditis E. Megaloblastic Anemia
B. Aspirin
C. Beta blockers 20. CML feature
D. Digitalis Ans BCR-ABL on PCR
E. Diuretics
21. A pt smoker develops, peripheral edema, raised 35. Most definitive diagnostic test for typhoid fever:
JVP, bilateral crackles hepatomegaly A. Widal test
Ans cor pulmonale B. Typhidot
C. Blood culture
22. Features of CRF D. Bone marrow culture
Ans hypocalcemia E. CBC

23. Tumor lysis syndrome (MEDICINE 2)


Ans hyperphosphatemia
1. A patient was admitted with abdominal pain, diarrhea,
24. Drug not used in congestive cardiac failure pigmented palmar creases and buccal mucosa. What is
Ans Morphine the most probable dx?
A. Addison's disease
25. A patient can not perform alternate hand B. Cushing syndrome
movements C. Pheochromocytoma
Ans Dysdiadochokinesia D. Hyperthyroidism
E. Hypoparathyroidism
26. A diabetic patient undergoes IV contrast
pyelography. What steps to prevent renal damage 2. A 50 year old female is 5 feet 6 inches and weight
Ans Plenty of fluids 170 pounds. There is a family history of diabetes
mellitus fasting blood glucose is 130 mg/dl on two
27. Cold agglutinins positive in occasions and HBA1C 6.5% She is asymptomatic, and
Ans Mycoplasma physical shows no abnormalities Which will be first step
of management
28. Nephrotic syndrome includes A. Observe
Ans proteinuria, pedal edema, hyperlipidemia B. Life style modification
C. Insulin
29. a business man with complains of illness but no D. Sulfonylurea
actual disease on investigations and visits multiple E. Metformin
times to doctors
Ans Munchausen syndrome 3. Gastric Lavage is contraindicated in
A. Aspirin poisoning
30. Tetanus iv immunoglobulin and which antibiotic B. Corrosive alkali poisoning
Ans IV metronidazole C. Diazepam poisoning
D. Iron tablet poisoning
31. Obsessive Compulsive Disorder (OCD) E. Paracetamol poisoning
Characterized By:
A. Repetitive Unwanted Thoughts 4. 45 year old Asian man with chronic and cirrhosis is
B. Anxiety Is Not Associated With Ocd. referred to your clinic for review. He is not currently on
C. Person Feels Depressed HBV treatment. Abdominal ultrasound shows altered
D. Thoughts Are Rational texture liver without any liver lesson spleen 13cm and
E. Patients Get Easily Rid Of These Thoughts no ascites seen His laboratory values are AST 45 UL.
ALT 34 UL HBeAg negative, HBeAb positive HBV DNA
32. female with fatigue tiredness disturbed sleep etc 3000 IU/mL, Anti HDV negative INR 1.3. Total bilirubin 1
And generalized anxiety disorder mg/dl. What is the most appropriate step at this time?
A. A liver biopsy to assess for inflammation
33. Investigation of choice for hydatid cyst disease B. An abdominal MRI
Ans ultrasound abdomen/acchinochocal antibodies C. Start antiviral treatment with tenofovir
D. Holding antiviral treatment for now and monitor liver
34. Spontaneous pneumothorax most commonly occurs E. Function tests over 6 months
due to
Ans primary TB/ emphysema
5. A young boy presented in dermatology OPD with 12. Hemochromatosis is best diagnosed by
pruritic skin rash that is intense itchy and feels difficulty Ans serum ferritin and serum transferrin
in sleeping due rash. On examination you find papules
a vesicles on his extensor surface of knee and elbows 13. Recurrent duodenal ulcers and diarrhea
and on buttocks. He ha history of passing large bulky Ans serum gastrin level
stools an weight loss for last one year. What underlying
skin disease? 14. A female in 14 weeks gestation came with
A. Psoriasis thyrotoxicosis. Whats the best treatment option
B. Lichen planus Ans carbimazole
C. Dermatitis herpetiformis
D Scabies 15. Patient with sjogren syndrome. What is investigation
E Pemphigoid Ans Ro/La antibodies

6. A 40 Year old make presents with c/o forgetfulness 16. 42 years old woman with chronic arthritis
continuous irregular movement of upper limbs & body complaining of grittiness in eyes and dry mouth, +ve
He also complain of being depressed AND History of schirmer's test. RA factor and anti Ro/La antibodies are
such illness is positive in his family following is true. positive. What is diagnosis?
A. Huntington Disease A. SLE
B. Wilson disease B. Felty's syndrome
C. Parkinson Disease C. Still's disease
D. Alzhiemmer disease D. Sjogren's syndrome
E. Haemochromatosis E. Episcleritis

7. A farm worker presented to the ER with finding of pin 17. Which the following disease has positive RA
point pupils, sweeting, bronchoconstriction, diarrhea Ans rheumatoid arthritis
and muscles fasciculation. What is the best treatment?
A. Administer sodium bicarbonate 18. Travelers diarrhea most common cause
B. Naloxone Ans E.coli
C. Pralidoxime
D. Lomotil 19. Endoscopic method of choice to stop variceal
E. All of these bleeding
Ans banding
8. Ankylosing spondylitis
Ans HLA B27 20. Acromegaly is best diagnosed by
Ans insulin tolerance test with GH levels
9. A patient presented with pain abdomen ataxia
constipation Peripheral blood examination showed 21. A patient with hypertension and suspected
basophilic stippling RBC's. He is suffering from pheochromocytoma. What is the best antihypertensive
poisoning due to drugs
A. Iron Ans phenoxybenzamine.
B. Lead
C. Cadmium 22. Gout is best diagnosed by analysis of
D. Arsenic Ans joint aspirate
E. Cadmium
23. Joint pain fo 1 year with scaly patches over elbows
10. Pellagra occurs in and extensor surfaces
Ans NIACIN deficiency Ans psoriatic arthritis

11. Pruritus, steatorrhea jaundice viral markers 24. Most common cause of stroke
negative. Whats cause Ans Hypertension
Ans PBC
25. A patient with stroke has aphasia. Where is site of
lesion
Ans left cerebral hemisphere

26. Giardiasis is treated by


Ans metronidazole

27. A patient with DKA has which one of the following


abnormalities
Ans metabolic acidosis
28. Cicatricial alopecia occurs due to
Ans Folliculitis decalvans

29. Antibodies in drug induced lupus.


Ans Antihistone antibodies

30. A patient with jaundice occurring during starvation,


viral markers and other systemic examination negative
Ans Gilbert syndrome

31. Dysphagia plus hoarseness of speech


Ans Vagus nerve

32. CSF examination of a patient shows


albuminocytologic dissociation. What is management
Ans IVIG immunoglobulins

33. Most consistent finding in PCOS


Ans Ovarian cysts on ultrasound (sain ji key), LH:FSH
ratio ( chokran ji key

34. Finding in ulcerative colitis on colonoscopy biopsy


Ans Goblet cell depletion

35. A 31 year old lady has presented with increased


urinary frequency muscle weakness. Her past medical
history includes severe depression, for which she takes
sertraline. On routine blood calcium is 11.5mg/dl
(normal 10.2mg/dl). Chest X Ray and urinary normal.
What is the most likely cause of hypercalcaemia?
A. latrogenic
B. Lung cancer
C. Primary hyperparathyroidism
D. secondary hyperparathyroidism
E. Thyrotoxicosis
45 Preprof 7. Confirmatory feature for diagnosis of Malaria is:
A. Anemia
1. Which one of the following features is common in R. Hemoglobinuria
both delirium and schizophrenia? C. Raised Alanine transaminase
A. A family history of psychopathy D. Ring form seen on peripheral blood film
B. Hallucination E. Splenomegaly
C. Memory impairment
D. Social withdrawal 8. Slapped cheek rash (fifth disease) is caused by:
E. Waxing and waning of symptoms A. Epstein-Barr virus
B. Herpes simplex virus
2. A 49-year-old woman with a major depressive C. II influenza virus
episode refuses to take food and drinks. What is the D. Herpes zoster
most appropriate treatment? E. Parvovirus B19
A. Antipsychotic medication via nasogastric tube
B. Electro convulsive therapy (ECT) 9. Initial therapy for Idiopathic Thrombocytopenic
C. Fluoxetine via nasogastric tube Purpura (ITP) is:
D. Monoamine oxidase inhibitor via nasogastric tube A. Glucocorticoids
E. Sedation and intravenous fluids B. Fresh frozen plasma transfusion
C. Plasmapheresis
3. Which one among the following features help to D. Platelet transfusion
distinguish dementia from severe depression? E. Splenectomy
A. Grasp reflex
B. Poor attention spans 10. Which one among the following is transmitted by
C. Persistent headache faeco-oral route?
D. Social withdrawal A. Brucellosis
E. Weight loss B. Hepatitis A
C. Hepatitis C
4. Treatment of patient with depressive disorder who D. Tuberculosis
has expressed some ideas and thoughts about suicide E. Scabies
is?
A. Lithium 11. A 35-year-old presents with fever and
B. Monoamine oxidase inhibitors lymphadenopathy. Lymph node biopsy confirms
C. Olanzapine Hodgkin's Lymphoma. Which one amongst the following
D. Selective serotonin-reuptake inhibitors types has the best prognosis?
E. Tricyclic antidepressant A. Nodular sclerosis
B. Lymphocyte predominant
5. Most common serious complication of chickenpox is: C. Lymphocyte depleted
A. Hepatitis D. Mixed cellularity
B. Osteomyelitis E. Promyelocytic
C. Meningitis
D. Pneumonia 12. Multiple Osteolytic lesions, hypercalcemia and
E. Pericarditis neurologic abnormalities are often seen in:
A. Burkitt's lymphoma
6. Most diagnostic investigation in patient with typhoid B. CNS Tumors
fever in 1 week is: C. Chronic myeloid Leukemia
A. Blood Culture D. Hodgkin's lymphoma
B. Stool Culture E. Multiple Myeloma
C. Typhidot test
D. Urine Culture
E. Widal test
13. A 15-Year-old patient presents with a history of 19. Hospital acquired pneumonia is considered if
fever, bleeding from gums and pallor for 20 days. His pneumonia occurs in:
peripheral blood smear shows pancytopenia. Next most A. 24 hours after being placed on respirator
important investigation would be: B. 48 hours after hospital admission
A. Bone marrow examination C. 72 hours after ICU admission
B. Coombs test D. Large medical centers
C. Reticulocyte count E. 05 days after hospital admission
D. Serum Folie acid level
E. Serum Iron level 20. Gold standard investigation in diagnosis of
pulmonary thromboembolism is:
14. A 40-year-old lady presents with anemia, A. Helical spiral CT scan of chest
paraesthesia in fingers and toes. His CBC shows B. MRI chest
neutrophil hypersegmentation. He is suffering from: C. Pulmonary angiography
A. Anemia of chronic disease D. Trans esophageal echocardiography
B. Aplastic anemia E. Ventilation perfusion sean
C. Iron deficiency anemia
D. Sickle cell anemia 21. Which amongst the following bronchogenic
E. Vitamin B12 deficiency anemia carcinoma is common in non-smoker patients?
A. Adenocarcinoma
15. An 18-year-old male with sickle cell disease B. Bronchoalveolar carcinoma
presents with osteomyelitis. Which organism is C. Large cell carcinoma
responsible for this problem? D. Squamous cell carcinoma
A. E-coli E. Small cell carcinoma
B. Group A streptococci
C. Mycobacterium tuberculosis 22. A 70-year-old obese man is admitted with a 6 hours
D. Pseudomonas history of left sided chest pain. An ECG reveals inferior
E. Salmonella wall Myocardial Infarction. Which of the following
confirms diagnosis?
16. Which of the following parameters is increased in A. Creatinine kinase
obstructive lung disease? B. C-reactive protein
A. FEVI C. LDH
B. FEV1/FVC D. SGPT
C. FEV & FEV1/FVC E. Troponin T
D. Peak expiratory flow rate
E. Total lung capacity 23. Which of the following medications should be
avoided in severe Aortic stenosis?
17. A 35-year-old man suddenly develops shortness of A. Antibiotics therapy in bacterial Endocarditis
breath, tachypnea following leg surgery. His chest X-ray B. Aspirin
is normal. The most common underlying cause is: C. Beta blockers
A. Asthma D. Digitalis
B. ARDS E. Diuretics
C. CCF
D. Fat embolism 24. Most common presenting complaints in patient with
E. Pulmonary embolism ST Elevation myocardial infarction is:
A. Chest Pain
18. Presence of pus in pleural space is termed as: B. Dry Cough
A. Chylothorax C. Dyspnea
B. Empyema D. Generalized Weakness
C. Exudative effusion E. Vomiting
D. Hemorrhagic effusion
E. Para pneumonic effusion
25. Which of the following conditions could result in 31. A 50-year-old male develops acute renal failure with
High cardiac output failure? dark urine one week after a flu- like illness. A renal
A. Cor pulmonale biopsy is likely to reveal:
B. Dilated cardiomyopathy A. Acute tubular necrosis
C. Hypertension B. Amyloidosis
D. Thyrotoxicosis C. Crescentic glomerulonephritis
E. Valvular heart disease D. Myeloma kidney
E. Renal infarction
26. Which of the following drug regimens should be
started in acute management of deep vein thrombosis 32. A 55-year-old man presented with generalized
(DVT)? weakness. He recently started furosemide. What
A. Give Streptokinase electrolyte disturbance he has developed?
B. Start Heparin followed by Warfarin A. Hypernatremia
C. Start Aspirin and Warfarin together immediately B. Hypokalemia
D. Start Warfarin immediately C. Hyponatremia
E. Start Aspirin immediately D. Hypocalcemia
E. Hyperkalemia
27. Which one among the following is a feature of
infective endocarditis? 33. Urinalysis results include proteinuria, many red
A. Erythema marginatum blood cells and red blood cell casts, 1-2 white blood
B. Osler's node cells per high power field. Which disease best fits with
C. Prolong QRS duration on ECG these findings?
D. Raised ESR A. Acute cystitis
E. Sydenham's Chorea B. Acute post-streptococcal glomerulonephritis
C. Acute pyelonephritis
28. A 22-year-old patient presented with sensori-neural D. Chronic pyelonephritis
deafness and renal failure. Investigations reveals: E. Minimal change disease
positive Anti glomerular antibodies. What is diagnosis?
A. Alport syndrome 34. Which one of the following is the most important
B. Down syndrome complication during the rapid correction of sodium?
C. Giant syndrome A. Central pontine myelinolysis
D. Good pasture syndrome B. Dehydration
E. Wagner disease C. Hypokalemia
D. Hyperkalemia
29. A 22-Year-old pregnant female has flank pain and E. Renal failure
fever. Her urine contains WBCS cast. What is the likely
diagnosis? 35. A 40-year-old male presented with intractable
A. Acute pyelonephritis vomiting. What single characteristic finding would you
B. Analgesic nephritis expect in a blood test?
C. Chronic Glomerulonephritis A. Hyperkalemia
D. Ectopic pregnancy B. Hyperchloremia
E. Pre-eclampsia C. Increased serum bicarbonate
D. Metabolic acidosis
30. A 48-year-old male presents with hemoptysis, E. Raised anion gap
microscopic hematuria and mass at the upper pole of
the right kidney. What is the most likely diagnosis? 36. Which of the following is a pro-inflammatory
A. Focal segmental glomerulosclerosis cytokine?
B. Good pasture's syndrome A. C-reactive protein
C. IgA nephropathy B. IL-4
D. Post streptococcal glomerulonephritis C. IL-10
E. Renal cell carcinoma D. Serum amyloid precursor protein
E. Tumor Necrosis factor-alpha
37. A patient with a 15-year-old history of rheumatoid 42. Which one of the following drugs works by inhibiting
arthritis develops splenomegaly and neutropenia. What the tumor necrosis factor?
is the diagnosis? A. Cyclosporin
A. Dermatomyositis B. Infliximab
B. Felty syndrome C. Methotrexate
C. Osteoporosis D. Montelukast
D. Polymyalgia rheumatica E. Sulphasalazine
E. SLE
43. A 28-year-old woman presented to the accident &
38. A male presents with severe headache, lethargy emergency room with sudden onset ankle edema. In the
and weight loss. Which of the following would make the past year she described joint pains, increasingly cold,
diagnosis of giant cell arteritis likely? numb hands and an intermittent rash on her face. She
A. Abnormal ESR was pale and weak, urinalysis was positive for protein
B. Occipital headache only and blood. The ESR was 92 and she had
C. Non-tender carotid arteries lymphocytopenia. The most likely diagnosis is?
D. Papilledema on fundoscopy A. Hypersensitivity vasculitis
E. The patient is below 50 years B. Lupus Nephritis
C. Polyarteritis nodosa
39. A 67-year-old woman presents with shoulder D. Rheumatoid arthritis
weakness and headaches, in the past 24 hours she E. Wegener's granulomatosis
describes some blurred vision in her right eye, she is
admitted to the hospital. What immediate action is 44. Antinuclear antibodies homogeneous pattern is
necessary? present more likely in following disease.
A. Fluorescein angiography of the eye A. Autoimmune hepatitis
B. Temporal artery biopsy B. Polymyositis
C. Commence oral prednisolone > 60 mg o.d. C. Primary biliary cholangitis
D. Intravenous cyclophosphamide D. SLE
E. Measure cryoglobulins E. Wagner's granulomatosis

40. Which of the following drugs is most likely to cause 45 A 30-year-old obese male presented in OPD with
systemic lupus-like syndrome? complaints of retrosternal burning pain, regurgitation
A. Baclofen and sore throat for the last 01 Year. He underwent
B. Methotrexate Esophago. gastroscopy which showed multiple linear
C. Metronidazole ulcers in the lower third of esophagus with Incompetent
D. Procainamide lower esophageal sphincter. Stomach was normal. What
E. Sulfasalazine is Diagnosis?
A. Achalasia Cardia
41. A 36-year-old female patient with a 2-year history of B. Corrosive esophageal injury
rash on face, photosensitivity. joint pain, swelling of C. Esophageal Tumor
periorbital area especially in morning. What is the likely D. Gastro-esophageal Reflux Disease
diagnosis? E. Hiatus hernia.
A. Beheets Syndrome
B. Dermatomyositis 46. A 35-year-old male presents with painful oral ulcers
C. Felty syndrome and loose motions 3 to 4 times a day for 5 months. He
D. Polymyalgia rheumatica has lost 6 kg weight loss in the course of illness.
E. SLE Colonoscopy shows skip lesions involving descending
and transverse colon. What is the most likely diagnosis?
A. Amoebic dysentery
B. Celiac Disease
C. Crohn's disease
D. Tropical sprue
E. Ulcerative colitis
47. Which of the following disorders can lead to 53. Regarding celiac disease.
fulminant liver failure? A. Definitive diagnosis requires colonic Biopsy
A. Alpha-1 antitrypsin disease B. Is associated is HLA B27
B. Gaucher's disease C. Is associated with Dermatitis Herpetiformis
C. Haemochromatosis D. is type of inflammatory bowel disease
D. Hepatitis B virus E. Negative IgA anti-TTG excludes the disease
E. Wilson's disease
54. Regarding complication of diabetes
48. Following drug safe in patients with cirrhosis of A. Stroke is the commonest complication
liver? B. Autonomic neuropathy has very good prognosis
A. ACE inhibitors C. Charcot arthropathy results from vasculopathy
B. Cephalosporin D. Impaired glucose tolerance is risk factor for
C. Codeine cardiovascular disease
D. Narcotics E. Retinopathy is neuropathic complication
E. NSAIDs
55. Regarding treatment of diabetes:
49. Patient is cirrhosis and ascites present with fever A. Dietary and lifestyle modification is 2nd line treatment
and generalized abdominal pain. SBP is diagnosed in type II diabetes
when ascites fluid DR shows: B. GLP-1 receptor agonists are best choice for type-I
A. Neutrophil count < 500 diabetes
B. Total WBC count > 250 C. Insulin is only treatment of type I diabetes
C. Total WBC is < 500 and neutrophil is ≤ 250 D. Oral hypoglycemic are mainstay of treatment in type I
D. Total WBC is > 500 and neutrophil is ≥ 250 of total diabetes
WBC. E. Sulfonylurea is first line drug in type II diabetes
E. WBC count is < 500
56. Regarding diagnosis of diabetes:
50. Regarding Hemochromatosis A. C-Peptide is usually raised in type I diabetes
A. Is autosomal dominant B. Diabetes is diagnosed when RBS is 200 on single
B. Is characterized by copper overload occasion without symptoms
C. Phlebotomy is treatment of choice C. HbA1c6 is diagnostic of diabetes
D. Serum ferritin < 1000 establish diagnosis. D. IGTT is labeled when plasma glucose 2 hour after 75
E. Transferrin saturation of <45 is diagnostic gram oral glucose blood is 140-199
E. Impaired fasting glycemia is labeled when RBS is
51. Regarding inflammatory bowel disease. between 90-125
A. Crohn's disease involves colon only
B. Ulcerative colitis involves any part of GIT from mouth 57. Regarding hyperthyroidism increase T4, T3 and
to anus. decrease TSH represents secondary hyperthyroidism:
C. Oral ulcers are more common in crohn's disease A. Decreased TSH and increased T4 T3 is
then in ulcerative colitis characteristic of primary hyperthyroidism
D. Transmural inflammation is characteristic of B. Grave's disease is a rare cause of hyperthyroidism in
ulcerative colitis female
E. Fistulas and fissures are common in ulcerative colitis. C. Increase T4 T3 and normal TSH represents normal
pattern
52. Regarding viral hepatitis D. Increase T4, T3 and decrease TSH pattern represent
A. Hepatitis B is transmitted by faeco-oral route secondary hyperthyroidism
B. Hepatitis C cannot cause acute hepatitis E. Increase T4, T3 and TSH pattern represent primary
C. Hepatitis E is transmitted by faeco-oral route hyperthyroidism
D. Hepatitis B is not transmitted by vertical transmission
E. Hepatitis A leads to Cirrhosis
58. Regarding Pheochromocytoma 64. Patient with Bell's palsy has pathology involving
A. Pheochromocytoma is part of MEN-2B which cranial nerve?
B. Pheochromocytoma is tumor lung parenchyma cells A. III
C. Pheochromocytoma is tumor of epithelial cell B. V
D. Serum VMA is diagnostic C. VII
E. Surgery is treatment of choice D. XI
E. XII
59. Regarding Hypothyroidism:
A. Decrease TSH, decrease T4, T3 represents primary 65. Lower motor neuron type of paraparesis is
hypothyroidism characterized by:
B. Hypothyroidism presents with weight loss A. Brisk reflexes
C. Increase TSH and decrease T4, 13 are hallmark of B. Hypotonia
primary hypothyroid C. Positive kerning sign
D. Serum prolactin is normal in hypothyroidism D. Spasticity
E. TSH is not sensitive marker of hypothyroidism E. Up-going plantars

60. Regarding amoebic liver abscess: 66. Pathognomonic sign of meningitis is:
A. Caused by entamoeba coli A. Presence of fever and cranial nerve deficit
B. Caused by entamoeba dispar B. Presence of hemiparesis, headache and fever
C. Caused by entamoeba moshkovskii C. Presence of fever, headache and neck rigidity
D. Caused by cystic form of entamoeba histolytica D. Presence of fever & monoparesis
E. Serum antibodies are found in more than 90% of E. Presence of neck rigidity and hemiparesis
cases
67. Subarachnoid hemorrhage is best diagnosed by:
61. A 43-year-old woman has diabetes mellitus, A. History
hypertension, hirsutism and central obesity. Her 24 hour B. Clinical Examination
urinary cortisol is highly raised and serum ACTH is very C. CT scan brain
low. MRI shows a normal pituitary gland. Likely D. X-ray skull
diagnosis is E. Fundoscopy
A. Acromegaly
B. Addison's disease 68. Gastric Lavage is contraindicated in
C. Cushing syndrome A. Aspirin poisoning
D. Pheochromocytoma B. Corrosive alkali poisoning
E. Prolactinoma C. Diazepam poisoning
D. Iron tablet poisoning
62. Diabetes mellitus may occur as a secondary E. Paracetamol poisoning
consequence of:
A. Addison's Disease 69. Which one of the following is least characteristic of
B. Albinism Addison's disease?
C. Chronic Pancreatitis A. Hypoglycemia
D. Hyperthyroidism B. Hyponatremia
E. Wilson's Diseases C. Hyperkalemia
D. Metabolic alkalosis
63. Specific test for diagnosis of multiple sclerosis is: E. Positive short ACTH test
A. CT scan brain
B. Delayed visual evoked response
C. Lumbar puncture
D. MRI brain
E. X-Ray skull
70. A 53-year-old man presents as his wife has noticed 45 Prof (MEDICINE 1)
a change in his appearance. He has also noticed his M1 paper same as preprof
hands seem large. On examination BP 170/95 mm Hg &
have bi- temporal hemi-anopia. What is the most (MEDICINE 2)
appropriate first line treatment?
A. Bromocriptine 1- Regarding A topic dermatitis all of the following are
B. External irradiation correct except
C. Octreotide Ans. Only one parents transfer a 80% Atopic dermatitis
D. Pegvisomant
E. Trans-sphenoidal surgery 2- Regarding hepatitis C which one is correct statement
Ans. Gives direct Antiviral agents

3- scenarios + another side effect of isotretinoin in Acne


vulgaris is?
Ans. Hypertriglyceridemia

Remaining prof Question From Preproff Batch 45 &


Annual Keys Of Batch 43
44 Preprof (MEDICINE 1) 6. Following is the common cause of nephrotic
syndrome in children:
1. Which One Of The Following Durg is Tricyclic A. Crescentic Glomerulonephritis
Antidepressant? B. Hypoparathyroidism
A. Sertraline C. Chronic renal failure
B. Fluoxetine D. Membranous glomerulonephritis
C. Venlafaxine E. Minimal lesion glomerulonephritis
D. Lithium
E. Amitriptyline 7. Hodgkin's disease is best diagnosed through:
A. Blood CP.
2. A decrease in plasma osmolality would increase. B. Whole body CT scan.
A. Urine production. C. Bone marrow Histopathology.
B. Renin production. D. Lymphangiography.
C. Thirst. E. Lymph node Histopathology.
D. Plasma sodium levels.
E. ADH production. 8. A 55-year-old man has just arrived in accident and
emergency complaining of 20 minutes of central
3. A 16-year-old boy presents with a low-grade fever crushing chest pain. Which feature is most indicative of
which started 1 week ago. The patient also reports myocardial infarction at this moment in time?
feeling fatigued and indicates pain in his joints. His A. ST elevation
parents mention that he has been visiting the toilet more B. Q waves
often than usual. A urine dipstick shows trace proteins, C. ST depression
while a blood test shows raised eosinophils. The most D. Raised troponin
likely diagnosis is: E. Inverted T waves
A. Diabetes mellitus
B. UTI 9. A 28-year-old female presented with fever for 07
C. Reactive arthritis days. Fever was sudden in onset, high grade,
D. Renal failure continuous associated with sweating. On examination:
E. Acute tubulointerstitial nephritis spleen is palpable 3 cm below the left costal margin. On
investigation blood culture for salmonella is positive.
4. A 45-year-male presented with fatigue, What is diagnosis?
breathlessness and palpitation. On examination: apex A. Malaria
beat is palpable in 5th ICS, taping in character. S1 is B. Pneumonia
loud with mid-diastolic murmur at mitral area. What is C. Amoebiasis
diagnosis? D. Typhoid
A. Aortic stenosis E. Liver abscess
B. Mitral stenosis
C. Pericarditis 10. Name ATT drug which cause peripheral neuropathy:
D. Atrial myxoma A. Streptomycin
E. Mitral regurgitation B. INH
C. PZA
5. Most affected area in intestinal TB is: D. Rifampicin
A. Ileocecal region E. Ethambutol
B. Sigmoid colon
C. Jejuno-ileal junction 11. Immune Thrombocytopenic Purpura is best
D. Duodeno-jejunal junction managed initially with:
E. Duodenum A. Azathioprine.
B. Platelet transfusions.
C. Whole blood transfusion
D. Prednisolone.
E. Folic acid supplements.
12. Appropriate antibiotic therapy for chlamydia 17. Macrocytic anemia with peripheral neuropathy is
pneumonia is: commonly seen in:
A. Amikacin A. Aplastic anemia.
B. Ampicillin B. Iron deficiency anemia.
C. Cefuroxime C. Vitamin B12 deficiency anemia.
D. Cefotaxime sodium D. Hemolytic anemia.
E. Erythromycin E. Folic acid deficiency anemia.

13. A 20-year-old male presented with fever, chest pain, 18. Steroids are usually required for management of
arthritis and skin rash. Patient also has history of sore following condition:
throat two months back. On examination early diastolic A. Aplastic anemia.
murmur on aortic area. ECG shows prolong PR interval. B. Folic acid deficiency anemia.
On investigation ASO titer raised. What is diagnosis? C. Autoimmune hemolytic anemia.
A. RA D. Vitamin B12 deficiency anemia
B. Infective endocarditis E. Hereditary spherocytosis.
C. Acute rheumatic fever
D. Aortic regurgitation 19. Palpitation can occur in:
E. SLE A. All of above
B. Complete Heart block
14. A 20-year-old girl presents with history of episodic C. Bradycardia
cough and shortness of breath. On further enquiry she D. Tachycardia
told that she has history of allergy. On examination E. Normal Heart Rate
pulse: 110 beats/ min, BP: 110/70 mmHg, Temp: 98.6°F,
Respiratory Rate: 22 br/min, and bilateral wheezes. 20. Following is the feature of chronic renal failure?
What is your diagnosis? A. Hyper-natremia
A. COPD B. Hyper-calemia
B. Bronchial Asthma C. Hypo-uricemia
C. Pulmonary TB D. Hypo-calcemia
D. Bronchiectasis E. Hypo-kalemia
E. Lung Abscess
21. Poikilocytosis is:
15. Which One Of The Following Durg Is Tricyclic A. Cells with equal size.
Antidepressant? B. Cells with unequal size.
A. Lithium C. Cells with equal diameter
B. Sertraline D. Cells with different shape
C. Amitriptyline E. Cells with same shape.
D. Venlafaxine
E. Fluoxetine 22. A 39-year-old Indian man presents to his GP with a
5-week history of haemoptysis, night sweats and weight
16. Obsessive Compulsive Disorder (OCD) loss. Which of the following investigations can be used
Characterized By: to confirm the diagnosis of tuberculosis?
A. Repetitive Unwanted Thoughts A. Blood cultures
B. Anxiety Is Not Associated With Ocd. B. Chest x-ray
C. Person Feels Depressed C. Computed tomography pulmonary angiogram (CIPA)
D. Thoughts Are Rational D. Ziehl-Nielsen sputum staining
E. Patients Get Easily Rid Of These Thoughts E. Tuberculin skin testing
23. Following is the most common nephrotoxic drug? 29. Which of the following organisms would typically be
A. Gentamycin found in patient with atypical community- acquired
B. Ascorbic acid pneumonia?
C. Ranitidine A. Legionella pneumophila
D. Amlodipine B. Haemophilus influenza
E. Penicillin C. Streptococcus pneumonia
D. Staphylococcus aureus
24. Most common cause of meningitis with rash on body E. Pseudomonas spp
of an adult is:
A. Streptococcus pneumonia 30. A 40-year-old smoker male with 12 pack year
B. Staphylococcus history of smoking presents with exertional shortness of
C. Meningococcus breath and cough. On examination Pulse 90 b/min, BP
D. Haemophilus species 130/80, respiratory rate 16 b/min, Temp 98 OF, normal
E. Tuberculosis vesicular breathing with poor air entry and rhonchi.
Chest X-ray shows hyperinflated lungs. What is
25. Enter Roll No. of Final Year MBBS Pre-Prof. Online diagnosis?
Test Session 2015-16 (MEDICINE-1): A. Aspergilloma
B. Asthma
26. A 18-year-old male presents with sudden right sided C. Lung Abscess
chest pain which worsens with cough and deep D. CA bronchus
inspiration. On examination: right chest is bulged with E. COPD
stony dull percussion note and absent breath sounds.
Chest X-Ray shows homogenous opacity with loss of 31. Drug of choice for cerebral malaria is:
costophrenic and cardiophrenic angles on right side. A. Sulfadoxine
What is radiological diagnosis? B. Chloroquine
A. COPD C. Tetracycline
B. Pneumothorax D. Halofantrine
C. Lung infarction E. Quinine
D. Bronchiectasis
E. Pleural Effusion 32. You are told by your registrar that a 69-year-old man
has been admitted to the chest ward with dyspnea,
27. A 21-year-old man presents with painless cyanosis and finger clubbing. His chest x-ray shows
haematuria which he has noticed in the last 3 days. He bilateral lower zone reticulo- nodular shadowing. From
suffers from type I diabetes which is well controlled, but the list below, which is the most likely diagnosis?
is otherwise fit and healthy. The patient has recently A. Bronchitis
recovered from a mild throat infection. Urine dipstick B. COPD
analysis reveals blood and protein in the urine. The C. Bronchogenic carcinoma
most likely diagnosis is: D. Bronchiectasis
A. Benign prostate hypertrophy E. Pulmonary fibrosis
B. Diabetic nephropathy
C. Urinary tract infection (UTI) 33. During chest pain most important and early required
D. IgA nephropathy investigation is:
E. Henoch-Schonlein purpura A. Cardiac enzymes
B. Thallium scan
28. For diagnosis of tuberculosis meningitis: C. Echocardiography
A. It is diagnosed clinically D. Exercise tolerance test (ETT)
B. Presence of kerning sign is diagnostic E. Electrocardiogram
C. CSF analysis is mandatory
D. Contrast CT brain is diagnostic tool
E. Chest x-ray is required
34. A 28-year-old male met with an accident and 40. A 55-year-old male presented with chest pain which
sustained severe crush injury. He is most likely to is brought on by exertion and relieved by rest. What is
develop: likely diagnosis?
A. Hyponatremia A. Pneumothorax
B. Acute myocardial infarction B. Esophageal spasm
C. Fulminant hepatic failure C. Bacterial pneumonia
D. Hypercalcemia D. Angina pectoris
E. Acute renal failure E. Pulmonary Embolism

35. Name of Institution: 41. Investigation of choice in COPD:


A. Chandka Medical College Larkana A. X-Ray Chest
B. Ghulam Muhammad Mahar Medical College Sukkur B. ABGS
C. Spirometry
36. Long term monitoring of iron replacement in Iron D. Sputum Culture
deficiency anemia requires: E. CBC
A. Reticulocyte count.
B. S. Iron. 42. Which of the following is negative symptom of
C. Blood CP Schizophrenia:
D. S.ferritin. A. Good Self Care
E. Reticulocyte Production Index. B. Talkativeness
C. Blunting Affect/Mood
37. A 50-year-old male smoker with history of smoking D. Delusion
20 pack years, presents with hemoptysis, chest pain E. Hallucination
and weight loss for one month. His chest X-Ray shows
mass in right side of chest with irregular margins. What 43. Which of following drug cause hyperkalemia?
is diagnosis? A. ACE inhibitors
A. Hydatid cyst B. Cephalosporin
B. CA lung C. Loop Diuretic
C. Consolidation D. Laxatives
D. Collapsed lung E. B2 agonist
E. Pulmonary infarction
44. A 48-year-old woman has been diagnosed with
38. Chveostek's Sign is manifestation of: essential hypertension and was commenced on
A. Hypokalemia treatment three months ago. She presents to you with a
B. Hypocalcemia dry cough which has not been getting better despite
C. Hyperphosphatemia taking cough linctus and antibiotics. You assess the
D. Hypermagnesemia patients medication history. Which of the following
E. Hyperkalemia antihypertensive medications is responsible for the
patient's symptoms?
39. Following is a feature of Pre-renal azotemia: A. Lisinopril
A. Urinary sodium concentration > 40 meq/l B. Atenolol
B. Reversed with replacement of fluids C. Amlodipine
C. Fractional excretion of Na > 1% D. Bendroflumethiazide
D. Commonly affects glomerulus E. Furosemide
E. Urinary osmolality < 500 mosm/kg
45. A 55 year male farmer has presented with fever for 49. A 37 year female farmer patient has presented with
03 month & dragging abdominal pain for 20 days in easy fatigability for 2 months and breathlessness for 7
OPD. O/E: He is febrile, anemic and having significant days. She is moderately anemic. Her CBC report
splenomegaly of 17 Cms. Lymph nodes are not reveals: Hct 20, while reticulocyte count is 4%.
palpable and chest is clear. Most likely he is having: Reticulocyte production index will be:
A. Acute myeloid leukemia. A. 2
B. Chronic myeloid leukemia. B. 5
C. Acute lymphoblastic leukemia. C. 1
D. Pulmonary tuberculosis D. 4
E. 3
46. A 29-year-old woman goes to see her GP
complaining of fatigue and palpitations. She says she 50. Which of the following is Non-modifiable risk factor
has also lost weight, though without dieting. On for angina pectoris?
examination, her pulse rate is approximately 120/min A. Smoking
and irregularly irregular. Her BP 142/89 mmHg and her B. Obesity
body mass index 19. There are no added cardiac C. Old Age
sounds. The ECG confirms the diagnosis of atrial D. Hypertension
fibrillation. What would you suggest as the most useful E. Diabetes Mellitus
next investigation?
A. Chest X-Ray 51. Alcohol Facilitate Which Of Following
B. Fasting blood sugar Neurotransmitter.
C. ECG A. Gaba (Gama Aminobutyric Acid)
D. Thyroid function tests (TSH.T3, T4 free) B. Acetylcholine
E. Full blood count C. Dopamine
D. Glycine
47. Which of the following is symptom of Depressive E. Serotonin
Disorder.
A. Grandiosity 52. Which of the following infection present with fever
B. Early Morning Waking and vesiculo-pustular rash?
C. Elation A. Typhoid Fever
D. Blunting B. Malaria
E. Breathlessness C. Giardiasis
(Verify) D. Chicken Pox
E. Amoebiasis
48. A 30-year-old male presented with high grade fever
for 03 weeks, associated with chills and sweating. On (MEDICINE 2)
examination: patient is anemic and spleen is palpable 2
cm below left costal margin. Pan-systolic murmur is 1. Which is the function of carbidopa?
audible at mitral area. Echocardiography shows A. It reduces dopamine excretion
vegetation. What is diagnosis? B. It prevents dopamine metabolism
A. Mitral stenosis C. It inhibits aromatic amino acid decarboxylase
B. Pericarditis D. It results increased dopamine synthesis
C. Infective endocarditis E. It causes dopamine release
D. Typhoid fever
E. Malaria
2. A 60-year-old man with alcoholic liver disease was 7. A 36 Years Female Patient With A 2-Year History Of
admitted with upper Gl bleed secondary to esophageal Rash On Face Photosensitivity Joint Pain Swelling
varices. The patient undergoes endoscopic variceal Periorbital Area Especially In Morning. What Is The
banding and discharged after 2 weeks in-hospital stay. Diagnosis?
Which of the following medications would act as A. Behcets Syndrome
prophylaxis in preventing a rebleed from his esophageal B. Polymyalgia rheumatica
varices? C. Felty syndrome
A. Propranolol D. SLE
B. Amlodipine E. Dermatomyositis
C. Ramipril
D. Furosemide 8. What specific sign you may notice in the urine
E. Irbesartan analyses in the patient with diabetes insipidus?
A. High proteinuria
3. A 50 year old woman presents with dry eyes, a dry B. Microscopic hematuria
mouth, an erythematous rash and polyarthralgia. C. Low level of specific gravity
Investigations: ANA strongly positive anti-Ro/SSA D. Leukocyturia
antibodies positive, rheumatoid factor positive.. IgM and E. Macroscopic hematuria
IgA levels are normal and the kappa/lambda ratio is
normal. What is the most likely diagnosis? 9. A 24-year-old nurse is exposed to meningococcal
A. Hyperviscosity syndrome meningitis in the emergency department. What is the
B. Myeloma associated vasculitis recommended treatment?
C. Systemic Lupus Erythematosus A. Require urgent surgical intervention
D. Rheumatoid arthritis with secondary Sjogren's B. Rifampin
Syndrome C. Isoniazid (INH)
E. Primary Sjogren's Syndrome D. Streptomycin
E. Ethambutol
4. A 67-year-old man presents feeling unwell and
complaining of general malaise. He mentions a long 10. Which of the following is not found in Addison's
history of alcohol abuse and his past medical history Disease?
shows deranged liver function tests. Which of the A. Hypoglycemia
following clinical signs does not form part of chronic liver B. Hypotension
disease? C. Hypernatremia
A. Koilonychia D. Hypovolemia
B. Spider naevia E. Hyperpigmentation
C. Finger clubbing
D. Palmer erythema 11. ??
E. Jaundice
12. Which of the following drugs is most likely to cause
5. Cushing's disease is distinguished from Cushing's systemic lupus-like syndrome?
syndrome by the presence of A. Procainamide
A. Adrenal cortical hyperplasia B. Methotrexate
B. A buffalo hump C. Metronidazole
C. A pituitary neoplasm D. Sulfasalazine
D. Diabetic glucose tolerance curve E. Baclofen
E. Hirsutism
13. Which of the following is the most common cause
6. ??? of colonic obstruction?
A. Crohn's stricture
B. Diverticulitis
C. Intussusception
D. Ventral hernia
E. Adenocarcinoma
14. Which one of the following is not symptom 21. A Hypertensive patient with severe headache and
hyperthyroidism. vomiting. He has got neck stiffness but no focal
A. Exophthalmia neurological deficit. What is the most probable
B. Weight gain diagnosis?
C. Increased heat production A. Subarachnoid Hemorrhage
D. Increased metabolism B. Hypertensive Encephalopathy
E. Tachycardia C. Meningitis
D. Migraine
15. Enter Roll No. of Final Year MBBS Pre-Prof. E. Brain Tumor
Online Test Session 2015-16 (MEDICINE - II):
25. Which of the following anti-epileptic medication is
16. What is the most common cause of bacterial broad spectrum?
meningitis? A. Carbazmazepine
A. Neisseria meningitidis B. Ethosuximide
B. Staphylococcus aureus C. Oxcarbazepine
C. Salmonella typhi D. Phenobarbitone
D. Streptococcus pneumoniae E. Valproic acid
E. Heamopbilus influenzae
28. When counseling patients with celiac disease. which
17. What is investigation of choice for celiac disease: of the following would you advise them to avoid?
A. serum TTG IgA level A. Barley
B. Stool test B. Rice
C. Endoscopy with Duodenal biopsy C. Corn
D. Both A & B D. Millet
E. Serum TSH E. Wheat
(Verify)
29. From, the list below, which of the following
18. Contact dermatitis is described as what type of carcinomas of the lung is highly associated with
reaction? exposure to asbestos?
A. Type IV hypersensitivity A. Malignant mesothelioma
B. Non-allergic B. Small cell carcinoma
C. Type III hypersensitivity C. Squamous cell carcinoma
D. Type Il hypersensitivity D. Adenocarcinoma
E. Type I hypersensitivity E. Large cell carcinoma

19. Psoriasis: 30. What is the investigation of choice for peptic ulcer
A. Mortality rate is more than 60% disease:
B. Erythroderma is less severe variant A. Ultra sound abdomen
C. Infectious disease B. Barium study
D. Nails are not involved C. PH monterining
E. Characterized by erythematous plaques on extensor D. Upper G.I endoscopy
surfaces E. X-ray chest

20. Which of the following is the most common risk 32. Which of the following anterior pituitary hormones
factor for irritable bowel syndrome (IBS)? plays a major role in the regulation of a nonendocrine
A. Antibiotic use target gland?
B. Depression A. Thyroid-stimulating hormone
C. Celiac disease B. Follicle-stimulating hormone
D. Low birth weight C. Luteinizing hormone
E. Bacterial gastroenteritis D. Prolactin
E. Adrenocorticotropic hormone
33. Antinuclear antibodies homogeneous pattern is 39. Primary hypothyroidism develops as a result of:
present more likely in following disease: A. Pathologies of pituitary gland
A. WAGNERS GRANULOMATOS B. Peripheral resistance to effect of thyroid hormones
B. SLE C. Loss of thyroid hormones during nephritis
C. PRIMARY BILIARY CHOLANGITIS D. Thyroid pathology
D. POLYMYOSITIS E. Hypothalamus Disorders
E. Autoimmune hepatitis
40. A 67-year-old woman presents to you with extensive
34. A 22-year-old unconscious man is brought into scalp hair loss which has been getting progressively
accident and emergency by street by passers The worse over the last year. You also notice thinning of the
patient's Glasgow Coma Scale is 12/15, he has a eyebrows. The patient's past medical history includes
respiratory rate of 10 and blood pressure of hypertension, left-sided pulmonary embolism one year
100/60mmHg During your examination you notice ago and hypercholesterolaemia. You assess the
pinpoint pupil what is treatment of choice? patient's medication list. Which one of the following
A. Naloxone drugs could be responsible for causing generalized
B. Flumazenil alopecia?
C. Naltrexone A. Aspirin
D. Lorazepam B. Bendroflumethiazide
E. Methadone C. Ramipril
D. Simvastatin
35. Developing of gigantism is conditioned by: E. Warfarin
A. Overwhelming secretion of somatostatin in adult
B. Overwhelming secretion of GH in adolescence 41. A 69-year-old man present with a 2-week history of
C. Overwhelming secretion of GH in adults abdominal pain which has worsened over the last few
D. Inborn sensitivity lack in tissues to GH days. On examination, the patient is jaundiced and the
E. Overwhelming secretion of GH in old age abdomen is distended with tenderness in the epigastric
region. In addition, there is a tender hepatomegaly and
36. Which one of the following is not biological activity of shifting dullness. Which of the following is a cause of
vitamin D: hepatomegaly?
A. Stimulation of synthesis of bone matrix A. Iron deficiency anemia
B. Increase of Ca2+ excretion in kidneys B. Crohn's disease
C. Stimulation of calcification matrix in bone C. Ulcerative colitis
D. Intensification of Ca2+ absorption in guts D. Left-sided heart failure
E. Stimulation of metabolism in muscles E. Budd-Chiari syndrome

37. Which of the following descriptions is more 42. Approved treatment for Acute Hepatitis B is?
characteristic of rigidity than spasticity? A. All of above
A. Velocity-dependent increase in muscle tone B. None of above
B. Clasp-knife phenomenon C. Entecavir
C. Increased resistance throughout the range of motion D. Tenofovir Alafenamide
D. Predominantly flexor muscle involvement E. Tenofovir Disoproxil Fumarate
E. Unconsciousness
43. Paroxysmal hypertension is most typically
38. The investigation of choice for Achalasia is: associated with:
A. Upper Gl endoscopy A. Pituitary adenoma
B. PH Monitoring B. Pheochromocytoma
C. X-ray chest C. Adrenal adenoma
D. Esophageal Manometry D. Thyroid adenoma
E. Barium esophagography E. Parathyroid adenoma
44. A 50-year-old overweight woman returns to your 49. You examined a 19-year-old man in your clinic who
office for GERD. She has had a normal upper presents with a history of transient jaundice. On direct
endoscopy, and has taken twice-daily PPI therapy. What questioning, you ascertain that the jaundice is
is the next best step in her further management? noticeable after periods of increased physical activity
A. Continue PPI and subsides after a few days. The patient has no other
B. Prescribe Histamine 2 Receptor antagonist. symptoms and physical examination is unremarkable.
C. Order upper GI endoscopy Full blood count is normal (with a normal reticulocyte
D. Referral to surgery for anti-renuх surgery count) and liver function tests reveal a bilirubin of 37
E. Biofeedback umol/L. The most appropriate is diagnosis:
A. Wilson's disease
45. Following parasitic infection is the cause of chronic B. Gilbert syndrome
liver disease? C. Primary biliary cholangitis
A. Schistosomiasis D. Obstructive jaundice
B. Leishmaniasis E. Hemochromatosis
C. Malaria
D. Dengue 50. Name of Institution:
E. Giardiasis A. Chandka Medical College Larkana
B. Ghulam Muhammad Mahar Medical College Sukkur
46.A 28-year-old woman presented to the accident &
emergency room with sudden onset ankle edema. In the 51. Which one of the following is a cause of hirsutism?
past year she described joint pains increasingly cold A. Hypothyroidism
numb hands and an intermittent rash on her face. She B. Anorexia nervosa
was pale and weak urinalysis was positive for protein C. Penicilliamine
and blood. The ESR was 92 and she had D. Psoralens
lymphocytopenia. The most likely diagnosis is? E. Polycystic ovarian Syndrome
A. Hypersensitivity vasculitis
B. Lupus Nephritis 52. A male presents with severe headache, lethargy
C. Polyarteritis nodosa and weight loss. Which of the following would make the
D. Rheumatoid arthritis diagnosis of giant cell arteritis likely?
E. Wegener's granulomatosis A. The patient is below 50 years
B. Papilledema on fundoscopy
47. 16 years old girl presented with complain of loose C. Non-tender carotid arteries
motion after taking routine meal for 2 years. bloating, D. Occipital headache only
abdominal pain and iron deficiency anemia what is your E. Raised ESR
diagnosis?
A. Tropical sprue
B. Celiac disease
C. Chronic pancreatitis
D. Hyperthyroidism
E. Lymphoma

48. Which one of the following drugs works by inhibiting


the tumor necrosis factor?
A. Cyclosporin
B. Infliximab
C. Methotrexate
D. Montelukast
E. Sulphasalazine
44 Prof (MEDICINE 1) 13. Multiple Osteolytic lesions, hypercalcemia and
neurologic abnormalities are most often seen in
1. Syphilis follow up A. Hodgkin's lymphoma
Ans? B. Chronic myeloid Leukemia
C. Multiple Myeloma
2. Most common anomaly in Down's Syndrome D. CNS Tumors
Ans Endocardial Cushions E. Burkitt's lymphoma

3. Good prognosis in Sarcoidosis Multiple osteolytic lesion


Ans Erythema nodosum Ans Multiple Myeloma

4. Crushing chest pain 15 minutes which of following 14. Iron deficiency anemia
enzyme level raises first in Myocardial infarction. Ans Decreased Ferritin
A. Myoglobin
B. CK MB 15. Aortic dissection
C. Trop T Ans Bicuspid aortic valve
D. LDH
E. AST 16. Which is the risk factor for the Lung Cancer
Ans Passive Smoker
5. TB lung
Ans Silica 17. Most common risk factor for fungal ball in fungus is
A. Tuberculosis cavity
6. Hodgkin lymphoma B. Pulmonary infarction
Ans ABVD (Adriamycin, bleomycin, vinblastine, and C. Abscess cavity
dacarbazine) D. Pulmonary edema
E. Bronchiectasis
7. TB drug for first 2 months
Ans INH RIF ETH PZN 18. ICS (Inhaled corticosteroids)
Ans decrease all cause mortality
8. Meningitis dx viral Note: In asthmatic and COPD
Ans Acyclovir
19. Minimal change disease
9. Which of the following is initial therapy for Idiopathic Ans Disturb GBM
Thrombocytopenic Purpura (ITP)
A. Splenectomy 20. Immediate treatment of hyperkalemia is?
B. Fresh frozen plasma transfusion A. Intravenous 10% calcium gluconate 10ml over 10min
C. Plasmapheresis B. Intravenous 5% dextrose 250 ml/hour
D. Platelet transfusion C. Intravenous 0.9% saline at 500 ml/hour
E. Glucocorticoids D. Oral calcium resonium
E. Loop diuretics
10. Definitive therapy for idiopathic pulmonary fibrosis is
A. Steroids 21. Hyperventilation leads to?
B. Oxygen A. Metabolic Acidosis
C. Lung transplantation B. Metabolic Alkalosis
D. Pulmonary rehabilitation C. Respiratory Acidosis
E. Opiates D. Respiratory Acidosis with Metabolic Alkalosis
E. Respiratory Alkalosis
11. Minimal change disease
Ans Prednisolone 22. Haloperidol
Ans Acute dystonia
12. Significant Splenomegaly
Ans CML
23. 70 years obese man is admitted with 6 hours history 34…
of left sided chest pain.An ECG reveals inferior wall MI 34…
which of following confirm diagnosis 35…
A. LDH
B. Creatinine kinase (MEDICINE 2)
C. SGPT
D. Troponin T 1. Parkinson + Rash
E. C-reactive protein Ans Seborrheic rashes

24. Early diastolic murmur occur in 2. Atopic dermatitis, not risk factor
A. Atrial septal defect Ans UV Rays
B. Aortic regurgitation
C. Hypertrophic obstructive cardiomyopathy 3. Specific test for diagnosis of multiple sclerosis is:
D. Mitral stenosis A. CT scan brain
E. Mitral regurgitation B. Delayed visual evoked response
C. Lumbar puncture
25. Toxic shock syndrome D. MRI brain
Ans Pyogenic exotoxin E. X-Ray skull

26. Groin pain fever RBC WBC cast 4. Hereditary Spherocytosis


Ans Acute pyelonephritis Ans Splenectomy

27. Whooping cough 5. Patient with Bell's palsy has pathology involving
Ans bordetella pertussis which cranial nerve?
A. III
28. Prosthetic valve B. V
Ans S.epidermidis C. VII
D. XI
29. Normal anion gap metabolic acidosis occurs E. XII
in?
A. Lactic acidosis 6. Sharp fascial Pain, worsen by eating
B. Alcoholic ketoacidosis Ans Trigeminal neuralgia
C. Diabetic ketoacidosis
D. Renal tubular acidosis 7. Atrophy of tongue, fasciculation
E. Methanol overdose Ans Bulbar Palsy

30. A patient comes in emergency department with 8. Insulin independent organ


chest pain and dyspnea, on ECG SHOWS ST Elevation Ans Brain
on lead II, III Avf which artery is involved
Ans Right coronary artery 9. Following is a DNA virus.
A. Hepatitis A virus
31. Drug of choice for bipolar disorders is ? B. Hepatitis B virus
A. Sodium Valproate C. Hepatitis C virus
B. Carbamazepine D. Hepatitis D vinis
C. Lithium carbonate E. Hepatitis E virus
D. Citalopram
E. Risperdal 10. Gluten Free Diet
DOC for bipolar Ans celiac disease
Ans Lithium
11. Anti Ttg
32. Nephritic in adult Ans celiac disease
Ans..
12. Cobble stones 24. Which one among following Is not characteristic
Ans Crohn disease features of Addison disease?
A. Hyperpigmentation
13. Complications of DM II B. Hypertension
Ans HONK (Hyperosmolar Nonketotic State) C. Hypoglycemia
D. Lymphocytosis
14. AMA+ve, Pruritus E. Neutropenia
Ans PBC
Not related to Addison Disease
15. Hx of ear discharge + hemiplegia Ans Hypertension
Ans Brain abscess
25. GERD predispose to
16. Jaw claudication + Headache Ans Adenocarcinoma
Ans Temporal Arteritis
26. Hepatitis Not Chronic
17. Pheochromocytoma Ans Hep: A.
Ans Surgery (TOC)
27. A 22-year-old female student has developed Joint
18. Primary hypothyroidism pains, anorexia, amenorrhoea and upper abdominal
Ans ↑ TSH T3 T4 pain for 10 months. She is jaundiced, has acne on face,
liver is palpable 3 cm with a total span of 15 ems.
19. In pituitary damage, which of the following Spleen is palpable for 2 cm. Pedal edema and shifting
hormones will be lost last? dullness are absent. Most likely she is having:
A. ACTH A. Chronic hepatitis B
B. GH B. Chronic Hepatitis C
C. FSH C. Cirrhosis liver
D. LH D. Autoimmune hepatitis
E. TSH E. Hemochromatosis

20. Diabetic neuropathy 28. Most diagnostic investigation for acute pancreatitis
Ans impotence on the 4th day of onset is:
A. Blood counts
21. Organophosphate Poisoning B. Serum amylase
Ans Atropinization C. Serum lipase
D. X-ray abdomen
22. Amoebic liver E. Ultrasound abdomen
Ans 90% antibody tre in stool.
29. 42 years old woman with chronic arthritis
23. A 25-year-old female having gestational complaining of grittiness in eyes and dry mouth, +ve
amenorrhea for the last three months develops signs & schirmer's test. RA factor and anti Ro/La antibodies are
symptoms of grave's disease. Which of the following is positive. What is diagnosis?
the best treatment option? A. SLE
A. Radioactive iodine B. Felty's syndrome
B. Propylthiouracil C. Still's disease
C. Methimazole / Neomercazole D. Sjogren's syndrome
D. Thyroidectomy E. Episcleritis
E. Propranolol
(In key its B)
30. Lower motor neuron type of paraparesis is
characterized by:
A. Brisk reflexes
B. Hypotonia
C. Positive kerning sign
D. Spasticity
E. Up-going plantars

31. Parkinson Disease


Ans loss of Substantia Nigra

32. DKA
Ans Diabetes Melitus I

33. Peptic ulcer Dx Endoscopic Biopsy


34. Classical triad of Reiter's syndrome is
A. Arthritis, Conjunctivitis, urethritis
B. Anemia, arthritis, Sacroiliitis
C. Anemia, rash, arthritis
D. Rash, conjunctivitis, stomatitis
E. Anemia, rash, Uveitis

35. SLE Scenenio


Ans ANA
43 Preprof 7. Which of the following is initial therapy for Idiopathic
Thrombocytopenic Purpura (ITP)
1. Herpes zoster occurs after infected with A. Splenectomy
A. small pox B. Fresh frozen plasma transfusion
B. Chicken pox C. Plasmapheresis
C. Measles D. Platelet transfusion
D. Mumps E. Glucocorticoids
E. Rubella
8. 35 year old presents with fever and lymphadenopathy
2. Cause of death in established tetanus is bone marrow biopsy confirms Hodgkin Lymphoma
A. Spinal cord compression which one among following type has best prognosis
B. Septic shock A. Nodular sclerosis
C. Renal failure B. Lymphocyte predominant
D. Respiratory failure C. Lymphocyte depleted
E. Pulmonary embolism D. Mixed cellularity
E. Promyelocytic
3. Incubation period of Rabies is
A. 24 hours 9. Multiple Osteolytic lesions, hypercalcemia and
B. 2 to 3 days neurologic abnormalities are most often seen in
C. 2 to 3 weeks A. Hodgkin's lymphoma
D. 3 to 7 weeks B. Chronic myeloid Leukemia
E. 6 to 12 months C. Multiple Myeloma
(Verify) D. CNS Tumors
E. Burkitt's lymphoma
4. Most common opportunistic fungal infection is with
A. Candida 10. 15 Years old patient presents with history of fever,
B. Aspergillosis bleeding from gums and pallor for last 20 days his
C. Pneumocystis jirovecii peripheral blood smear shows pancytopenia most
D. Cryptococcus important investigation is
E. Blastomycosis A. Serum Iron level
B. Serum Folic acid level
5. Intestinal obstruction is most likely to occur as C. Coombs test
complication of infestations with D. Reticulocyte count
A. Enterobius vermicularis E. Bone marrow examination
B. Nectar americans
C. Ascaris Lumbricoides 11. Which of the following parameters is increased in
D. Strongyloides stercoralis obstructive lung disease
E. Wuchereria bancrofti A. FEVI
B. FEV1/FVC
6. In presence of Anemia, the ability to produce C. Total lung Capacity
adequate circulating red blood cell is best measured by D. Peak expiratory flow rate
A. WBC count E. FEV & FEV1/FVC
B. Serum iron binding capacity
C. Reticulocyte count 12. Immediate treatment of Tension Pneumothorax is
D. Total eosinophil count A. Chest intubation
E. Total Lymphocyte count B. Needle decompression
C. Bronchodilators
D. Steroids
E. Antibiotics
13. Cushing syndrome and SIADH as paraneoplastic 19. Blood pressure of 170/130mmHg with retinal
syndrome is caused by which of the following lung hemorrhage suggest
cancer A. White coat hypertension
A. Small cell carcinoma B. Malignant hypertension
B. Large cell carcinoma C. Hypertensive urgency
C. Squamous cell carcinoma D. Hypertension emergency
D. Adenocarcinoma E. Stage 2 Hypertension
E. Carcinoid tumor
20. 70 years obese man is admitted with 6 hours history
14. 40 years old man suddenly develops shortness of of left sided chest pain.An ECG reveals inferior wall MI
breath, tachypnea following leg surgery. Chest X-ray is which of following confirm diagnosis
normal. The most common underlying cause is A. LDH
A. Asthma B. Creatinine kinase
B. Pulmonary embolism C. SGPT
C. Fat embolism D. Troponin T
D. CCF E. C-reactive protein
E. ARDS
21. Which of following medication should be avoided in
15. Most common risk factor for fungal ball in fungus is severe Aortic stenosis
A. Tuberculosis cavity A. Digitalis
B. Pulmonary infarction B. Beta blockers
C. Abscess cavity C. Diuretics
D. Pulmonary edema D. Aspirin
E. Bronchiectasis E. Antibiotics therapy in bacterial Endocarditis

16. Status Asthmatics is defined as failure of asthma to 22. Which of the following condition could result in High
resolve with therapy in cardiac failure
A. 6 hours A. Hypertension
B. 12 hours B. Valvular heart disease
C. 24 hours C. Thyrotoxicosis
D. 42 hours D. Dilated cardiomyopathy
E. One Week E. Cor pulmonale

17. Pulmonary hypertension is diagnosed when mean 23. Saw tooth appearance on ECG tracing is diagnostic
pulmonary artery pressure at rest is of
A. 15 mmHg A. Ventricular tachycardia
B. 20 mmHg B. Atrial Flutter
C. 25 mmHg C. Atrial Fibrillation
D. 30 mmHg D. Ventricular Fibrillation
E. 35 mmHg E. WPW Syndrome

18. Definitive therapy for idiopathic pulmonary fibrosis is 24. Valvular Abnormalities and Chamber Dilatation is
A. Steroids best diagnosis by
B. Oxygen A. ECG
C. Lung transplantation B. ECHO
D. Pulmonary rehabilitation C. Nuclear imaging techniques
E. Opiates D. Treadmill exercise testing
E. Troponin T
25. Which of the following drug regimen should be 31. The most common appearance of Glomeruli in adult
started in acute management of deep vein thrombosis with nephritic syndrome is
(DVT) A. Proliferative Glomerulonephritis
A. Start Aspirin immediately B. Membranous Glomerulonephritis
B. Start Warfarin immediately C. Wire loop
C. Start Aspirin and Warfarin together immediately D. Normal appearance of Glomeruli
D. Start Heparin followed by Warfarin E. Crescent glomerulonephritis
E. Give Streptokinase
32. 48 years old male presents with hemoptysis,
26. Which of following feature is more common in microscopic hematuria and mass at the upper pole of
Constrictive Pericarditis than in Cardiac Tamponade the right kidney. What is the most likely diagnosis?
A. Pulsus paradoxus A. Good pasture's syndrome
B. Kussmaul sign B. Post streptococcal glomerulonephritis
C. Four chamber diastolic equilibrium C. Renal cell carcinoma
D. Hypotension D. IgA nephropathy
E. Prominent X-trough E. Focal segmental glomerulosclerosis

27. In community the most common disorder over age 33. Most acceptable explanation of Oliguria of acute
of 65 years is renal failure is
A. Alzheimer's disease A. Rise in blood pressure
B. Late onset Schizophrenia B. Reduced Glomerular Filtration rate (GFR)
C. Mood disorders C. Increased Glomerular Filtration rate
D. Delirium D. Prerenal failure
E. Vascular dementia E. Hypokalemia
(Verify)
34. Hyperventilation leads to?
28. Which of the following most common type of A. Metabolic Acidosis
Hallucination in people suffering from dementia B. Metabolic Alkalosis
A. Functional C. Respiratory Acidosis
B. Gustatory D. Respiratory Acidosis with Metabolic Alkalosis
C. Olfactory E. Respiratory Alkalosis
D. Tactile
E. Visual 35. A 55-years-old man presented with generalized
weakness he recently started furosemide. The mont
29. Which of the following antidepressant in elderly likely electrolyte disturbance developed is
A. Amitriptyline A. Hypernatremia
B. Imipramine B. Hypokalemia
C. Moclobemide C. Hyponatremia
D. Sertraline D. Hypocalcemia
E. Venlafaxine E. Hyperkalemia

30. 22 Years old pregnant female has flank pain and 36. Which of following is most sensitive for diagnosis
fever Her urine contains WBCS cast what is likely H-pylori
diagnosis A. Urea breath test
A. Chronic Glomerulonephritis B. Barium meal examination
B. Analgesic nephritis C. Stool test
C. Acute pyelonephritis D. Urine test
D. Ectopic pregnancy E. Ultrasound abdomen.
E. Preeclampsia
37. Most useful immediate marker of acute pancreatitis 43. A 22-year-old female student has developed Joint
is: pains, anorexia, amenorrhoea and upper abdominal
A. Serum amylase pain for 10 months. She is jaundiced, has acne on face,
B. Serum lipase liver is palpable 3 cm with a total span of 15 ems.
C. Serum elastase Spleen is palpable for 2 cm. Pedal edema and shifting
D. Serum trypsinogen dullness are absent. Most likely she is having:
E. Serum carboxy polypeptidase A. Chronic hepatitis B
B. Chronic Hepatitis C
38. Which of the inflammatory bowel disease is most C. Cirrhosis liver
commonly associated with Primary sclerosing D. Autoimmune hepatitis
cholangitis? E. Hemochromatosis
A. Ulcerative colitis
B. Crohn's disease 44. Following is the parameter of Child-pugh
C. Microscopic colitis classification:
D. Irritable bowel syndrome A. Anemia
E. Lymphocytic colitis B. SGPT
C. Prothrombin time
39. Regarding chronic Hepatitis C infection: D. Pedal edema
A. 80% patients are deeply jaundiced E. GCS
B. More than 90% of patients spontaneously become
seronegative. 45. Wilson Disease is managed with
C. 10% of patients may develop acute liver failure A. Beta interferon
D. Antibodies to HCV are protective B. Lamivudine
E. Can be treated with pegylated interferon with ribavirin C. Alpha interferon
D. Penicillamine
40. Following is the significant finding in Chronic Liver E. Corticosteroids
Disease?
A. Anemia 46 In motor neuron disease
B. Thrombocytosis. A. There is sudden onset of muscular weakness
C. Leukocytosis. B. Both upper and lower motor neuron sings are present
D. Neutropenia. C. In primary lateral sclerosis deep tendon reflexes are
E. Thrombocytopenia. absent
D. MRI brain in diagnostic tool
41. Following is a DNA virus. E. CSF analysis is needed to start treatment
A. Hepatitis A virus
B. Hepatitis B virus 47. Investigation of choice for Guillain-Barre syndrome
C. Hepatitis C virus (GBS)
D. Hepatitis D vinis A. Complete Blood Count
E. Hepatitis E virus H. Serum electrolytes
C. Nerve conduction velocity
42. No effective Vaccination is available for: D. MRI brain
A. Hepatitis C virus E. CT scan brain.
B. Hepatitis B virus
C. Hepatitis A virus 48. Regarding Ischemic stroke:
D. Steatohepatitis A. It is gradual onset of muscular deficit
E. Autoimmune hepatitis B. CSF analysis is required for diagnosis
C. It can be treated with disprin
D. Blood CBC is diagnostic
E. Passing Nasogastric tube for feeding is mandatory
49. Regarding parkinson's disease:
A. Presence of hyperreflexia is needed for diagnosis 56. Common clinical feature of Cushing's syndrome is:
B. Cogwheel rigidity is its characteristic feature A. Weight loss
C. It can be treated with Riluzole B. Skin Pigmentation
D. Baclofen is a drug of choice C. Moon shape face with Hirsutism
E. Anticholinergic drugs are contraindicated D. Hypotension
E. Tachycardia
50. Drug treatment of absence seizures includes:
A Phenytoin 57. Thyroid gland produces following hormones except:
B. Carhumazepine A. Thyroglobulin
C. Lacosamide B. Tetra-iodothyronine
D. Ethosuximide C. Tri-iodothyronine
E. Valproic acid D. Mono-iodothyronine
E. Parathormone
51. Regarding multiple sclerosis:
A. It is degenerative disease of more than 60 years of 58. A 25-year-old female having gestational
age amenorrhea for the last three months develops signs &
B. It is the disease of peripheral nerves symptoms of grave's disease. Which of the following is
C. Around 80% of patients have relapsing & remitting the best treatment option?
clinical course A. Radioactive iodine
D. Visual evoked potentials have no role in diagnosis B. Propylthiouracil
E. X-Ray brain is diagnostic C. Methimazole / Neomercazole
D. Thyroidectomy
52. Following is not the feature of acute adrenal crisis! E. Propranolol
A. Hyperkalemia (In key its B)
B. Hypoglycemia
C. Severe hypertension 59. Which of the following is not a feature of Cushing's
D. Hyponatremia syndrome?
E. Hypocalcemia A. Easy bruising
B. Proximal myopathy
53. Biochemical findings in primary Hypothyroidism C. Hypertension
include: D. Psychosis
A. Normal TSH and low T3 and T4 levels E. Peripheral neuropathy
B. Elevated TSH, low T3 & T4 levels
C. Elevated TSH, normal T4 & T3 level 60. A 20-years-old female primigravida presented in the
D. Elevated TSH, T3 & T4 level first trimester of pregnancy with blood sugar 300 mg/dL.
E. Low TSH, Low T3 & T4 level Which one among the following drugs is most
appropriate for the control of her blood sugar?
54. Features of Addison's disease include: A. Sulfonylurea
A. Hypopigmentation B. Insulin
B. Hypotension C. Biguanides therapy
C. Hirsutism D. Acarbose
D. Hyperglycemia E. Sulfonyl ureas & biguanides
E. Cardiomegaly on X-Ray Chest

55. Drug used in the treatment of Cushing's syndrome


is:
A. Thyroxine
B. Prednisolone
C. Ketoconazole
D. High dose multivitamin
E. Synthetic ACTH
61. A 16-years-old patient arrives at the emergency 67. Classical triad of Reiter's syndrome is
department with severe thirst, dehydration and A. Arthritis, Conjunctivitis, urethritis
confusion. The patient is breathing rapidly and has a B. Anemia, arthritis, Sacroiliitis
fruity smell. His blood sugar is 578 mg/dl, pH of C. Anemia, rash, arthritis
7.3. The likely diagnosis is: D. Rash, conjunctivitis, stomatitis
A. Diabetics Ketoacidosis E. Anemia, rash, Uveitis
B. Hyperosmolar Hyperglycemic nonketotic coma.
C. Hypoglycemia 68. Backache of Ankylosing spondylitis does not
D. Diabetic Neuropathy improve:
E. Cushing's Disease A. At rest
B. During exercise
62. Following are the three most common initial C. With Badminton
symptoms of diabetes Mellitus type-II? D. In Swimming
A. Headache, Vomiting & Fatigability E. With Jogging
B. Polydipsia, polyuria & weight loss
C. Double vision, weight gain & increase heart rate 69. Radiological finding of bamboo spine is present in:
D. Tingling of feet, weight loss & normally healing of A. Rheumatoid arthritis
wounds. B. SLE
E. Increased appetite, vertigo & vomiting C. Psoriatic arthritis
D. Ankylosing spondylitis
63. Regarding diabetes mellitus: E. Osteoarthritis
A. Type I DM is commonly seen in persons above about
40 years 70. RA factor is absent in:
B. Type II DM tends to occur only in thin persons A. Rheumatoid Arthritis
C. In Type I DM, oral hypoglycemic drugs are effective B. SLE
D. Serum insulin level is absolutely absent in type II DM C. MCTD
E. HbA1C is a good monitor to the diagnosis of the D. Polymyositis
diabetes mellitus E. Psoriatic Arthritis

64. Screening test for SLE is:


A. Anti CCP
B. Antinuclear antibodies
C. Anti-Jo antibody
D. RA Factor
E. Anti-SCL. 70

65. Which antibody is more specific for Rheumatoid


A. Anti dsDNA
B. Antinuclear antibodies
C. Anti-smith antibody
D. Anti CCP
E. Anti-histone antibody

66. Which type of anemia is included in diagnostic


criteria for SLE?
A. Microcytic anemia
B. Megaloblastic Anemia
C. Autoimmune haemolytic anemia
D. Sideroblastic anemia
E. Iron deficiency anemia
43 Prof (MEDICINE 1) 11. A 22-year-old patient presented with sensori-neural
deafness and renal failure. Investigations reveals:
1. Normal anion gap metabolic acidosis occurs positive Anti glomerular antibodies. What is diagnosis?
in? A. Alport syndrome
A. Lactic acidosis B. Down syndrome
B. Alcoholic ketoacidosis C. Giant syndrome
C. Diabetic ketoacidosis D. Good pasture syndrome
D. Renal tubular acidosis E. Wagner disease
E. Methanol overdose
12. Prosthetic Valve Endocarditis
2. Typhoid treatment Ans Staphylococcus Epidermidis
Ans Fluoroquinolones
13. A patient with pansystolic murmur with tricuspid
3. Varicella zoster occurs after infected with regurgitation
A. small pox Ans VSD
B. Chicken pox
C. Measles 14. Most common valvular defect in Marfan syndrome
D. Mumps Ans Dilation of Aortic Sinuses

4. A patient with Bilateral Cervical lymphadenopathy 15. V. Wave in JVP shows


and red lips Ans Passive filling of right atrium
Ans Kawasaki Disease
16. Most common anomaly in Down's Syndrome
5. P Vivax treatment Ans Endocardial Cushions
Ans Chloroquine + Primaquine
17. A patient with renal complains friction rib heard on
6. Diagnosis of tuberculous leprosy rather than apex and left sternal border
lepromatous leprosy Ans Fibrinous Pericarditis
Ans Acid fast bacilli Skin Smear
18. Definitive therapy for idiopathic pulmonary fibrosis is
7.Immediate treatment of hyperkalemia is? A. Steroids
A. Intravenous 10% calcium gluconate 10ml over 10min B. Oxygen
B. Intravenous 5% dextrose 250 ml/hour C. Lung transplantation
C. Intravenous 0.9% saline at 500 ml/hour D. Pulmonary rehabilitation
D. Oral calcium resonium E. Opiates
E. Loop diuretics
19. Common pulmonary complications of SLE Include:
8. Treatment of Pauci Immune Glomerulonephritis A. Bronchogenic carcinoma
Ans Prednisolone Cyclophosphamide B. Pneumothorax.
C. Stridor.
9. Polyuria D. Pulmonary nodules.
Ans Greater than 3L E. Pleural effusion

10. Anemia in Chronic Renal failure is maintained with 20. Most common opportunistic fungal infection is with
Ans Erythropoietin A. Candida
B. Aspergillosis
C. Pneumocystis jirovecii
D. Cryptococcus
E. Blastomycosis

Most common fungal infection


Ans Candidiasis
21.Pulmonary hypertension is diagnosed when mean 29. A Patient with AML poor prognosis
pulmonary artery pressure at rest is Ans Cytogenetic abnormalities
A. 15 mmHg
B. 20 mmHg 30. Which of the following antidepressant in elderly
C. 25 mmHg A. Amitriptyline
D. 30 mmHg B. Imipramine
E. 35 mmHg C. Moclobemide
D. Sertraline
22. Cushing syndrome and SIADH as paraneoplastic E. Venlafaxine
syndrome is caused by which of the following lung
cancer 31. Which one of the following features is common in
A. Small cell carcinoma both delirium and schizophrenia?
B. Large cell carcinoma A. A family history of psychopathy
C. Squamous cell carcinoma B. Hallucination
D. Adenocarcinoma C. Memory impairment
E. Carcinoid tumor D. Social withdrawal
E. Waxing and waning of symptoms
23. Presence/Accumulation of pus in pleural space is
termed as? 32. Treatment option for newly diagnosed schizophrenia
A. Para pneumonic effusion Ans Atypical antipsychotic
B. Chylothorax
C. Emphyema 33. B2 Agonist
D. Exudative effusion Ans Relaxing the respiratory smooth muscle
E. Hemorrhagic effusion
34. Blood pressure of 170/130mmHg with retinal
24. At PT with squamous cell carcinoma undergoing for hemorrhage suggest
surgery which of the following contraindication for A. White coat hypertension
surgery B. Malignant hypertension
Ans Superior Vena Cava Obstruction C. Hypertensive urgency
D. Hypertension emergency
25. Woman has history prolonged consumption of E. Stage 2 Hypertension
alcohol CBC shows MCV 112 FL taking phenytoin for
convulsion most likely diagnosis 35. Status Asthmatics is defined as failure of asthma to
Ans Folate deficiency resolve with therapy in
A. 6 hours
26. An 18-year-old male with sickle cell disease B. 12 hours
presents with osteomyelitis. Which organism is C. 24 hours
responsible for this problem? D. 42 hours
A. E-coli E. One Week
B. Group A streptococci
C. Mycobacterium tuberculosis Status asthmaticus
D. Pseudomonas Ans 24 hours
E. Salmonella

27. PNH diagnosis


Ans flow cytometry for CD55 CD59

28. A Pt heavy nasal bleeding with APTT increase, PT


increased fibrinogen 0.7(1.8-8)
Ans Cryoprecipitate
(MEDICINE 2) 7. A farm worker presented to the ER with finding of pin
point pupils, sweeting, bronchoconstriction, diarrhea
1. A 54 year old man with small cell lung cancer and muscles fasciculation. What is the best treatment?
complains of muscles weakness. Which one of the A. Administer sodium bicarbonate
following is not feature of Lambert Eaton Syndrome: B. Naloxone
A. Proximal muscles more commonly affected C. Pralidoxime
B. Hyporeflexia D. Lomotil
C. Dry mouth E. All of these
D. Impotence
E. Repeated muscles contractions lead to decreased 8. Dose of Atropine that should be given in case of
muscles strength Organophosphorus compound poisoning is:
A. Atropine 1mg
2. Drug treatment of absence seizures includes: B. Atropine 2-4mg
A Phenytoin C. Atropine 1 ampoule
B. Carhumazepine D. Atropine should be administered until full
C. Lacosamide atropinization occur
D. Ethosuximide E. Atropine should be administered until cyanosis
E. Valproic acid disappears

3. First line anti-epileptic drug for myoclonic seizures is: 9. Which one the following condition is most strongly
A. Carbamazepine associated with erythema multiforme?
B. Clonazepam A. Crohn's disease
C. Ethosuximide B. Herpes simplex virus
D. Sodium Valproate C. Sarcoidosis
E. Topiramate D. Streptococcal infection
E. Tuberculosis
4. Which one of the following is most associated with
down beat nystagmus 10. A 54 year old man presents with a brown velvety
A. Arnold chiari malformation rash on the back of .. and around axilla. A clinical
B. Acoustic neuroma diagnosis of acanthosis nigricans is made. Which of the
C. Cerebellar vermis lesion following condition is most associated with this kind of
D. Jugular foramen syndrome rash:
E. Pseudo bulbar palsy A. Hypothyroidism
B. Psoriasis
5. Regarding stroke: C. Tuberculosis
A. It is gradual onset of muscular deficit D. Ulcerative colitis
B. CSF analysis is required or diagnosis E. Acute pancreatitis
C. It can be treated with disprin
D. Blood CBC is helpful for diagnosis 11. Glucose goal in patients with diabetic ketoacidosis
E. Passing Nasogastric tube for feeding is mandatory is:
A. 70-100 mg/dl
6. Regarding Parkinson's Disease: B. 100-150 mg/dl
A. Presence of rigidity is needed for diagnosis C. 150-250 mg/dl
B. Loss of postural reflex is characteristic D. 250-300 mg/dl
C. It can be treated with Riluzole E. 300-350 mg/dl
D. Baclofen is a drug of choice
E. Anticholinergic drugs are contraindicated
12. Following are the three most common initial 18. Best initial Investigation for hemochromatosis is:
symptoms of diabetes Mellitus type-II? A. Transferrin saturation + ferritin level
A. Headache, Vomiting & Fatigability B. Hematocrit + ferritin level
B. Polydipsia, polyuria & weight loss C. Liver biopsy with perl's stain
C. Double vision, weight gain & increase heart rate D. Serum iron + ferritin level
D. Tingling of feet, weight loss & normally healing of E. Serum iron + hematocrit
wounds.
E. Increased appetite, vertigo & vomiting 19. Which protein synthesized by liver is commonly and
significantly is elev hepatocellular carcinoma:
13. Regarding diabetes mellitus: A. Albumin
A. Type I DM is commonly seen in persons of about 40 B. Alpha fetoprotein
years C. Protein C
B. Type II DM tends to occur in obese persons D. Thyroxine binding globulin B
C. In Type I DM, oral hypoglycemic drugs are effective E. C reactive protein
D. Serum insulin level is absolutely absent in type II DM
E. HbA1c is a good monitor for the diagnosis of 20. A patient with upper Gl symptoms, test positive for H
diabetes mellitus pylori following Breath Test. Which one of the following
condition is most strongly with H pylori infection:
14. Painful neuropathy in feet, treatment option is A. Gastric adenocarcinoma
Ans Duloxetine B. Gastro oesophageal reflex disease
C. Oesophageal Cancer
15. A 40 years old male was admitted after an episode D. Duodenal ulceration
of haemtemsis secondary to oesophageal varices. E. Atrophic gastritis
Which of the following statement about UGI hemorrhage
is true? 21. A 27 year old female is investigated for bloody
A. I/V ranitidine reduce the risk of rebleed diarrhea. This started around six week ago, she is
B. I/V nitrates reduce the risk of rebleed currently passing 3 to 4 loose motion a day which
C. Oral omeprazole reduce the risk of rebleed normally contain a small amount of blood. Other than
D. Oral tranexamic acid reduce the risk of rebleed feeling lethargic she remains systemically well with no
E. I/V somatostatin analogue reduces the risk of rebleed fever or significant abdominal pain. A colonoscopy is
performed which shows inflammatory changes in the
16. Which one of the following is most often seen in ascending colon consistent with ulcerative colitis. Blood
patient with Wilson's Disease? shows, Hb 14.2 g/DL. PLATELETS 323000, WBCS
A. Presence of Kayser Fleisher ring 8000, CRP 22 mg/l. What is the most appropriate 1st
B. A raised copper level line medication to induce remission?
C. Bilateral Upper motor neuron signs A. Rectal aminosalicylate
D. Onset of symptoms usually between 50 and 75 years B. Oral aminosalicylate
E. Poor prognosis with treatment C. Oral prednisolone
D. Intravenous corticosteroids
17. A 36 years male with decompensated cirrhosis liver E. Rectal corticosteroids
presents with ascites and peripheral edema. Which one
of the following is true concerning management of 22. A 65 year old man with liver cirrhosis of unknown
ascites: cause is reviewed in clinic. Which one of the following
A. A low salt diet is un-necessary if patient is on factors is most likely to indicate poor pron
diuretics A. Alanine transaminase greater than 200 u/l
B. Loop diuretics are preferable to potassium sparing B. Caput medusa
agents C. Ascites
C. Paracentesis is less likely to be useful if patient has D. Gynecomastia
peripheral edema E. Splenomegaly
D. Paracentesis is contraindicated if the patient
prothrombin time is prolonged
E. Weight loss greater than 1.5 kg/day is the ideal
23. 72 years old female presented with diarrhea a 29. The laboratory findings in Rheumatoid Arthritis
Sigmoidoscopy shows mat Which plaque adherent Gl A. Anti ccp antibodies are positive
mucosa, what is most likely diagnosis? B. Rheumatoid factor positive in 50% cases
A. Crohn's disease C. ANA in 50% cases
B. Colorectal cancer D. ESR is normal
C. Ischemic colitis E. ESR and C reactive proteins slightly raised
D. Ulcerative colitis
E. Pseudomembranous colitis 30. Hypothyroidism is not feature of
A. Hashimoto's thyroiditis
24. 23 years old man is investigated for chronic diarrhea B. Postpartum thyroiditis
associated with inflammatory markers. A bowel biopsy C. Grave's disease
is taken which one of the foll most suggestive ulcerative D. Primary hyperparathyroidism
colitis: E. Radioactive iodine
A. Cobble stone appearance
B. inflammation affecting serosa 31. Which of the following drugs is the most appropriate
C. Goblet cell depletion medical treatment of hypertension associated with
D. Multiple granuloma Conn's Syndrome? for
E. Skip lesion A. Bendroflumethiazide
B. Altricial
25. In which of the following RA factor is not positive: C. Perindopril
A. Sjogren's syndrome D. Amlodipine
B. Felty syndrome E. Spironolactone
C. Infective endocarditis
D. SLE 32. A 41 year old woman presents with palpitations and
E. Polymyositis heat Intolerance. On examination her pulse is 90/min
and a small diffuse goitre is noted which is tender to
26. Osteoporosis (Marble Bone Disease) is due to touch. The thyroid function shows, free T4 24 pmo/l,
defect in: TSH less than 0.05 mull. What is the most likely
A. Osteoclast function diagnosis?
B. PTH receptors A. Grave's disease
C. Osteoblast function B. Sick thyroid syndrome
D. Calcium resorption in proximal tubule C. De Quervain Thyroiditis
E. Calcium absorption D. Hashimoto's thyroiditis
E. Toxic multinodular goiter
27. 42 years old woman with chronic arthritis
complaining of grittiness in eyes and dry mouth, +ve 33. A 43 years old man is found to have
schirmer's test. RA factor and anti Ro/La antibodies are phaeochromocytoma which antihypertensive medication
positive. What is diagnosis? should started first?
A. SLE A. Atenolol
B. Felty's syndrome B. Phenoxybenzamine
C. Still's disease C. Doxazosin
D. Sjogren's syndrome D. Propranolol
E. Episcleritis E. Rampril

28. An elderly patient with pain in his shoulders with 34. Which one of the following is recognized cause of
transient blindness and unilateral headache, what is hypokalemia associated with hypertension:
diagnosis A. Barter's syndrome
Ans Polymyalgia Rheumatica B. Ciclosporin
C. Gitelmen's syndrome
D. Liddle's syndrome
E. Renal tubular acidosis
35. 42 years old woman presents as she has noticed a
droop in the right side of her face since she woke up
this morning. There is no associated weakness,
dysphagia, or visual disturbances. On examination you
noticed right sided upper and lower facial paralysis.
Which one of the following would be most consistent
with a diagnosis of Bell's Palsy:
A. Vesicular rash around the ear
B. Hyperacusis
C. Sensory lose over the distribution of facial nerve
D. Pins and needles in right arm
E. Rhinorrhoea
42 Preprof ….. 9. Poor prognosis in Pneumonia seen
Ans Respiratory rate > 35 bpm
42 Prof (MEDICINE 1)
10. A 35-year-old presents with fever and
1. Which of following drug cause hyperkalemia? lymphadenopathy. Lymph node biopsy confirms
A. ACE inhibitors Hodgkin's Lymphoma. Which one amongst the following
B. Cephalosporin types has the best prognosis?
C. Loop Diuretic A. Nodular sclerosis
D. Laxatives B. Lymphocyte predominant
E. B2 agonist C. Lymphocyte depleted
D. Mixed cellularity
2. Hyperventilation leads to? E. Promyelocytic
A. Metabolic Acidosis
B. Metabolic Alkalosis 11. Initial therapy for Idiopathic Thrombocytopenic
C. Respiratory Acidosis Purpura (ITP) is:
D. Respiratory Acidosis with Metabolic Alkalosis A. Glucocorticoids
E. Respiratory Alkalosis B. Fresh frozen plasma transfusion
C. Plasmapheresis
3. Which of the following parameters is increased in D. Platelet transfusion
obstructive lung disease? E. Splenectomy
A. FEVI
B. FEV1/FVC 12. Aspergillus commonly seen in
C. FEV & FEV1/FVC Ans Tuberculous cavity
D. Peak expiratory flow rate
E. Total lung capacity 13. Which of the following is symptom of Depressive
Disorder.
4. Wave seen on JVP in tricuspid regurgitation A. Grandiosity
Ans V wave B. Early Morning Waking
C. Elation
5. Mass lesion seen commonly in HIV/AIDS patients D. Blunting
Ans Toxoplasma E. Breathlessness
(Verify)
6. Hyperuricemia caused by ATT
A. Isoniazid 14. Megaloblastic anemia seen in
B. Ethambutol Ans Folate and B12 deficiency
C. Pyrazinamide
D. Streptomycin 15. Panic disorder
E. Eritampucin Ans Acute Episódic anxiety

7. Pulmonary hypertension is diagnosed when mean 16. Regarding Aortic stenosis


pulmonary artery pressure at rest is? Ans slow rising Wave
A. 15 mmHg
B. 30 mmHg 17. Early Diastolic murmur
C. 25 mmHg Ans Aortic regurgitation
D. 30 mmhg
E. 35 mmHg 18. Glomerulonephritis that causes Partial lipodystrophy
Ans Mesengiocapillary Glomerulonephritis
8. Which investigation you will perform in infective
endocarditis
Ans Blood culture for streptococcus viridans
19. Which one of the following features is common in 27. A 48-year-old male presents with hemoptysis,
both delirium and schizophrenia? microscopic hematuria and mass at the upper pole of
A. A family history of psychopathy the right kidney. What is the most likely diagnosis?
B. Hallucination A. Focal segmental glomerulosclerosis
C. Memory impairment B. Good pasture's syndrome
D. Social withdrawal C. IgA nephropathy
E. Waxing and waning of symptoms D. Post streptococcal glomerulonephritis
E. Renal cell carcinoma
20. In presence of Anaemia, the ability to produce
adequate circulating red blood cell is best measured 28. A 28yr old lady with 38 weeks gestation period
by? present with pulse 100 beats/min RR 24, BP 110/60,
A. WBC count JVP is raised, Bilateral crackles are present, EFL 38%
B. Serum iron binding capacity DX
C. Reticulocyte count Ans Peripartum Cardiomyopathy
D. Total eosinophil count
E. Total Lymphocyte count 29. 40 years old man suddenly develops shortness of
breath and tachypnea following leg surgery, Chest X-ray
21. A diabetic patient, with nephrotic Syndrome the is normal. The most common underlying cause is?
proteinuria ++ Which of following prevents immediately A. Asthma
correct proteinuria B. Pulmonary embolism
Ans ACEI C. Fat embolism
D. CCF
22. Herpes/Varicella zoster occurs after infected with E. ARDS
A. small pox
B. Chicken pox 30. A patient with chest pain, SOB Comes in
C. Measles Emergency with acute myocardial infarction, St
D. Mumps Elevation on ECG, what will be immediate treatment
E. Rubella Ans Thrombolytic

23. A patient comes in emergency department with 31. Which is the risk factor for the Lung Cancer
chest pain and dyspnea, on ECG SHOWS ST Elevation Ans Passive Smoker
on lead II, III Avf which artery is involved
Ans Right coronary artery 32. AML is investigated by
Ans Bone marrow trephine biopsy
24. Acute Exacerbation of Asthma is Caused by
Ans Influenza 33. Which of following is not seen in Rabies
Ans opisthotonus
25. A patient with CVP Line, he is probably of risk of
organism catalase positive and coagulase -ve 34. In COPD inhaler corticosteroid is used for
Ans Streptococcus Epidermidis Ans Decrease the Severity of exacerbation of COPD

26. A 22-Year-old pregnant female has flank pain and 35. A 70yr old man, history A.fib, on treatment of ACEI,
fever. Her urine contains WBCS cast. What is the likely Diustic, Aspirin, After heat develops Episodes of
diagnosis? myocardial infarction/chest pain attack, with woolaxen of
A. Acute pyelonephritis left arm, CNS and Cardio Examination on
B. Analgesic nephritis echocardiography is otherwise normal what will be your
C. Chronic Glomerulonephritis next Step
D. Ectopic pregnancy Ans No Action
E. Pre-eclampsia
(MEDICINE 2) 11. Most common non-iatrogenic Cause of Cushing
syndrome
1. Lumbar puncture is safely done at? Ans Pituitary Adenoma
A. L4-L5
B. L5-S1 12. A patient with pruritus, more marked at night
C. L2-L3 Ans Scabies
D. L1-L2
E. T12-L1 13. Which of the following is least common.
Characteristic of Addinson's disease
2. 1st line of Treatment for Herpes Zoster Ans Metabolic Alkalosis
Ans oral acyclovir
14. Most common cause of Addinson's Disease
3. In a patient with ascending paralysis there is Ans Autoimmune
subsequently respiratory muscle Involvement. CSF
examination showed albuminocytological dissociation. 15. A patient positive for Anti-HCV antibody LFTis are
Treatment to be given is: normal, u/s abdomen is normal what is next appropriate
A. Oral Prednisolone step?
B. IV methylPrednisolone Ans PCR
C. IV immunoglobulins
D. Cyclosporin 16. Which of following is marker of replication of
E Dexamethasone Hepatitis B
Ans HBV-DNA
4. GERD predispose to
Ans Adenocarcinoma 17. A 22 old boy presents with sudden onset of
unilateral headache, transient loss of eye site
5. Which of following will not occur due to H. pylori (Blindness), left shoulder pain
Ans GERD Ans polymyalgia Rheumatica

6. A woman in the first trimester of pregnancy was 18 A unknown patient of cirrhosis, develop, pain an
found to have a blood sugar level of 300mg. What abdomen, tenderness, fever, what appropriate step
would be the most appropriate treatment for her? Ans Ascitic fluid DR
A. Sulfonylurea
B. Acarbose 19. For Prognostic factor Malignant melanoma
C. Acetylcholine Ans Depth of invasion
D. Insulin
E. Biguanides 20. Gastric Lavage is contraindicated in
A. Aspirin poisoning
7. Diagnostic investigation of choice for celiac disease B. Corrosive alkali poisoning
Ans IgA transglutaminase C. Diazepam poisoning
D. Iron tablet poisoning
8. Sensitive investigation for acromegaly E. Paracetamol poisoning
Ans oral glucose tolerance test with GH
21. Histopathological finding in ulcerative colitis is
9. Minimum Limit of HbA1c for DM-II diagnosis Ans Crypt Abscess.
Ans 6.5
22. A condition in which secondary Hyper- Lipedermia,
10. A Atropine dose for the organophosphate poisoning Hypercholesterolemia opposes to Triglyceridermia
(patient with miosis, storm of breath Hypotensive Ans Diabetes Melitus
Ans Until secretions are controlled
23. Marker of Polymyositis
Ans Anti Jo antibody
24. A 28 yr old patient presents with Dysarthria, 34. Spontaneous Bacterial Peritonitis is managed with:
Nystagmus, Extensor babinski sign, Ankle and knee A. Benzylpenicillin 6 million units 6 hourly.
Reflex are absent, optic Neuritis ?? B. Cefotaxime 1gm OD.
Ans MS?? C. Cefotaxime 1gm 8 hourly.
D. Cefotaxime 2gm 8 hourly.
25. A patient with Dementia, urinary incontinence Gait E. Clarithromycin + amoxicillin
problem
Ans Normal pressure hydrocephalus 35. Mallory bodies are characteristically present in?
A. Alcoholic hepatitis
26. bilateral facial nerve palsy occurs in B. Alcoholic fatty liver
Ans Sarcoidosis C. Hepatitis B
D. Wilson disease
27. A pt comes with Hypotension, pupil constriction E. Primary biliary cirrhosis
Cyanosis
Ans Phylepidrine

28. Lateral medullary syndrome involves


Ans Posterior inferior cerebellar artery

29. A 25 year old woman without any past medical


history presents with a 3 month history of arthralgia.
She had no past medical history of note. Examination of
swelling of the distal interphalangeal joints of the middle
and beg fingers of the hand and wrist on the right plus a
Swollen left ankle Investigations show. ESR 40mm/hr
(0-10). Most likely diagnosis?
A. Acute exacerbation of osteoarthritis
B. Psoriatic arthropathy
C. Rheumatoid arthritis
D. Reactive arthritis
E. SLE

30. A boy presents with headed, fever and Neck


stiffness
Ans CSF Examination positive menigism

31. A 85 yr old man with iron deficiency anemia with


unknown cause what investigation to follow
Ans Colonoscopy

32. In a lower motor neuron lesion, the following


develops?
A. Spasticity
B. Hyperreflexia
C. Babinski’s sign
D. Muscle wasting
E. Hyperesthesia

33. Treatment of choice in Wilson's disease


Ans Pencilinamike + Zn
41 Preprof 7. A 25-year-old female having gestational amenorrhea
for the last three months develops signs & symptoms of
1. Irregularly irregular pulse is observed in: grave's disease. Which of the following is the best
A. Atrial fibrillation. treatment option?
B. Sinus Bradycardia. A. Radioactive iodine
C. Complete Heart Block B. Propylthiouracil
D. Sinus Tachycardia C. Methimazole/ neomercazole
E. Hypertension D. Thyroidectomy.
E. Propranolol
2. Pansystolic murmur in found in (Verify)
A. Aortic Regurgitation
B. Mitral Regurgitation 8. When starting T4 therapy for an elderly patient with
C. Aortic Stenosis long standing hypothyroidism, it is important to begin
D. Pulmonary stenosis with small doses to avoid which of following
E. Tricuspid Stenosis complication
A. Exophthalmos
3. Regarding Jugular venous pulse: B. Acute renal failure
A. Measured as Cms of blood. C. Seizures
B. Measured as Cms of water. D. Hemolysis
C. Canon waves observed in mitral stenosis. E. Ischemic heart diseases
D. Position does not changes with respiratory phase
E. Pressure is measured in mm Hg clinically. 9. Which one of following drug of choice in SIADH
A. Demeclocycline
4. A 55-year-old male presented with chest pain which B. Vasopressin
is brought on by climbing stairs, after taking heavy C. Thiazide diuretics
meals and while riding on bicycle for one month. It is D. Chlorpropamide
relieved by rest. Which investigation will reveal best E. Loop Diuretics
A. VQ Scan
B. Upper Gl Endoscopy. 10. Regarding Diabetic patient
C. Electrocardiography. A. Should not take carbohydrate diet
D. Echocardiography. B. 30% calories can be obtained from saturated fat
E. Spinal radiography C. 50% calories are required from protein
D. 60% calories from carbohydrates.
5. Which of the following hormone has the greatest E. Protein should constitute 20-25% of calories
effect on basal metabolic rate?
A. Growth hormone 11. Primary Hypothyroidism is managed with:
B. Cortisol A. Methimazole
C. Somatostatin B. lodine replacement
D. Thyroxine C. Carbimazole
E. Parathormone D. Thyroxine
E. Corticosteroids
6. Primary Hypothyroidism is characterized by
A. Thin skin
B. Hypernatremia
C. Elevated TSH Level
D. Low LDI. cholesterol
E. low total cholesterol level
12. Microalbuminuria is present if: 17. Cutaneous leishmaniasis can be transmitted
A. Albumin: Creatinine ratio (ACE) 25-30 mg/mmol through the
creatinine A. Bite of an infective anopheles mosquito
B. Albumin : Creatinine ratio (ACE) > 30 mg/mmol B. Bite of an infective female sand flies
creatinine C. Bite of an infective male sand flies
C. Albumin: Creatinine ratio (ACE) 30-70 mg/mmol D. Bite of an infective male mosquito
creatinine E. Bite of an infective aedes mosquito
D. Albumin: Creatinine ratio (ACE) 45-40 mg/mmol
creatinine 18. Greatest risk for infection transmitted by blood
E. Albumin: Creatinine ratio (ACE) 1.5-20 mg/mmol transfusion is
creatinine A. Syphilis
B. Hepatitis B
13. Iron deficiency anemia commonly presents with the C. Hepatitis C
following feature. D. HIV
A. Bite cells E. Urinary tract infection
B. Foam cells
C. Howell-Jolly bodies 19. Drug of choice in treatment of amoebic hepatitis is:
D. Hypersegmented neutrophils A. Mebendazole
E. Pencil cells B. Albendazole
C. Metronidazole
14. Important Management step for G6PD deficiency D. Omeprazole
A. Corticosteroids E. Ciprofloxacin
B. Azathioprine.
C. Cyclophosphamide. 20. Appropriate antibiotic therapy for chlamydia
D. Enzyme supplements. pneumonia
E. Supportive A. Ampicillin
B. Erythromycin
15. 28 years female has presented with joint pains and C. Imipenem
easy fatigability for 8 months. O/E.: she has an anemic D. Amikacin
and palpable spleen of 2 cm. CBC is showing E. Cefuroxime
Hb.8Gm/dl, microcytosis with hypochromia. Serum Iron
is 40, TIBC 200, Ferritin 70 and saturation 20%. Anemia 21. Most specific investigation for Enteric fever on the
is most likely due to: 5th day of the infection
A. Iron deficiency A. typhidot test
B. Thalassemia B. Widal test
C. Sideroblastic anemia C. Blood Culture
D. Inflammation D. CBC
E. Leukemia E. Stool DR

16. Regarding iron deficiency anemia: 22. Which of the following DMARDs is the most
A. MCV is normal in first stage Hepatotoxic drug?
B. MCV is increased in the first stage. A. Leflunomide
C. MCV is decreased in the first stage. B. Methotrexate
D. Bone marrow is normal. C. Sulfasalazine
E. Reticulocyte count decrease with iron replacement D. Chloroquine
E. Azathioprine
23. Systemic sclerosis featured by 30. 25 year old teacher has presented with high grade
A. Renal Impairment with proteinuria and hypertensive continued fever, cough and right sided chest pain for 03
B. Z deformity of thumb weeks. Chest movements are reduced on the right side
C. Rheumatoid nodule with dull percussion below 4 intercostal space and
D. Scaly she rashes on extensor surface decreased vocal resonance. most likely she has:
E. Proximal myopathy A. Community acquired pneumonia
B. Non-resolving pneumonia.
24. Which of the following arthritis cause nail C. Pulmonary tuberculosis.
change/ nail erosion D. Pleural effusion.
A. Rheumatoid arthritis E. Empyema thoracic
B. Osteoarthritis
C. Gouty arthritis 31. 18 year normal male student, while playing.
D. Psoriatic arthritis developed chest pain and cyanosis, chest is
E. Seronegative arthritis hyperresonant on right side He has developed:
A. ARDS
25. SLE is more common B. Respiratory failure.
A. Female C. Pneumonia
B. Male D. Pneumothorax
C. Old female E. Pleural effusion
D. Young females
E. Old age 32. Glasgow coma scale:
A. Primarily devised for hemorrhagic stroke patients.
26. Treatment of choice for rheumatoid arthritis is B. Primarily devised for ischemic stroke patients
A. Steroids C. Primarily devised for head injury patients
B. NSAIDS D. Helpful for hepatic encephalopathy patients
C. DMARDS E. Useful for uremic encephalopathy.
D. Cyclosporine
E. Diuretics 33. Cardinal features of parkinson's disease are:
A. Rigidity, rest tremor, bradykinesia and gait
27. Rheumatoid arthritis is: Disturbance.
A. Erosive polyarthritis B. Rigidity, rest tremor, postural instability and galt
B. Erosive monoarthritis disturbance
C. Degenerative polyarthritis C. Rigidity, dysphagia, bradykinesia and gait
D. Erosive oligoarthritis disturbance.
E. Non erosive arthritis D. Rigidity, rest tremor, sleep disturbance and gait
disturbance.
28. Common pulmonary complications of SLE Include: E. Rigidity, mood disorders, bradykinesia and gait
A. Bronchogenic carcinoma disturbance.
B. Pneumothorax.
C. Stridor. 34. 50 year old male has presented with weakness of
D. Pulmonary nodules. right arm, right leg and difficulty in speech since 6
E. Pleural effusion. hours. O/E; pulse 90/min; blood pressure 130/96
mmHg. Right upper and lower limbs are having power
29. Transudative pleural effusions are commonly 0/5, reflexes exaggerated with up going planter. Eye
caused by: globes deviated to the right side. Speech is dysarthric.
A. Tuberculosis. Left side is normal. Most likely lesion is:
B. Pneumonia. A. Ischemic stroke
C. Hypothyroidism. B. SAH
D. Nephrotic syndrome. C. In the right cerebral cortex
E. Rheumatoid arthritis D. In the left internal capsule
E. Hemorrhagic in nature
35. 20 years male has presented with quadriparesis 41. The confirmatory investigation in the TB abdomen
since 2 days starting from lower limbs. Power is 1/5 and is?
reflexes are absent best management step is: A. Colonoscopy
A. I/V steroids in high dose B. Colonoscopy with biopsy
B. Leukapheresis C. Laparoscopic biopsy
C. I/V steroids in low dose D. Stool for occult blood
D. Plasmapheresis E. Sputum culture
E. Mechanical ventilation
42. Regarding child turcotte-pugh score used for
36. False, firm, unshakable beliefs which are resistant to prognosis of cirrhosis
logic or evidence to the contrary are called: A. Hepatic Encephalopathy is not included.
A. Delusions B. Serum unconjugated Bilirubin is included.
D. Hallucinations C. Serum Albumin is included.
D. Illusions D. Serum Ammonia level is included.
D. Obsessions E. Prothrombin time INR is not included.
E. Overvalued ideas
43. Neonates born to hepatitis B-infected mother should
37. Which of the following is treatment of choice for be managed with:
phobic anxiety disorders A. Hepatitis B surface antigen vaccine
A. Antipyretics B. Entecavir.
B. Benzodiazepines C. Tenofovir.
C. Behavior therapy D. Hepatitis B hyper immune globulin.
D. Mood stabilizers E. Hepatitis B surface antigen vaccine and hepatitis B
E. Tricyclic antidepressants hyper immune globulin

38. A patient presents with the complaints of Persistent 44. H. Pylori eradication therapy Includes:
re experiencing (flashback, nightmares), fairly constant A. Omeprazole amoxicillin 250 mg +Metronidazole
state of hyperarousal didact The diagnosis is: 400mg
A. Acute stress reaction B. Ranitidine + amoxicillin +Metronidazole 400mg 250
B. Agoraphobia mg
C. Obsessive compulsive disorder C. Omeprazole + clarithromycin 500mg +Metronidazole
D. Panic disorder 400mg
E. Post-traumatic stress disorder D. Omeprazole + bismuth +Metronidazole 400mg
E. Omeprazole + levofloxacin 250 mg+Metronidazole
39. Which of the inflammatory bowel disease is most 200mg.
commonly associated with Primary sclerosing
cholangitis? 45. 25 years male patient has presented with: HBV DNA
A. Ulcerative colitis 30000/miel, Anti HCV and Anti-HDV negative, US
B. Crohn's disease showing parenchymal changes. He needs
C. Microscopic colitis A. Adefovir for 16 weeks.
D. Irritable bowel syndrome B. Oral tenofovir for a year.
E. Lymphocytic colitis C. Interferon and ribavirin for 6 months.
D. Gamma interferon for 24 weeks.
40. Spontaneous Bacterial Peritonitis is managed with: E. Liver transplantation.
A. Benzylpenicillin 6 million units 6 hourly.
B. Cefotaxime 1gm OD. 46. Herpes zoster is commonly complicated with:
C. Cefotaxime 1gm 8 hourly. A. Post herpetic neuralgia.
D. Cefotaxime 2gm 8 hourly. B. Non healing ulcers
E. Clarithromycin + amoxicillin C. Encephalitis.
D. Aseptic meningitis
E. Pneumonia
47. Atopic eczema: 41 prof ….
A. Commonly Onset is in adult-hood.
B. Commonly Onset is in the first 2 years of life.
C. Usually skin is wet
D. Itching is absent.
E. Exudation is present

48. Renal biopsy is indicated in:


A. CKD with normal size kidneys.
B. CKD with small kidneys.
C. Solitary kidney.
D. Kidney <60% of predicted size.
E. Nephrotic syndrome in all children.

49. Renal artery stenosis is more commonly predicted


with:
A. Difficult to control hypertension.
B. Bilateral symmetrical kidneys.
C. Absence of peripheral vascular disease,
D. Absent renal bruit.
E. Improving renal function with ACE inhibitors.

50. Natural course of Hepatitis B in adults is:


A. 10% proceed to chronic hepatitis.
B. 30 % proceed to chronic hepatitis.
C. 50 % proceed to chronic hepatitis.
D. 70 % proceed to chronic hepatitis.
E. 90 % proceed to chronic hepatitis
40 Preprof 7. Patient receiving ATT, develops gout. Drug which
should be stopped?
1. An organism first produces lung abscess and later A. Isoniazid
meningitis. The organism is? B. Ethambutol
A. Staphylococcus aureus C. Pyrazinamide
B. Pneumococci D. Streptomycin
C. Streptococci E. Eritampucin
D. Mycobacterium
E. Chlamydia species 8. Characteristic feature of cl. Botulinum infection?
(Verify) A. Fever
B. Diarrhea
2. Most common cause of subacute bacterial C. Flaccid paralysis of respiratory muscle
endocarditis is? D. Projectile vomiting
A. Staph aureus E. Dysentery
B. Strep viridans
C. Strep pneumoniae 9. A chronic smoker presents with the hemoptysis on
D. Staph epidermiditis bronchoscopy there is a fungating mass in the right
E. Actinomycosis main bronchus. The most likely carcinoma in this case
would be?
3. The parasite associated with cholangiocarcinoma is? A. Squamous cell carcinoma
A. Clonorchis sinensis B. Small cell lung carcinoma
B. Leishmania donovani C. Bronchiolar carcinoma
C. Schistosoma japonicum D. Adenocarcinoma
D. Schistosoma mansoni E. Large cell carcinoma
E. Toxoplasma gondi
10. ?
4. The most common secondary infection in a case of
influenza in elderly person is due to? 11. Asbestosis various diseases in people working in its
A. Haemophilus influenza proximity. The lesion found is
B. Klebsiella pneumoniae A. Bronchogenic carcinoma
C. Pneumococci B. Mesothelioma
D. Staphylococci C. Pleural plaques
E. Streptococci D. Laryngeal carcinoma
(Verify) E. Pneumoconiosis

5. Epstein barr virus has a significant role in the 12. The following features are characteristic of states
development of? asthmaticus?
A. Burkitt's lymphoma A. Pulse rate of more than 60 beats/min
B. Granulocytic leukemia B. Rise in pulse volume during expiration
C. Kaposi sarcoma C. Harsh vesicular breathing
D. Scrotal carcinoma D. Gallop rhythm
E. Squamous cell carcinoma E. Respiratory rate of more than 30 min

6. A patient presents with itching (and some other 13. A pt of pulmonary th receiving anti tb drugs for two
symptoms) on workup found to have intestinal worm months develop pain and needles sensation in legs.
infestation. His blood workup will most likely show The drug most likely to cause is?
increase? A. Ethambutol
A. Neutrophils B. Isoniazid
B. Eosinophils C. Pyrazinamide
C. Lymphocytes D. Rifampicin
D. Monocytes E. Streptomycin
E. Basophils
14. Following substance predisposes to lung cancer? 21. The Incisura angularis represents which anatomical
A. Nickel location?
B. Beryllium A. The junction between the cardia and fundus
C. Coal tar fumes B. The junction between the cardia and body
D. Silica C. The junction between the body and pyloric part of the
E. Oxygen stomach
(Verify) D. The junction between the cardia and pyloric part of
the stomach
15. Normal iron absorption from duodenum and jejunum E. The junction between the fundus and pyloric part of
is? the stomach
A. 10%
B. 50% 22. Which of the following features best distinguishes
C. 100% Crohn's disease from ulcerative colitis?
D. 20% A. Oral ulcers
E. 85% B. Rectal bleeding
(Verify 10 to 30%) C. Continuous colonic involvement on endoscopy
D. Noncaseating granulomas
16. The lack of hydrochloric acid secretion from E. Crypt abscesses
stomach results in?
A. Delayed emptying of stomach 23. A patient with DVT is being treated with warfarin.
B. Hemolytic anemia Which one test would you use to monitor the drug level
C. Megaloblastic anemia in the serum?
D. Protein indigestion A. PT
E. Increased gastrin secretion B. APTT
C. Clotting time
17. Mallory bodies are characteristically present in? D. Bleeding time
A. Alcoholic hepatitis E. CBC
B. Alcoholic fatty liver
24. A child is suffering from recurrent abdominal pain as
C. Hepatitis B
well as pain in the fingers and toes. On examination, he
D. Wilson disease
is having mild splenomegaly. The hemoglobin type
E. Primary biliary cirrhosis
responsible for this is?
18. A 70-year-old male developed hepatoma. The A. Hb C
enzyme to be elevated is: B. Hb S
A. AFP C. Hb SC
B. CEA D. Hb A
C. HCG E. Hb B
D. SGOT
E. Alkaline phosphatase 25. Involvement of inguinal lymph nodes along with
splenic involvement in Hodgkin lymphoma is?
19. Hepatitis B-induced liver injury is due to: A. Stage 1
A. HBsAg B. Stage 2
B. HBcAg C. Stage 3
C. HBeAb D. Stage 4
D. HBcAb E. Stage 5
E. Unknown antigen
26. The Philadelphia chromosome is a feature of?
20. Which of the following vaccines is live-attenuated? A. CML
A. BCG B. Leukemia
B. Rabies C. Hodgkin lymphoma
C. Pertussis D. Lymphoma
D. Tetanus E. Anemia
E. Hepatitis B
27. Eosinophilia in peripheral blood is a characteristic 33. A 60-year-old man develops a tremor in his fingers.
of? The tremor is most pronounced when he reaches for his
A. Aplastic syndrome coffee cup or points to an object. Which component of
B. Cystic fibrosis the motor system is involved?
C. Polyarteritis nodosa A. Basal ganglia
D. Malaria B. Cerebellar hemisphere
E. Kala-azar C. Cerebellar vermis
(verify) D. Frontal eye field
E. Motor nucleus of the thalamus
28. Arterial thrombosis is characterized by?
A. Red color 34. In a lower motor neuron lesion, the following
B. Line of zahn develops?
C. Thrombophlebitis A. Spasticity
D. Development of lower extremities B. Hyperreflexia
E. Propagation along the flow of blood C. Babinski’s sign
D. Muscle wasting
29. Aplastic anemia is caused by? E. Hyperesthesia
A. Silver therapy
B. Phenylbutazone 35. The neurotransmitter of the nigrostriatal pathway to
C. CIN II the caudate nucleus and putamen is?
D. Probenecid A. GABA
E. Carbimazole B. Glycine
C. Dopamine
30. Vitamin B12 deficiency may result from? D. Serotonin
A. GERD E. Norepinephrine
B. Blind loop syndrome
C. Meckel’s diverticulum 36. Loss of position and vibration sense occurs due to a
D. Phenytoin treatment lesion of?
E. Aplastic anemia A. Descending corticospinal tract
B. Anterior white column of the spinal cord
31. In polycythemia rubra vera? C. Posterior white column of the spinal cord
A. Urokinase production is increased D. Anterior limb of the internal capsule
B. Splenomegaly is an uncommon feature E. Spinal lemniscus
C. Leukocyte alkaline phosphatase may be decreased
D. Oxygen saturation is high 37. Lumbar puncture is safely done at?
E. The platelet count may be raised A. L4-L5
B. L5-S1
32. Babinski’s sign will not be positive in? C. L2-L3
A. Sleep D. L1-L2
B. Alcoholic E. T12-L1
C. Infants
D. Lesions of corticospinal tracts 38. A 14-year-old girl has primary amenorrhea,
E. Disease of cerebellum well-developed breasts, secondary sexual
characteristics, slight pubic hair, blind-ending vagina,
and absent uterus. The chromosomal analysis will
reveal?
A. 45,X0
B. 45,X
C. 46,XY
D. 46,XX
E. 45,XXY
39. Which area of the breast is most commonly involved 45. Following is a premalignant lesion?
in carcinoma? A. Intradermal nevus
A. Upper inner quadrant B. Junctional nevus
B. Upper outer quadrant C. Condyloma
C. Subareolar region D. Dysplastic nevus
D. Lower inner quadrant E. Melanoma
E. Lower outer quadrant
46. Radiation affects causes injury to?
40. The most common tumor of the thyroid gland is? A. Cell membrane
A. Papillary carcinoma B. Nucleus
B. Follicular carcinoma C. Cytoplasm
C. Anaplastic carcinoma D. Mitochondria
D. Medullary carcinoma E. DNA
E. Lymphoma
47. Radiation exposure during infancy is linked to which
41. In Paget’s disease of bone, the following enzyme is one of the following carcinomas?
increased in blood? A. Breast
A. Alkaline phosphatase B. Melanoma
B. Acid phosphatase C. Thyroid
C. Serum calcium D. Lung
D. SGPT E. Colon
E. Creatine kinase
48. For every 100 mg/dL increase in glucose, the serum
42. The second most common cause of osteoporosis sodium falls by?
after old age is? A. 0.6 meq/L
A. Menopause B. 1.6 meq/L
B. Metastatic bone disease C. 2.6 meq/L
C. Immobility/decreased exercise D. 3.6 meq/L
D. Hyperparathyroidism E. 4.6 meq/L
E. Corticosteroids
49. The normal range of potassium is?
43. Insulin is inhibited by? A. 1.5–3.5 meq/L
A. Beta-blockers B. 3.5–5.5 meq/L
B. ACTH C. 5.5–7.5 meq/L
C. Glucagon D. 7.5–9.5 meq/L
D. Gastrin E. 10–11.5 meq/L
E. Acetylcholine
50. Giant cell arteritis occurs in?
44. A woman in the first trimester of pregnancy was A. Temporal arteritis
found to have a blood sugar level of 300mg. What B. Polyarteritis nodosa
would be the most appropriate treatment for her? C. Wegener’s granulomatosis
A. Sulfonylurea D. Takayasu arteritis
B. Acarbose E. Hypersensitivity vasculitis
C. Acetylcholine
D. Insulin 51. A patient is having an irregularly irregular pulse on
E. Biguanides ECG, there are no P waves, and the QRS is normal.
The diagnosis is?
A. Atrial fibrillation
B. Atrial flutter
C. Wolff-Parkinson-White Syndrome
D. Atrial tachycardia
E. Atrial ectopic beats
52. A peptide that can increase blood pressure acutely 58. Regarding neuroleptic malignant syndrome?
and cause hypokalemia chronically is? A. It occurs when haloperidol is used with anesthesia
A. Angiotensin II B. It is associated with hypothermia
B. Atrial natriuretic factor C. It occurs due to dantrolene
C. Desmopressin D. It is not a medical emergency
D. Endorphin E. Rhabdomyolysis occurs
E. Oxytocin
59. A 55-year-old man has had an arterial blood
53. ECG shows a long PR interval and otherwise a clue pressure of 150/100 mmHg for several years. He
less normal ECG. This is? suddenly develops a severe headache caused by
A. Bradycardia spontaneous subarachnoid hemorrhage. What is the
B. 1st degree heart block diagnosis?
C. Normal A. Adult poststreptococcal glomerulonephritis
D. Tachycardia B. Adult polycystic kidney disease
E. Need for cardioversion C. Childhood polycystic kidney disease
D. IgA nephropathy
54. JVP is differentiated from carotid pulse? E. Hypertension
A. Does not change with abdominal pressure
B. Does not change with posture 60. Renal carcinoma spreads early by which of the
C. Does not change with respiration following routes?
D. Pulsation varies with inspiration A. Renal artery
E. Prominent outward movement B. Local spread
C. Renal vein
55. A patient is found to have a mid-diastolic murmur, D. Nerves
rumbling in character at the apex. The underlying cause E. Lymphatics
is?
A. Aortic stenosis 61. Microscopic hematuria is the presentation along with
B. Pulmonary stenosis casts and albumin most probable the case is that of?
C. Mitral regurgitation A. UTI
D. VSD B. Kidney stone
E. Mitral stenosis C. Urethritis
D. Glomerulonephritis
56. A patient is brought to the emergency department E. Cystitis
with a history of chest pain and dyspnea. The ECG
showed persistent ST elevation. The diagnosis is? 62. The cause of selective proteinuria is?
A. Non-ST elevation myocardial infarction A. Post-streptococcal glomerulonephritis
B. Q-wave myocardial infarction B. Minimal change glomerulonephritis
C. Acute myocardial infarction C. Membranous glomerulonephritis
D. Pericarditis D. Crescentic glomerulonephritis
E. Unstable angina E. Chronic glomerulonephritis

57. The most common antidepressant used in nocturnal 63. A patient with burns covering 60% of their body
enuresis is? surface area is admitted. The following should be kept
A. Haloperidol in mind in the assessment of their condition?
B. Alprazolam A. Hyperkalemia
C. Lithium B. Hypokalemia
D. Imipramine C. Hypercalcemia
E. Fluoxetine D. Hypocalcemia
E. Hyponatremia
64. You are asked to review the peripheral blood smear 70. A chronic smoker presents with hemoptysis. On
from a patient with anemia. The lactate dehydrogenase bronchoscopy, there is a fungating mass in the right
is elevated, and there is hemoglobinuria. This patient is main bronchus. The most likely carcinoma in this case
likely to have which physical examination finding? would be?
A. Goiter A. Squamous cell carcinoma
B. Heme-positive stools B. Small cell lung carcinoma
C. Mechanical second heart sound C. Bronchiolar carcinoma
D. Splenomegaly D. Adenocarcinoma
E. Thickened calvarium E. Large cell carcinoma

65. An organism first produces lung abscess and later 71. A child is suffering from recurrent abdominal pain as
meningitis. The organism is? well as pain in the fingers and toes. On examination, he
A. Staphylococcus aureus is having mild splenomegaly. The hemoglobin type
B. Pneumococci responsible for this is?
C. Streptococci A. Hb C
D. Mycobacterium tuberculosis B. Hb S
E. Chlamydia species C. Hb CS
D. Hb A
66. A soldier presented with a 3-month history of fever, E. Hb E
bilateral lymphadenopathy, hepatosplenomegaly,
anemia, and fatigue. He has a history of sleeping on the 72. Relevant to asthma, the most appropriate cytokine
floor and being bitten by a fly. The diagnosis is? is?
A. Kala-azar A. IL-3
B. CML B. IL-5
C. Filariasis C. IL-18
D. Schistosomiasis D. IL-4
E. Malaria E. IL-6

67. The most common cause of subacute bacterial 73. Which lung function test is diagnostic of asthma?
endocarditis is? A. FEV1
A. Staphylococcus aureus B. FVC
B. Streptococcus viridans C. CFRC
C. Streptococcus pneumoniae D. VC
D. Staphylococcus epidermidis E. TLC
E. Actinomycosis
74. Asbestosis causes various diseases in people
68. A patient with DVT is being treated with heparin. working in its proximity. The lesion found is?
What one test would you use to monitor the heparin A. Bronchogenic carcinoma
level in the serum? B. Pleural plaque
A. PT C. Laryngeal carcinoma
B. APTT D. Pneumoconiosis
C. Clotting time E. Mesothelioma
D. Bleeding time
E. CRP

69. Philadelphia chromosome is a feature of?


A. CML
B. Leukemia
C. Hodgkin lymphoma
D. Lymphoma
E. Anemia
40 Prof (MEDICINE 1) 14. Uncontrolled HTN
Ans Lateral apex beat
1. Regarding Hyperkalemia
Ans Tall/Tented P waves 15. Diagnostic test for pulmonary TB
Ans AFB sputum
2. Reg minimal change disease
Ans Good response to glucocorticoids 16. Iron def
Ans common in pregnancy
3. A patient is found to have a mid-diastolic murmur,
rumbling in character at the apex. The underlying cause 17. Iron therapy is monitored by
is? Ans Ferritin
A. Aortic stenosis
B. Pulmonary stenosis 18. vesicular rash on lip and mouth
C. Mitral regurgitation Ans Herpes simplex
D. VSD
E. Mitral stenosis 19. scenario of rash afria patchie
Ans immune thrombocytic purpura
Harsh mid diastolic murmur
Ans Mitral stenosis 20. Intestinal amebiasis
Ans liver abscess
4. Atrial fibrillation
Ans Irregularly irregular pulse 21. Reg interstitial lung disease
Ans Decreased chest expansion
5. scenario diabetic pt pain radiating to left arm relieved
by rest 22. Exudative pleural effusion
Ans Angina pectoris Ans hypothyroidism?

6. pansystolic murmur radiates to axilla 23. Scenario water diarrhea then anuria
Ans MR Ans Acute renal failure

7. Investigation in MI 24 Malaria transmitted by


Ans ECG Ans Female anopheles mosquito

8. JVP 25. In hypertension


Ans 3cm above stenal Ans LVH

9. Treatment of anemia in CRF 26. Risk factor of chronic bronchitis


Ans Erythropoietin Ans smoking

10. Bone Disorder CRF 27. Psychiatric scenario


Ans Due to vit D3 def Ans hydroschondic

11. Nephrotic syndrome 28. Psychiatric scenario


Ans Hypercholesterolemia generalized edema Ans Generalized anxiety
hypoalbuminemia
29. Scenario of rashes afebrile
12. Teacher scenario Ans aplastic anemia
Ans Pneumonia
30. scenario rashes hepatomegaly
13. Scenario Ans leukemia
Ams pulmonary TB with plural effusion
31. Treatment of severe of pneumonia 10. Main stay nt of autoimmune hepatitis
Ans clarithromycin +cl amolic Ans Corticosteroids

32. Regarding asthma 11. Hyponatremia is mainly seen in


Ans Usually occurs in childhood Ans Addison disease

33. Scenario of not opening his mouth 12. Scenario meningitis


Ans Tetanus immunoglobulin Ans CSF DR gram stain

34. Prevention of rabies 13. Scenario of TB meningitis rx


Ans Active-immunization ? Ans antituberculosis drugs
Note Rabies can be prevented by vaccination of Host.
14. Gastritis is caused by
35 Cause of hyponatremia Ans NSAIDS
Ans Excess water intake
15. Acute gouty arthritis rx
(MEDICINE 2) Ans diclofenac sodium

1. Screening test for SLE is: 16. Diarrhea defined as


A. Anti CCP Ans Stool more than 200 gm
B. Antinuclear antibodies
C. Anti-Jo antibody 17. Rx of organophosphorus poisoning
D. RA Factor Ans atropine pralidoxime +supportive
E. Anti-SCL. 70
18. Rx of choice for generalized seizure
2. Scenario of RA Ans Sodium valproate
Ans Anti Ccp
19. Modifiable factor for stroke
3. Scenario lower limb paralysis then upper rx is Ans Smoking
Ans plasmapheresis
20. contraindication to LP
4. Ischemic stroke within 2hr Ans Raised intracranial pressure
Ans Thrombolysis
21. Incomplete RNA hep virus
5. koilonychia Ans Hep D
Ans Spoon shaped nails
22. A woman in the first trimester of pregnancy was
6. Spontaneous Bacterial Peritonitis is managed with: found to have a blood sugar level of 300mg. What
A. Benzylpenicillin 6 million units 6 hourly. would be the most appropriate treatment for her?
B. Cefotaxime 1gm OD. A. Sulfonylurea
C. Cefotaxime 1gm 8 hourly. B. Acarbose
D. Cefotaxime 2gm 8 hourly. C. Acetylcholine
E. Clarithromycin + amoxicillin D. Insulin
E. Biguanides
7. Hypothyroidism
Ans High TSH Low T4 Gestational DM rx
Ans Insulin
8. Scabies caused by
Ans Mites

9. Gold standard for h pylori


Ans Microbiological culture
23. A 25-year-old female has gestational amenorrhea
for the last three months and signs & symptoms of
Grave’s disease. Which of the following is the best
treatment?
A. Radioactive iodine
B. Propylthiouracil
C. Methimazole/Neomercazole
D. Thyroidectomy
E. Propranolol
(In key its B)

Hyperthyroidism pregnancy rx
Ans PTU

24. Reg sjogren syndrome


Ans occur blw 40 to 60 of age

25. Rx of oral candidiasis


Ans Nystatin

26. Hypoglycemia defined as


Ans less than 63 glucose

27. HB DNA 3000 hep B dx rx is


Ans Tenofovir for 12 month

28. Normal palpable liver span is


Ans 8 to 12 cm

29. Most common cause of spinal cord compression


Ans metastasis ??
Osteoarthritis is most common

30. Rx of sulfonylurea induced hypoglycemia is


Ans Isotonic glucose sol

31. Mainstay treatment of addison disease


Ans corticosteroids

32. 50 yr man alternative constipation blood weight loss


Ans Tb abdomen

33. Mucus blood in stool low grade fever


Ans bacillary dysentery

34. Dose of RSV


Asn 100 or 55 ml

35. Hyperpyrexia
Ans 104 or 106.7 (both are written on net)
39 (Mid Term test) 7. A patient arrives at the emergency department with
severe thirst, dehydration, and confusion. The patient is
1. Drug used in the treatment of Cushing's syndrome: breathing rapidly and has a fruity odor. His blood sugar
A. Insulin is 578 mg/dL, osmolality of 100 mos, and a pH of 7.3.
B. Prednisolone The likely diagnosis is:
C. Ketoconazole A. Diabetic Ketoacidosis
D. High-dose multivitamin D B. Hyperosmolar Hyperglycemic Nonketotic Coma
E. Synthetic ACTH C. Hypoglycemia
D. Diabetic Neuropathy
2. Plasma urea/creatinine ratio of 20:1 is seen in:
A. Rhabdomyolysis 8. Palpitation can occur in:
B. Ureteric calculi A. Tachycardia
C. Pre-renal failure B. Bradycardia
D. Chronic glomerulonephritis C. Normal Heart Rate
E. Congestive cardiac failure D. Complete Heart Block
E. All of the above
3. Screening test for SLE is:
A. Anti ds-DNA 9. Out-patient screening methods for confirming the
B. Antinuclear antibodies presence of Cushing's syndrome include:
C. Anti-Smith antibody A. Overnight dexamethasone test
D. RA Factor B. 24-hour urinary free cortisol test
E. Anti-histone antibody C. Insulin tolerance test
D. 48-hour low dexamethasone test
4. Following is the significant diagnostic finding in E. Random cortisol measurements
Chronic Liver Disease:
A. Anemia 10. A 14-year-old girl has a 3-month history of a
B. Thrombocytosis progressively enlarging mass in the thyroid lobe. She
C. Leukocytosis has developed hoarseness of voice over the past 2
D. Neutropenia weeks. Physical examination reveals a 3 cm firm mass
E. Thrombocytopenia in the left upper lobe of the thyroid gland. There is no
associated lymphadenopathy. Of the following, the
5. Acute Hepatitis is more common due to: MOST helpful test for establishing the diagnosis is:
A. Hepatitis E virus A. Fine-needle aspiration biopsy of the thyroid gland
B. Hepatitis D virus B. Measurement of the thyroid-stimulating hormone
C. Hepatitis B virus level
D. Autoimmunity C. Measurement of the thyroxine level
E. Plasmodium falciparum D. Radionuclide scan of the thyroid gland
E. Ultrasonography of the thyroid gland
6. Which of the inflammatory bowel diseases is most
commonly associated with sclerosing cholangitis? 11. Following is a DNA virus:
A. Ulcerative colitis A. Hepatitis A virus
B. Crohn's disease B. Hepatitis B virus
C. Microscopic colitis C. Hepatitis C virus
D. Irritable bowel syndrome D. Hepatitis D virus
E. Lymphocytic colitis E. Hepatitis E virus
12. Following are the three most common symptoms of 18. No effective vaccination is available for:
diabetes mellitus: A. Hepatitis C virus
A. Headache, Vomiting & Fatigability B. Hepatitis B virus
B. Polydipsia, increased urination & weight loss C. Hepatitis A virus
C. Double vision, infections & increased heart rate D. Steatohepatitis
D. Tingling of feet, weight loss & delayed healing of E. Autoimmune hepatitis
wounds
E. Increased appetite, vertigo & vomiting 19. Diabetes mellitus may occur as a secondary
consequence of:
13. A 55-year-old male presented with chest pain, which A. Addison's Disease
is brought on by climbing stairs and relieved by rest. B. Albinism
What is the most likely cause of this condition? C. Chronic Pancreatitis
A. Pulmonary Embolism D. Hyperthyroidism
B. Esophageal spasm E. Wilson's Disease
C. Peptic ulcer disease
D. Angina pectoris 20. Which of the following is a non-modifiable risk factor
E. Bacterial pneumonia for angina pectoris?
A. Hypertension
14. Regarding Ulcerative colitis, the first-line therapy for B. Diabetes Mellitus
mild rectal disease is: C. Old Age
A. Steroids D. Smoking
B. Immunosuppressants E. Obesity
C. Biological agents
D. 5-Aminosalicylates 21. Most diagnostic investigation for acute pancreatitis
E. Surgery on the 4th day of onset is:
A. Blood counts
15. Which antibody is present in drug-induced SLE? B. Serum amylase
A. Anti dsDNA C. Serum lipase
B. Antinuclear antibodies D. X-ray abdomen
C. Anti-Smith antibody E. Ultrasound abdomen
D. Anti-histone antibody
22. Which type of anemia is included in the diagnostic
16. A 28-year-old male met with an accident and criteria for SLE?
sustained severe crush injury. He is most likely to A. Microcytic anemia
develop: B. Megaloblastic anemia
A. Acute renal failure C. Autoimmune hemolytic anemia
B. Hypercalcemia D. Sideroblastic anemia
C. Hyponatremia E. Iron deficiency anemia
D. Acute myocardial infarction
E. Fulminant hepatic failure 23. Following is a feature of pre-renal azotemia:
A. Fractional excretion of Na > 1%
17. The following is a recognized feature of primary B. Urinary osmolality < 500 mosm/kg
hyperparathyroidism: C. Urinary sodium concentration > 40 meq/L
A. Renal colic D. Not reversed with replacement fluids
B. Shortened metacarpals E. Commonly affects glomerulus
C. Polyuria (Wrong)
D. Constipation
E. Hyperchloremic acidosis
24. In pituitary damage, which of the following 31. Following is the most common nephrotoxic drug:
hormones will be lost last? A. Penicillin
A. ACTH B. Amlodipine
B. GH C. Gentamicin
C. FSH D. Ascorbic acid
D. LH E. Ranitidine
E. TSH
32. Which of the following is not a cause of nephrogenic
25. Following is a parameter of Child-Pugh diabetes insipidus?
classification: A. Demeclocycline
A. Anemia B. Lithium
B. Serum conjugated bilirubin C. Multiple myeloma
C. Prothrombin time D. Sickle cell anemia
D. Pedal edema E. Anoxic encephalopathy
E. GCS
33. A 25-year-old female has gestational amenorrhea
26. Wilson disease is managed with: for the last three months and signs & symptoms of
A. Beta interferon Grave’s disease. Which of the following is the best
B. Lamivudine treatment?
C. Alpha interferon A. Radioactive iodine
D. Penicillamine B. Propylthiouracil
E. Corticosteroids C. Methimazole/Neomercazole
D. Thyroidectomy
27. The first-line treatment for diabetes is: E. Propranolol
A. Diet & exercise (In key its B)
B. Diet, exercise & metformin
C. DPP4 inhibitors 34. A 22-year-old female student has developed joint
D. Diet, exercise & insulin pains, anorexia, amenorrhea, and upper abdominal pain
E. Glibenclamide, metformin & insulin for 10 months. She is jaundiced, has acne on her face,
and the liver is palpable 3 cm with a total span of 15 cm.
28. Based on the recent findings in infective Spleen is palpable for 2 cm. Pedal edema and shifting
endocarditis, the two most important parameters for the dullness are absent. Most likely, she has:
diagnosis of this infection are: A. Chronic hepatitis B
A. Laboratory abnormalities & positive blood culture B. Chronic hepatitis C
B. Positive blood culture & capillary changes C. Cirrhosis liver
C. ECG changes & positive physical findings D. Autoimmune hepatitis
D. Positive physical findings & positive blood culture E. Hemochromatosis
E. Laboratory abnormalities & ECG changes
35. The secretion of which of the following is inhibited
29. Antibiotics-associated colitis is diagnosed by: by somatostatin?
A. Blood culture A. Dopamine
B. Raised C-reactive protein B. Glucagon
C. Stool detailed report C. Sympathetic nervous system
D. Stool culture D. Somatostatin
E. Stool toxins E. Norepinephrine

30. Classical triad of Reiter's syndrome is:


A. Arthritis, Conjunctivitis, Urethritis
B. Anemia, Arthritis, Urethritis
C. Anemia, Rash, Arthritis
D. Rash, Conjunctivitis, Urethritis
E. Anemia, Rash, Urethritis
36. During chest pain, the most important and early 42. A 20-year-old male presented with fever, chest pain,
required investigation is: arthritis, and skin rash. The patient also has a history of
A. Electrocardiogram sore throat two months back. On examination, early
B. Cardiac enzymes diastolic murmur on the aortic area is present. ECG
C. Echocardiography shows prolonged PR interval. On investigation, ASO is
D. Thallium scan raised. What is the diagnosis?
E. Exercise tolerance test (ETT) A. SLE
B. RA
37. Nocturnal diarrhea is a feature of all of the following C. Acute rheumatic fever
except: D. Aortic regurgitation
A. Irritable bowel syndrome E. Infective endocarditis
B. Crohn's colitis
C. TB abdomen 43. Postural hypotension is more commonly associated
D. Colon carcinoma with:
E. Diabetic gastroparesis A. Irritable bowel syndrome
B. Crohn’s colitis
38. Backache in ankylosing spondylitis does not C. TB abdomen
improve: D. Colon carcinoma
A. At rest E. Diabetic gastroparesis
B. During exercise
C. With badminton 44. The confirmatory investigation in celiac disease is:
D. In swimming A. Presence of Howell-Jolly bodies
E. With jogging B. Anti-TTG antibodies
C. Colonic biopsy
39. Following is a feature of chronic renal failure: D. Jejunal biopsy
A. Hypercalcemia E. Serum IgA level
B. Hyperphosphatemia
C. Hypokalemia 45. Radiological finding of bamboo spine is present in:
D. Hyponatremia A. Rheumatoid arthritis
E. Hypouricemia B. SLE
C. Psoriatic arthritis
40. Which of the following is not a feature of Cushing's D. Ankylosing spondylitis
syndrome? E. Osteoarthritis
A. Easy bruising
B. Proximal myopathy 46. RA factor is absent in:
C. Hypertension A. Rheumatoid arthritis
D. Psychosis B. SLE
E. Peripheral neuropathy C. Infective endocarditis
D. Polymyositis
41. Regarding diabetes mellitus: E. Psoriatic arthritis
A. Type I DM is commonly seen in persons of about 40
years 47. Regarding thyrotoxicosis, which of the following is
B. Type II DM tends to occur in obese persons not correct:
C. In Type I DM, oral hypoglycemic drugs are effective A. Serum thyroxine (T4) concentration may be normal
D. Serum insulin level is absolutely absent in type II DM B. When it occurs as a symptom of Grave's diseases, a
E. HbA1c is a good monitor for the diagnosis of high serum TSH is responsible for thyroid activity.
diabetes mellitus C. The patient may be lethargic & depressed
D. Palmar erythema could be a feature
E. Onycholysis may occur
48. The thyroid gland produces the following hormones
except:
A. Thyroglobulin
B. Tetra-iodothyronine (T4)
C. Tri-iodothyronine (T3)
D. Mono-iodothyronine
E. Calcitonin

49. A 45-year-old male presented with fatigue,


breathlessness, and palpitation. On examination, the
apex beat is palpable in the 5th ICS, tapping in
character. S1 is loud with a mid-diastolic murmur at the
mitral area. What is the diagnosis?
A. Atrial myxoma
B. Mitral stenosis
C. Aortic stenosis
D. Mitral regurgitation
E. Pericarditis

50. A 18-year-old male student has developed body


aches, anorexia, and upper abdominal pain for 17
months. He is jaundiced, with a liver palpable 3 cm
below the costal margin and a total span of 15 cm. The
spleen is palpable for 2 cm. Pedal edema and shifting
dullness are absent. Anti-HCV is negative, anti-HDV is
negative, and HBV DNA is detected. The best
management step is:
A. Supportive management
B. Vaccination for Hepatitis B virus
C. Omeprazole 40 mg OD
D. Tenofovir 300 mg OD
E. Entecavir 300 mg OD
https://drive.google.com/drive/folders/1AC1yc7EPfVcuQ
TUevzfhMIvnQwmzsduy

https://drive.google.com/drive/folders/1622Cc8Bh8VOxh
D_2LLdth3OpxEn_aBdx

You might also like